You are on page 1of 201

CRIMINAL LAW I – DIGESTED CASES BY: BOAQUINA, MARY CRISCHAEL

DIGESTED CASES IN
CRIMINAL LAW I

BOAQUIÑA, Mary Crischael C.


5:30-8:30PM Friday FCJ303
Atty. Jedrek C. Ng

1
CRIMINAL LAW I – DIGESTED CASES BY: BOAQUINA, MARY CRISCHAEL
2

General Principles

PEOPLE V. SANTIAGO
G.R. 17584. March 8, 1922

FACTS: The accused was driving an automobile at the rate of 30 miles an hour on a highway 6
meter wide, notwithstanding the fact that he had to pass a narrow space, the appellant did not
take the precaution and did not proceed with vigilant care that under the circumstances an
ordinary prudent man would take in order to avoid possible accidents that might occur, as
unfortunately did occur, his automobile ran over a 7 years old boy, Porfirio Parondo, who was
killed instantly as a result of the accident.
He prosecuted for the crime of homicide by reckless negligence. Not agreeable with the
sentence, he comes to court alleging that the Court erred in not taking judicial notice of the fact
that the appellant was being prosecuted under Act No. 2886 of the PH Legislature and that the
Act is unconstitutional and gave no jurisdiction in this case.

ISSUE: Whether or not Act No. 2886 is valid and constitutional.

HELD: Yes. Act No. 2886 is valid and is not violative of any constitutional provisions.

This Act is attacked on account of the amendments that it introduces in General Orders No. 58,
the defense arguing that the Philippine Legislature was, and is, not authorized to amend General
Orders No. 58, as it did by amending section 2 thereof because its provisions have the character
of constitutional law. Said section 2 provides as follows:
All prosecutions for public offenses shall be in the name of the United States against the
persons charged with the offenses. (G. O. No. 58, sec. 2 ).
Act No. 2886, which amends it, by virtue of which the People of the Philippine Island is made the
plaintiff in this information, contains the following provisions in section 1:
SECTION 1. Section two of General Orders, Numbered Fifty-eight, series of nineteen hundred,
is hereby amended to read as follows:
"SEC. 2. All prosecutions for public offenses shall be in the name of the People of the Philippine
Islands against the persons charged with the offense."

The provisions of Act No. 2886 does not have the character of constitutional law. The provisions
of the General Order No. 58, which was amended by Act No. 2886, have the character of statutory
law, the power of the Legislature to amend it is self-evident, even if the question is considered
only on principle.

2
CRIMINAL LAW I – DIGESTED CASES BY: BOAQUINA, MARY CRISCHAEL
3

US V. PABLO
G. R. No. L-11676. October 17, 1916

FACTS: Andres Pablo, a policeman, went by order of his chief to the barrio of Tuyo to raid a
jueteng game; but before he arrived there the players left and ran away; however, on his arrival
at a vacant lot he found Francisco Dato, and a short distance away, a low table. He also found
thereon a tambiolo (receptacle) and 37 bolas. He also saw Maximo Malicsi and Antonio Rodrigo
escaped but refrained from arresting them and only arrested Francisco Dato. Andres Pablo
testified under oath and narrated the incident.

Provincial Fiscal charged Andres Pablo with crime of perjury. Andres Pablo falsely testified under
oath in the justice of peace court saying he had not seen the alleged gambles Maximo Malicsi
and Antonio Rodrigo in consideration for P15 he received as bribe.

ISSUE: Whether or not the respondent is guilty of the crime of perjury or false testimony under
Art. 318 to 324 of the Revised Penal Code.

HELD: YES. The said articles of the Penal Code are in force and are properly applicable to crimes
of false testimony.

In consideration of the fact that in the case at bar the evidence shows it to have been duly proven
that the defendant, Andres Pablo, in testifying in the cause prosecuted for gambling at jueteng,
perverted the truth, for the purpose of favoring the alleged gamblers, Maximo Malicsi and
Antonio Rodrigo, with the aggravating circumstance of the crime being committed through
bribery, for it was also proved that the defendant Pablo received P15 in order that he should
make no mention of the said two gamblers in his sworn testimony, whereby he knowingly
perverted the truth, we hold that, in the commission of the crime of false testimony, there
concurred the aggravating circumstance of price or reward, No. 3 of article 10 of the Code, with
no mitigating circumstance to offset the effects of the said aggravating one; wherefore the
defendant has incurred the maximum period of the penalty of arresto mayor in its maximum
degree to prision correccional in its medium degree, and a fine.

3
CRIMINAL LAW I – DIGESTED CASES BY: BOAQUINA, MARY CRISCHAEL
4

MAGNO V. CA
G.R. No. 96132. June 26, 1992

FACTS: Magno, was in the process of putting up a car repair shop but did not have complete
equipment and lacked funds. He approached Corazon Teng for his needed car repair service
equipment. Mancor was willing and able to supply the pieces of equipment needed if LS Finance
could accommodate petitioner and provide him credit facilities. The arrangement went through
on condition that petitioner has to put up a warranty deposit of 30% of the total value of the
pieces of equipment to be purchased. Magno could not come up with such amount, so he
requested Joey Gomez to look for someone who could lend him the equivalent amount of the
warranty deposit. However, unknown to him, it was Corazon Teng who advanced the deposit in
question, on condition that the same would be paid as a short term loan at 3% interest.

The equipment were delivered to petitioner who in turn issued a postdated check and gave it to
Joey Gomez. When the check matured, petitioner requested through Joey Gomez not to deposit
the check as he was no longer banking with Pacific Bank. To replace the first check issued, he
issued six postdated checks. Two checks were deposited and cleared while the four were held
momentarily by Corazon Teng, on the request of Magno as they were not covered with sufficient
funds. Subsequently, he could not pay LS Finance the monthly rentals, thus it pulled out the
garage equipments. Upon knowing that Corazon Teng was the one who advanced the warranty
deposit he went to see Corazon Teng and promised to pay but failed to do so.

ISSUE: Whether or not Magno is guilty for the violation of BP Blg. 22.

HELD: No. Petitioner did not even know that the checks he issued were turned over by Joey
Gomez to Mrs. Teng. This fact evoke suspicion that the transaction is irregular and immoral. It
unfolds the kind of transaction that is shrouded with mystery, gimmickry and doubtful legality.
This modus operandi has in so many instances victimized unsuspecting businessmen, who
likewise need protection from the law, by availing of the deceptively called "warranty deposit"
not realizing that they also fall prey to leasing equipment under the guise of a lease-purchase
agreement when it is a scheme designed to skim off business clients. For all intents and purposes,
the law was devised to safeguard the interest of the banking system and the legitimate public
checking account user. It did not intend to shelter or favor nor encourage users of the system to
enrich themselves through manipulations and circumvention of the noble purpose and objective
of the law. There is no doubt that petitioner's four (4) checks were used to collateralize an
accommodation, and not to cover the receipt of an actual "account or credit for value" as this
was absent, and therefore petitioner should not be punished for mere issuance of the checks in
question. Magno was acquitted.

4
CRIMINAL LAW I – DIGESTED CASES BY: BOAQUINA, MARY CRISCHAEL
5

ROMUALDEZ V. COMELEC
G.R. No. 167011 April 30, 2008

FACTS: This is a complaint against Spouses Romualdez for making false information as to their
residence in their applications as new voters in Burauen, Leyte, violating the OEC and RA 8189 or
Voter’s Registration Act of 1996. The complainant-affidavit requested the COMELEC to conduct
preliminary investigation, and if the evidence so warrants, the corresponding Information against
petitioners be filed before the RTC for the prosecution of the same.
Spouses Romualdez said that they intend to reside in Burauen, Leyte since 1989 and that they
took actual residence in Burauen by leasing the house of Renomeron for 5 years.

ISSUE: Whether or not the right to due process was violated.

HELD: No. In the first place, there appears to be no incongruity between the charges as contained
in the Complaint-Affidavit and the Information filed before the RTC, notwithstanding the
denomination by private respondent of the alleged violations to be covered by Section 261(y)(2)
and Section 261(y)(5) of the Omnibus Election Code and Section 12 of Republic Act No. 8189.
Evidently, the Information directed to be filed by the COMELEC against petitioners, and which
were, in fact, filed with the RTC, were based on the same set of facts as originally alleged in the
private respondent’s Complaint-Affidavit.

In Lacson, this court underscored the elementary rule that the jurisdiction of a court is
determined by the allegations in the Complaint or Information, and not by the evidence
presented by the parties at the trial. Indeed, it was articulated that the real nature of the criminal
charge is determined not from the caption or preamble of the Information nor from the
specification of the provision of law alleged to have been violated, they being conclusions of law,
but by the actual recital of facts in the Complaint or Information.
Petitioners’ reliance on Lacson, however, does not support their claim of lack of due process
because the charges contained in the Complaint-Affidavit and the charges as directed by the
COMELEC to be filed are based on the same set of facts. Petitioners cannot claim that they were
not able to refute or submit documentary evidence against the charges that the COMELEC filed
with the RTC. Petitioners were afforded due process because they were granted the opportunity
to refute the allegations in private respondent’s Complaint-Affidavit.

Petitioners were reasonably apprised of the nature and description of the charges against them.
It likewise bears stressing that preliminary investigations were conducted whereby petitioners
were informed of the complaint and of the evidence submitted against them. They were given
the opportunity to adduce controverting evidence for their defense. In all these stages,
petitioners actively participated.

5
CRIMINAL LAW I – DIGESTED CASES BY: BOAQUINA, MARY CRISCHAEL
6

SOUTHERN HEMISPHERE ENGAGEMENT NETWORK V. ANTI-TERRORIST COUNCIL


G.R. NO. 178552. October 5, 2010

FACTS: Petitioners assailed the definition of terrorism under RA 9372 (the Human Security Act of
2007) claiming it is vague and broad, in that terms like “widespread and extraordinary fear and
panic among the populace” and “coerce the government to give in to an unlawful demand” are
nebulous, leaving law enforcement agencies with no standard to measure the prohibited acts.
Petitioner-organizations assert locus standi on the basis of being suspected “communist fronts”
by the government, whereas individual petitioners invoke the “transcendental importance”
doctrine and their status as citizens and taxpayers.

ISSUE: Whether or not the Human Security Act of 2007 be facially challenged on the grounds of
vagueness and overbreadth doctrines?

HELD No. A facial invalidation of a statute is allowed only in free speech cases, wherein certain
rules of constitutional litigation are rightly excepted. Facial challenge is allowed to be made to a
vague statute and to one which is overbroad because of possible “chilling effect” upon protected
speech. The possible harm to society in permitting some unprotected speech to go unpunished
is outweighed by the possibility that the protected speech of others may be deterred and
perceived grievances left to fester because of possible inhibitory effects of overly broad statutes.
This rationale does not apply to penal statutes. Criminal statutes have general in terrorem effect
resulting from their very existence, and, if facial challenge is allowed for this reason alone, the
State may well be prevented from enacting laws against socially harmful conduct. In the area of
criminal law, the law cannot take chances as in the area of free speech.

6
CRIMINAL LAW I – DIGESTED CASES BY: BOAQUINA, MARY CRISCHAEL
7

ESTRADA V. SANDIGANBAYAN
G.R. No. 148560. November 19, 2001

FACTS: Estrada, the highest-ranking official to be prosecuted under the Crime of Plunder, wishes
to impress the Court that the assailed law is so defectively fashioned that it crosses that thin but
distinct line which divides the valid from the constitutionality infirm. That there was a clear
violations of the fundamental rights of the accused to due process and to be informed of the
nature and cause of the accusation.

ISSUE: Whether or not Plunder as defined in RA 7080 is a malum prohibitum.

HELD: No. It is Mala In Se. The acts punished are inherently immoral or inherently wrong, they
are mala in se and it does not matter that such acts are punished in a special law, especially since
in the case of plunder that predicate crimes are mainly mala in se. Its abomination lies in the
significance and implications of the subject criminal acts in the scheme of the larger socio-political
and economic context in which the state finds itself to be struggling to develop and provide for
its poor and underprivileged masses. Reeling from decades of corrupt tyrannical rule that
bankrupted the government and impoverished the population, the Philippine Government must
muster the political will to dismantle the culture of corruption, dishonesty, green and syndicated
criminality that so deeply entrenched itself in the structures of society and the psyche of the
populace. [With the government] terribly lacking the money to provide even the most basic
services to its people, any form of misappropriation or misapplication of government funds
translates to an actual threat to the very existence of government, and in turn, the very survival
of people it governs over.

7
CRIMINAL LAW I – DIGESTED CASES BY: BOAQUINA, MARY CRISCHAEL
8

PEOPLE V. DACUYCUY
G.R. No. L-45127. May 5, 1989

FACTS: This is a complaint against private respondents, public school officials of Leyte, for
violation of Republic Act No. 4670 otherwise known as the Magna carta for Public School
Teachers. The respondents pleaded not guilty and petitioned for certeriori and prohibition with
preliminary injuction before the Court of First Instance of Leyte, Branch VII alleging that:
a. The Municipal Court of Hindang has no jurisdiction over the case due to the correctional nature
of the penalty of imprisonment (as state in Sec. 32 of R.A. No. 4670) prescribed for the offense
b. Section 32 of R.A. No. 4670 is unconstitutional because, (1) the term of imprisonment is
unfixed and may run to reclusion perpetua; and (2) it constitutes an undue delegation of
legislative power, the duration of the penalty of imprisonment being solely left to the discretion
of the court as if the latter were the legislative department of the Government.
On March 30, 1976, the petition was transferred to Branch IV where the respondent Judge, Judge
Dacuycuy ruled that R.A. No. 4670 is valid and constitutional but cases for its violation fall outside
of the jurisdiction of municipal and city courts.

ISSUE: Whether or not Repbulic Act No. 4670 is unconstitutional.

HELD: Yes. Republic Act No. 4760 is unconstitutional. Section 32 violates the constitutional
prohibition against undue delegation of legislative power by vesting in the court the responsibility
of imposing a duration on the punishment of imprisonment, as if the courts were the legislative
department of the government.

8
CRIMINAL LAW I – DIGESTED CASES BY: BOAQUINA, MARY CRISCHAEL
9

PESIGAN V. ANGELES
G.R. No. L-64279. April 30, 1984

FACTS: Two Pesigans, carabao dealers, transported 26 carabaos and a calf from Camarines Sur to
Batangas. They were provided with health certificate, permit to transport and certificate of
inspection. In spite of the permit and certificates, the carabaos were confiscated. The
confiscation was basis on the aforementioned Executive Order No. 626-A which provides "that
henceforth, no carabao, regardless of age, sex, physical condition or purpose and no carabeef
shall be transported from one province to another. The carabaos or carabeef transported in
violation of this Executive Order as amended shall be subject to confiscation and forfeiture by
the government to be distributed ... to deserving farmers through dispersal as the Director of
Animal Industry may see fit, in the case of carabaos". Doctor Miranda distributed the carabaos
among twenty-five farmers of Basud, and to a farmer from the Vinzons municipal nursery.

ISSUE: Whether or not the executive order shall be enforced against Pesigans.

HELD: We hold that the said executive order should not be enforced against the Pesigans on April
2, 1982 because, as already noted, it is a penal regulation published more than two months later
in the Official Gazette dated June 14, 1982. It became effective only fifteen days thereafter as
provided in article 2 of the Civil Code and section 11 of the Revised Administrative Code. The
word "laws" in article 2 (article 1 of the old Civil Code) includes circulars and regulations which
prescribe penalties. Publication is necessary to apprise the public of the contents of the
regulations and make the said penalties binding on the persons affected thereby.

In the instant case, the livestock inspector and the provincial veterinarian of Camarines Norte
and the head of the Public Affairs Office of the Ministry of Agriculture were unaware of Executive
Order No. 626-A. The Pesigans could not have been expected to be cognizant of such an executive
order. It results that they have a cause of action for the recovery of the carabaos. The summary
confiscation was not in order. The recipients of the carabaos should return them to the Pesigans.
However, they cannot transport the carabaos to Batangas because they are now bound by the
said executive order. Neither can they recover damages. Doctor Miranda and Zenarosa acted in
good faith in ordering the forfeiture and dispersal of the carabaos.

9
CRIMINAL LAW I – DIGESTED CASES BY: BOAQUINA, MARY CRISCHAEL
10

MINUCHER V. SCALZO
G.R. No. 142396. February 11, 2003

FACTS: This is a case filed against Minucher for the Violation of the “Dangerous Drugs Act of
1972,” following a “buy-bust operation” conducted by Philippine police narcotic agents
accompanied by Scalzo in the house of Minucher, an Iranian national, where heroin was said to
have been seized. Minucher was later acquitted by the court. Minucher later on filed for damages
due to trumped-up charges of drug trafficking made by Arthur Scalzo.

As his defense, Scalzo said that he had acted in the discharge of his official duties as being merely
an agent of the Drug Enforcement Administration of the United States Department of Justice. He
subsequently filed a motion to dismiss the complaint on the ground that, being a special agent
of the United States Drug Enforcement Administration, he was entitled to diplomatic immunity.
He attached to his motion Diplomatic Note of the United States Embassy addressed to DOJ of the
Philippines and a Certification of Vice Consul Donna Woodward, certifying that the note is a true
and faithful copy of its original. Trial court denied the motion to dismiss.

ISSUE: Whether or not Scalzo is indeed entitled to diplomatic immunity.

HELD: Yes. Scalzo is a foreign agent, operating within a territory, can be cloaked with immunity
from suit as long as it can be established that he is acting within the directives of the sending
state. The consent or imprimatur of the Philippine government to the activities of the United
States Drug Enforcement Agency, however, can be gleaned from the undisputed facts in the case.

The job description of Scalzo has tasked him to conduct surveillance on suspected drug suppliers
and, after having ascertained the target, to inform local law enforcers who would then be
expected to make the arrest. In conducting surveillance activities on Minucher, later acting as the
poseur-buyer during the buy-bust operation, and then becoming a principal witness in the
criminal case against Minucher, Scalzo hardly can be said to have acted beyond the scope of his
official function or duties.

10
CRIMINAL LAW I – DIGESTED CASES BY: BOAQUINA, MARY CRISCHAEL
11

LIANG V. PEOPLE
G.R. No. 125865. January 28, 2000

FACTS: Petitioner is charged for allegedly uttering defamatory words against fellow ADB worker.
Department of Foreign Affairs (DFA) stating that petitioner is covered by immunity from legal
process under Agreement between the ADB and the Philippine Government regarding the
Headquarters of the ADB (hereinafter Agreement) in the country. Based on the said protocol
communication that petitioner is immune from suit, the MeTC judge without notice to the
prosecution dismissed the two criminal cases. The latter led a motion for reconsideration which
was opposed by the DFA.

ISSUE: Whether or not the petitioner is covered by immunity

HELD: No. The DFA's determination that a certain person is covered by immunity is only
preliminary which has no binding effect in courts. In receiving the DFA's advice and in dismissing
the two criminal cases without notice to the prosecution, the latter's right to due process was
violated. The needed inquiry in what capacity petitioner was acting at the time of the alleged
utterances requires for its resolution evidentiary basis that has yet to be presented at the proper
time.
Mere invocation of the immunity clause does not result in the dropping of the charges. Under
Sec. 45 of the Agreement which states: "Ocers and staff of the Bank including for the purpose of
this Article experts and consultants performing missions for the Bank shall enjoy the following
privileges and immunities:
a.) immunity from legal process with respect to acts performed by them in their ocial capacity
except when the Bank waives the immunity."
The immunity mentioned therein is not absolute, but subject to the exception that the act was
done in "ocial capacity." It is therefore necessary to determine if the case falls within the ambit
of Section 45(a).

Under the Vienna Convention on Diplomatic Relations, a diplomatic agent, assuming petitioner
is such, enjoys immunity from criminal jurisdiction of the receiving state except in the case of an
action relating to any professional or commercial activity exercised by the diplomatic agent in
the receiving state outside his ocial functions. The commission of a crime is not part of official
duty.

11
CRIMINAL LAW I – DIGESTED CASES BY: BOAQUINA, MARY CRISCHAEL
12

PEOPLE V. NARVAEZ
G.R. Nos. L-33466-67. April 20, 1983

FACTS: Narvaez shot Fleischer and Rubia during the time the two were constructing a fence that
would prevent Narvaez from getting into his house and rice mill. The defendant was taking a nap
when he heard sounds of construction and found fence being made. He addressed the group and
asked them to stop destroying his house and asking if they could talk things over. Defendant lost
his "equilibrium," and shot Fleisher with his shotgun. He also shot Rubia who was running
towards the jeep where the deceased's gun was placed.
Prior to the shooting, Fleischer and Co. (the company of Fleischer's family) was involved in a legal
battle with the defendant and other land settlers of Cotabato over certain pieces of property. At
the time of the shooting, the civil case was still pending for annulment (settlers wanted granting
of property to Fleisher and Co. to be annulled). At time of the shooting, defendant had leased his
property from Fleisher (though case pending and ownership uncertain) to avoid trouble.
On June 25, defendant received letter terminating contract because he allegedly didn't pay rent.
He was given 6 months to remove his house from the land. Shooting was barely 2 months after
letter. Defendant claims he killed in defense of his person and property. CFI ruled that Narvaez
was guilty. Aggravating circumstances of evident premeditation offset by the mitigating
circumstance of voluntary surrender. For both murders, CFI sentenced him to reclusion perpetua,
to indemnify the heirs, and to pay for moral damages.

ISSUE: Whether or not CFI erred in convicting defendant-appellant despite the fact that he acted
in defense of his person.

HELD: No. The courts concurred that the fencing and chiselling of the walls of the house of the
defendant was indeed a form of aggression on the part of the victim. However, this aggression
was not done on the person of the victim but rather on his rights to property. On the first issue,
the courts did not err. However, in consideration of the violation of property rights, the courts
referred to Art. 30 of the civil code recognizing the right of owners to close and fence their land.
Although is not in dispute, the victim was not in the position to subscribe to the article because
his ownership of the land being awarded by the government was still pending, therefore putting
ownership into question. It is accepted that the victim was the original aggressor.

12
CRIMINAL LAW I – DIGESTED CASES BY: BOAQUINA, MARY CRISCHAEL
13

PEOPLE V. LACSON
G.R. No. 149453. May 28, 2002

FACTS: Lacson was charged with multiple murder for the killing of 11persons. Petitioners contend
that Sec. 8, Rule 117 of the RRCP is not applicable to the case and the time-bar in said rule should
not be applied retroactively. They contend that even on the assumption that the respondent
expressly consented to a provisional dismissal of the cases, the two-year bar in Section 8 of Rule
117 of the Revised Rules of Criminal Procedure should be applied prospectively and not
retroactively against the State. To apply the time limit retroactively to the criminal cases against
the respondent and his co-accused would violate the right of the people to due process, and
unduly impair, reduce, and diminish the State's substantive right to prosecute the accused for
multiple murder.

Lacson asserts that the new rule under Sec. 8 of Rule 117 of the RRCP may be applied retroactively
since there is no substantive right of the State that may be impaired by its application to the
criminal cases in question since the State's witnesses were ready, willing and able to provide their
testimony but the prosecution failed to act on these cases until it became politically expedient in
April 2001 for them to do so. According to the respondent, penal laws, either procedural or
substantive, may be retroactively applied so long as they favor the accused. The Court agrees
with the respondent that procedural laws may be applied retroactively. As applied to criminal
law, procedural law provides or regulates the steps by which one who has committed a crime is
to be punished.

ISSUE: Whether or not the new time-bar be applied retroactively.

HELD: No. The time-bar of two years under the new rule should not be applied retroactively
against the State. The Court agrees with the petitioners that to apply the time-bar retroactively
so that the two-year period commenced to run on March 31, 1999 when the public prosecutor
received his copy of the resolution of Judge Agnir, Jr. dismissing the criminal cases is inconsistent
with the intendment of the new rule. Instead of giving the State two years to revive provisionally
dismissed cases, the State had considerably less than two years to do so.
If the Court applied the new time-bar retroactively, the State would have only 1 year and 3
months or until March 31, 2001 within which to revive these criminal cases. The two-year period
fixed in the new rule is for the benefit of both the State and the accused. It should not be
emasculated and reduced by an inordinate retroactive application of the time-bar therein
provided merely to benefit the accused. For to do so would cause an "injustice of hardship" to
the State and adversely affect the administration of justice in general and of criminal laws in
particular.

13
CRIMINAL LAW I – DIGESTED CASES BY: BOAQUINA, MARY CRISCHAEL
14

GO V. DIMAGIBA
G.R. No. 151876. June 21, 2005

FACTS: Dimagiba issued to Susan Go 13 checks which were dishonored for the reason account
closed. He was prosecuted for 13 counts of violation of BP 22 under separate complaints. After
a joint trial, the MTCC convicted the accused in the 13 cases but RTC denied the appeal of
Dimagiba and sustained his conviction. He filed a Motion for Reconsideration praying for the
recall of the Order of Arrest, arguing that the penalty of fine only, instead of imprisonment also,
should have been imposed on him, citing Supreme Court Administrative Circular (SC-AC) No. 12-
2000, which allegedly required the imposition of a fine only instead of imprisonment also for BP
22 violations, if the accused was not a recidivist or a habitual delinquent. Motion for
Reconsideration was denied. He was arrested and imprisoned for the service of his sentence but
filed a Petition for a writ of habeas corpus. The RTC directed his immediate release.
The RTC invoked Vaca v. CA and Supreme Court Administrative Circular (SC-AC) No. 12-2000,
which allegedly required the imposition of a fine only instead of imprisonment also for BP 22
violations, if the accused was not a recidivist or a habitual delinquent. The RTC held that this rule
should be retroactively applied in favor of Dimagiba because he was a first-time offender and an
employer of at least 200 workers who would be displaced as a result of his imprisonment; and
the civil liability had already been satisfied.

ISSUE: Whether or not SC-AC No. 12-2000 must be applied retroactively on this case, eliminating
imprisonment from Dimagiba's penalty.

HELD: NO. SC Admin. Circular No. 12-2000 is not a penal law, hence, Article 22 of the Revised
Penal Code(retroactive effect of penal laws) is not applicable. The circular applies only to those
cases pending as of the date of its effectivity and not to cases already terminated by
final judgment.
SC Admin. Circular No.12-2000 merely lays down a rule of preference in the application of
the penalties for violation of B.P.Blg. 22. It does not amend B.P. Blg. 22, nor defeat the legislative
intent behind the law. The circular merely urges the courts to take into account not only the
purpose of the law but also the circumstances of the accused, whether he acted in good faith or
on a clear mistake of fact without taint of negligence, and such other circumstance which the
trial court or the appellate court believes relevant to the penalty to be imposed.

14
CRIMINAL LAW I – DIGESTED CASES BY: BOAQUINA, MARY CRISCHAEL
15

U.S. V. AH SING
G.R. No. L-13005. October 10, 1917

FACTS: Ah Sing bought opium in Saigon, brought them on board and had them in his possession
during the trip from Saigon to Cebu. The authorities found the 8 cans of opium hidden in the
ashes below the boiler of the steamer’s engine. Ah Sing confessed that the same was his but he
did not say anything as to the purpose why he bought them nor did he say that it was his intention
to import the prohibited drug. There is no other evidence, direct or indirect, to show that the
intention of the accused was to illegally import the opium in the country.

ISSUE: Whether the defendant is guilty of illegal importation of opium into the Philippine Islands.

HELD: Yes. Applying the Opium Law, the Supreme Court expressly held that any person unlawfully
imports or brings any prohibited drug into the Philippine Islands, when the prohibited drug is
found under this person's control on a vessel which has come direct from a foreign country and
is within the jurisdictional limits of the Philippine Islands. In such case, a person is guilty of illegal
importation of the drug unless contrary circumstances exist or the defense proves otherwise.
Applied to the facts herein, it would be absurd to think that the accused was merely carrying
opium back and forth between Saigon and Cebu for the mere pleasure of so doing. It would
likewise be impossible to conceive that the accused needed so large an amount of opium for his
personal use. No better explanation being possible, the logical deduction is that the defendant
intended this opium to be brought into the Philippine Islands. We accordingly find that there was
illegal importation of opium from a foreign country into the Philippine Islands. To anticipate any
possible misunderstanding, let it be said that these statements do not relate to foreign vessels in
transit, a situation not present. Ah Sing was proven guilty beyond reasonable doubt of illegal
importation.

15
CRIMINAL LAW I – DIGESTED CASES BY: BOAQUINA, MARY CRISCHAEL
16

MIQUIABAS V. COMMANDNG GENERAL


G.R. No. L-1988. February 24, 1948

FACTS: Petitioner is a Filipino citizen and a civilian employee of the US Army in the Philippines.
He has been charged with disposing in the Port of Manila Area of things belonging to the United
States Army, in violation of the 94th Article of War of the United States. He has been arrested
for that reason and a General Court-Martial appointed by respondent tried and found him guilty
and sentenced him to 15 years imprisonment. This sentence, however, is not yet final for it is still
subject to review. Thus, a petition for a writ of habeas corpus was filed by petitioner against the
Commanding General Philippine-Ryukyus Command, United States Army, who is alleged to have
petitioner under custody and to have appointed a General Court-Martial to try petitioner in
connection with an offense over which the said court has no jurisdiction.

ISSUE: Whether or not the General Court-Martial appointed by respondent has jurisdiction to try
petitioner for the offense allegedly committed by the petitioner.

HELD: No. It is contended that petitioner must be released immediately by respondent without
prejudice to any criminal action which may be instituted in the proper court of the Philippines.
The offense at bar cannot be considered as committed within a base that would bring the case
within the jurisdiction of the general court-martial. It may be stated as a rule that the Philippines,
being a sovereign nation, has jurisdiction over all offenses committed within its territory. But it
may, by treaty or by agreement, consent that the United States or any other foreign nation, shall
exercise jurisdiction over certain offenses committed within certain portions of said territory.
The offense at bar cannot be considered as committed within, but without, a base, since it has
been committed in the Port of Manila Area, which is not one of the bases mentioned in the
Agreement, and is merely temporary quarters located within the present limits of the City of
Manila.

16
CRIMINAL LAW I – DIGESTED CASES BY: BOAQUINA, MARY CRISCHAEL
17

U.S. V. BULL
GR. No. L-5270. January 15, 1910

FACTS: A steamship vessel engaged in the transport of animals commanded by H.N. Bull docked
in the port of Manila, Philippines. It was found that said vessel from Ampieng, Formosa carried
677 heads of cattle without providing appropriate shelter and proper suitable means for securing
the animals which resulted for most of the animals to get hurt and others to have died while in
transit. This cruelty to animals is said to be contrary to Acts No. 55 and No. 275 of the Philippine
Constitution. It is however contended that cases cannot be filed because neither was it said that
the court sitting where the animals were disembarked would take jurisdiction, nor did it say
about ships not licensed under Philippine laws, like the ship involved.

ISSUE: Whether or not the court had jurisdiction over an offense committed on board a foreign
ship while inside the territorial waters of the Philippines.

HELD: No court of the Philippines has jurisdiction over any crimes committed in a foreign ship on
the high seas, but the moment it entered into territorial waters, it automatically would be subject
to the jurisdiction of the country. The offense, assuming that it originated in Formosa, which the
Philippines would have no jurisdiction, continued until it reached Philippine territory which is
already under jurisdiction of the Philippines.
Every state has complete control and jurisdiction over its territorial waters. The Supreme Court
of the United States has recently said that merchant vessels of one country visiting the ports of
another for the purpose of trade would subject themselves to the laws which govern the ports
they visit, so long as they remain.

17
CRIMINAL LAW I – DIGESTED CASES BY: BOAQUINA, MARY CRISCHAEL
18

PEOPLE V. WONG CHENG


G.R. No. L-18924 October 19, 1922

FACTS: The Attorney-General urges the revocation of the order of the Court of First Instance of
Manila, sustaining the demurer presented by Wong Cheng who was accused of having illegally
smoked opium, aboard the merchant vessel Changsa of English nationality while said vessel was
anchored in Manila Bay two and a half miles from the shores of the city. The demurer alleged
lack of jurisdiction on the part of the lower court, which so held and dismissed the case.

ISSUE: Whether or not the courts of the Philippines have jurisdiction over crimes committed
aboard a foreign vessel anchored in our jurisdiction waters.

HELD: Two fundamental rules on this particular matter in connection with International Law. The
French rule, according to which crimes committed aboard a foreign merchant vessels should not
be prosecuted in the courts of the country within whose territorial jurisdiction they were
committed, unless their commission affects the peace and security of the territory; and English
rule which is based on the territorial principle and followed in the United States. In this rule,
crimes perpetrated under such circumstances are in general triable in the courts of the country
within territory they were committed.
As to whether the United States has ever consented by treaty or otherwise to renouncing such
jurisdiction or a part thereof, we find nothing to this effect so far as England is concerned, to
which nation the ship where the crime in question was committed belongs. Mere possession of
opium aboard a foreign vessel in transit was held by this court not triable by or courts, because
it being the primary object of our Opium Law to protect the inhabitants of the Philippines against
the disastrous effects entailed by the use of this drug, its mere possession in such a ship, without
being used in our territory, does not being about in the said territory those effects that our
statute contemplates avoiding. Hence such a mere possession is not considered a disturbance of
the public order.
To smoke opium within our territorial limits, even though aboard a foreign merchant ship, is
certainly a breach of the public order here established, because it causes such drug to produce
its pernicious effects within our territory. It seriously contravenes the purpose that our
Legislature has in mind in enacting the aforesaid repressive statute.

18
CRIMINAL LAW I – DIGESTED CASES BY: BOAQUINA, MARY CRISCHAEL
19

U.S. V. LOOK CHOW


G.R. No. L-5887. December 16, 1910

FACTS: Complaint were filed against Look Chaw for unlawful possession and sale of opium.
Steapship Erroll of English nationality that came from Hongkong, and was bound for
Mexico, arrived at the ports of Manila and Cebu. There were sacks of opium fund during the
inspection. The firemen and crew of foreign vessels, pursuant to the instructions he had from the
Manila custom-house, were permitted to retain certain amounts of opium, always provided it
should not be taken shore so it was returned. Look Chaw admitted that he had bought these
sacks of opium, in Hongkong with the intention of selling them as contraband in Mexico or Vera
Cruz, and that, as his hold had already been searched several times for opium, he ordered two
other Chinamen to keep the sack.

ISSUE: Whether or not the Philippine court has jurisdiction.

HELD: Yes. It is the general rule that the mere possession of a thing of prohibited use in these
Islands, aboard a foreign vessel in transit, in any of their ports, does not constitute a crime triable
by the courts of this country, on account of such vessel being considered as an extension of its
own nationality. But there is an exception to the said rule—when the article, whose use is
prohibited within the Philippine Islands, in the present case a can of opium, is landed from the
vessel upon Philippine soil, thus committing an open violation of the laws of the land with respect
to which, as it is a violation of the penal law in force at the place of the commission of the crime,
only the court established in that said place itself had competent jurisdiction, in the absence of
an agreement under an international treaty.

19
CRIMINAL LAW I – DIGESTED CASES BY: BOAQUINA, MARY CRISCHAEL
20

PEOPLE V. LOL-LO and SARAW


G.R. No. 17958. February 27, 1922

FACTS: The six vintas were manned by 24 armed Moros. The said Dutch boats were carrying men,
women and children. At first, the Moros asked for food. But when they got on the Dutch boats,
they asked for themselves all the vessel’s cargo, attacked nearly all of the men and brutally
violated two of the women by methods too tremendous to be described. All of the persons on
the Dutch boat, except the two young women, were again placed on it and holes were made in
it, the idea that it would submerge. The Moros finally arrived at Maruro, a Dutch possession. Two
of the Moro marauders were Lol-lo, who also raped one of the women, and Saraw. At Maruro,
the two women were able to escape.

ISSUE: Whether or not the provisions of the Penal Code dealing with the crime of piracy are still
in force.

HELD: Yes. It is a crime not against any particular state but against all mankind. It may be
punished in the competent tribunal of any country where the offender may be found or into
which he may be carried. The jurisdiction of piracy unlike all other crimes has no territorial
limits. As it is against all so may it be punished by all.
Penal code dealing with the crime of piracy, notably articles 153 and 154, to be still in force in
the Philippines

20
CRIMINAL LAW I – DIGESTED CASES BY: BOAQUINA, MARY CRISCHAEL
21

ARTICLE 3: Definition of Felony

U.S. V. AH CHONG
G.R. No. 5272. March 19, 1910

FACTS: Ah Chong was employed as a cook in Mckinley, & Pascual, the deceased, was employed
as a house boy and was his roommate. At 10 pm, Ah Chong was suddenly awakened by someone
trying to open the door. He called out twice, "Who is there?" Fearing that the intruder was a
robber, he called out again, "If you enter the room, I will kill you." At the moment he was struck
by the chair which was placed against the door. Believing he was attacked, he took the kitchen
knife he kept under his pillow, and struck the intruder who turned out to be his roommate,
Pascual. He then and there admitted that he had stabbed his roommate, believing he was a
robber because he forced open the door of their sleeping room, despite warnings.

ISSUE: Whether or not Ah Chong can invoke mistake of fact as a defense

HELD: Yes. There is no criminal liability, provided always that the alleged ignorance or mistake of
fact was not due to negligence or bad faith. “actus non facit reum nisi mens sit rea”- the act itself
does not make a man guilty unless his intention were so.
To justify the act there must be:
1. Unlawful aggression on the part of the person killed
2. Reasonable necessity of the means employed to prevent or repel it
3. Lack of sufficient provocation on the part of the person defending himself
There is an innocent mistake of fact without any fault or carelessness on the part of the accused,
because having no time or opportunity to make further inquiry, and being pressed by
circumstances to act immediately, the accused had no alternative but to take the facts as they
then appeared to him.

21
CRIMINAL LAW I – DIGESTED CASES BY: BOAQUINA, MARY CRISCHAEL
22

PEOPLE V. OANIS
G.R. No. 47722. July 27, 1943

FACTS: Oanis and Galanta were instructed to arrest Balagtas, an escaped convict, and if
overpowered, to get him dead or alive. Proceeding to the suspected house, they went into the
room of Irene, and on seeing a man sleeping with his back towards the door where they were,
simultaneously or successively fired at him.
Awakened by the gunshots, Irene saw her paramour already wounded. it turned out later that
the person shot and killed was not the notorious criminal Balagtas but a peaceful and innocent
citizen named Serapio Tecson, Irene's paramour.Their testimonies are certainly not credible not
only because they were to exculpate themselves of the crime, but also because they are
materially contradictory.

ISSUE: Whether or not mistake of fact is a defense

HELD: No. An officer is never justified in using unnecessary force or in treating him with violence,
or in resorting to dangerous means when the arrest could be effected otherwise. Balagtas was a
notorious criminal, a fugitive, but these facts alone constitute no justification for killing him when,
in effecting his arrest, he offers no resistance, or in fact no resistance can be offered, as when he
is asleep. The crime committed is not merely criminal negligence, the killing being intentional and
not accidental. As the deceased was killed while asleep, the crime committed is murder with the
qualifying circumstance of alevosia, with mitigating circumstance: Offenders acted in the
performance of a duty or in the lawful exercise of right.

22
CRIMINAL LAW I – DIGESTED CASES BY: BOAQUINA, MARY CRISCHAEL
23

PEOPLE V. PUGAY
G.R. No. 74324. November 17, 1988

FACTS: At a town fiesta fair, Gabion and Henry were sitting in the ferris wheel when Pugay and
Samson with several companions arrived, who appeared to be drunk. As they saw Miranda
walking nearby, they started making fun of him. Not contented, Pugay took a can of gasoline and
poured it on the body of Miranda and Samson set him on fire. Pugay argued that he thought it
was water, while Samson denied setting Miranda on fire, claiming that someone else did it.

ISSUE: Whether or not there was conspiracy in the commission of the crime.

HELD: No. There is nothing in the records showing that there was previous conspiracy or unity of
criminal purpose and intention between the two accused-appellants immediately before the
commission of the crime. Their meeting at the scene of the incident was accidental. It is also clear
that the accused Pugay and his group merely wanted to make fun of the deceased. Hence, the
respective criminal responsibility of Pugay and Samson arising from different acts directed
against the deceased is individual and not collective, and each of them is liable only for the act
committed by him. There can be no doubt that the accused Samson knew very well that the liquid
poured on the body of the deceased was gasoline and a flammable substance for he would not
have committed the act of setting the latter on fire if it were otherwise. Criminal liability shall be
incurred by any person committing a felony although the wrongful act done be different from
that which he intended.
Pugay is guilty of homicide through reckless imprudence. Samson is only guilty of the crime of
homicide.

23
CRIMINAL LAW I – DIGESTED CASES BY: BOAQUINA, MARY CRISCHAEL
24

PEOPLE V. GARCIA
G.R. No. 153591. February 23, 2004

FACTS: Billon and his younger sister, Sanily, boarded a jeep on their way to school. Bentley
crossed the street and waited on the center island for Sanily to cross. While Sanily was crossing
the street, a jeepney driven by Garcia hit her on the left side of the body. Sanily fell and was
thrown to the ground a meter away from the vehicle. The jeepney stopped. But as Bentley was
running towards his sister, the vehicle suddenly accelerated with its front tire running over
Sanily’s stomach. She was brought to a hospital but was transferred to another due to lack of
medical facilities. Sanily died.

ISSUE: Whether or not he is guilty of Murder or Reckless imprudence resulting in homicide

HELD: Reckless Imprudence. There’s no malicious intent. Trial court erred in convicting him of
murder qualified by evident premeditation.
The elements of Evident Premeditation. are: (1) a previous decision to commit the crime; (2) an
overt act indicating that he clung to his determination; (3) a lapse of time between the decision
to commit the crime and its actual execution sufficient to allow him to reflect upon the
consequences.
Garcia had no intention to kill. He cannot be held liable for an intentional felony.
In intentional crimes, the act itself is punished; in negligence or imprudence, what is principally
penalized is the mental attitude or condition behind the act, the dangerous recklessness, lack of
care or foresight. Reckless imprudence consists in voluntarily, but w/o malice, doing or failing to
do an act from which material damage results by reason of inexcusable lack of precaution on the
part of the person performing such act.
Garcia is guilty beyond reasonable doubt of the crime reckless imprudence resulting in homicide

24
CRIMINAL LAW I – DIGESTED CASES BY: BOAQUINA, MARY CRISCHAEL
25

GARCIA V. CA
G.R. No. 157171. March 14, 2006

FACTS: During the canvassing period for the May elections in Pangasinan, Election officers Garcia
et. al., decreased the votes received by senatorial candidate Pimentel, from 6,998 votes, to 1,921
votes. Garcia was found guilty beyond reasonable doubt, of the crime defined under RA No. 6646,
for decreasing the votes of Senator Pimentel in the total of 5,034 and in relation to BP Blg. 881,
considering that this finding is a violation of Election Offense. Petitioner contends the CA's
judgment is erroneous, based on speculations, surmises and conjectures, instead of substantial
evidence; and there was no motive on her part to reduce the votes of private complainant.
Respondent contends that good faith is not a defense in the violation of an election law, which
falls under the class of mala prohibita.

ISSUE: Whether or not a violation of Section 27(b) of Rep. Act No. 6646, classified under Mala in
se or mala prohibita

HELD: Act prohibited under RA are mala in se. Intentionally increasing or decreasing the number
of votes received by a candidate is inherently immoral, since it is done with malice and intent to
injure another.

25
CRIMINAL LAW I – DIGESTED CASES BY: BOAQUINA, MARY CRISCHAEL
26

MANUEL V. PEOPLE
G.R. No. 165842. November 29, 2005

FACTS: Eduardo, 39, married, met the complainant Tina a 21 year old, a Computer Secretarial
student. Afterwards, Eduardo went to Baguio City to visit Tina. Fast forward, Eduardo proposed
marriage on several occasions, assuring her that he was single. Few years after they got married,
Eduardo left, and did not return. Tina became curious and made inquiries from the NSO where
she learned that Eduardo had been previously married.
Eduardo testified that he informed Tina of his previous marriage but she nevertheless agreed to
marry him and that he declared that he was "single" in his marriage contract with Tina because
he believed in good faith that his first marriage was invalid.
He insisted that he married Tina believing that his first marriage was no longer valid because he
had not heard from his wife Rubylus for more than 20 years.
He did not know that he had to go to court to seek for the nullification of his first marriage before
contracting subsequent marriage.

ISSUE: Whether or not he is guilty of Bigamy

HELD: Yes, he contracted a second marriage before the former has been legally dissolved, or
before the absent spouse has been declared presumptively dead. The felony is consummated on
the celebration of the second marriage or subsequent marriage. When the act or omission
defined by law as a felony is proved to have been done or committed by the accused, the law
presumes it to have been intentional. For one to be criminally liable for a felony by dolo, there
must be a confluence of both an evil act and an evil intent. General rule is that a mistake of fact
or good faith of the accused is a valid defense in a prosecution for a felony by dolo. Such defense
negates malice or criminal intent.

26
CRIMINAL LAW I – DIGESTED CASES BY: BOAQUINA, MARY CRISCHAEL
27

PEOPLE V. DELIM
G.R. No. 142773. January 28, 2003

FACTS: Modesto, and his family were about to eat their dinner when Marlon, Robert and Ronald,
armed with a short handgun, barged into the house. Marlon poked his gun at Modesto while
Robert and Ronald grabbed and hog-tied the him. He was also gagged with a cloth and was
herded out of the house.
Leon and Manuel, also armed, stayed by the door & ordered the wife &son to stay. Few days
after, Randy, with his relatives, found Modesto dead under thick bushes in a grassy area. Autopsy
showed the cause of death was a gunshot wound at the head and the stab wounds found were
defensive wounds.

ISSUE: Whether or not conspiracy and treachery are present

HELD: Yes. There’s conspiracy but no treachery. It is sufficient if, at the time of commission of
the crime, the accused had the same purpose and were united in its executed. Appellants acted
in unison when they abducted Modesto. So their acts were synchronized and executed with
precision evincing a preconceived plan to kill Modesto
For Treachery to be appreciated prosecution needs to prove:
a. employment of means of execution gives the person no opportunity to defend himself
b. the means of execution is deliberately and consciously adopted
In the appellant’s case, there are no evidence to the particulars on how Modesto was
assaulted and killed and this in fact does mean that treachery cannot be proven since it
cannot be presumed that Modesto was defenseless during the time that he was being
attacked and shot at by the appellants.

27
CRIMINAL LAW I – DIGESTED CASES BY: BOAQUINA, MARY CRISCHAEL
28

IVLER V. SAN PEDRO


G.R. No. 172716. November 17, 2010

FACTS: Following a vehicular collision, Ivler was charged with two separate offenses: (1) reckless
imprudence resulting in slight physical injuries for injuries sustained by respondent Evangeline L.
Ponce; and (2) reckless imprudence resulting in homicide and damage to property for the death
of respondent Ponce’s husband Nestor C. Ponce and damage to the spouses’ vehicle
Ivler pleaded guilty to the charge in reckless imprudence resulting in slight physical injuries but
moved to quash the Information of reckless imprudence resulting in homicide and damage to
property for placing him in jeopardy of second punishment for the same offense of reckless
imprudence.

ISSUE: Whether or not Ivler’s constitutional right under the Double Jeopardy Clause bars further
proceedings in the information charging him with reckless imprudence resulting in homicide and
damage to property.

HELD: Yes.
1. Reckless Imprudence is a Single Crime; its Consequences on Persons and Property are
Material Only to Determine the Penalty
2. Prior Conviction or Acquittal of Reckless Imprudence Bars Subsequent Prosecution for the
Same Quasi-offense
As the careless act is single, whether the injurious result should affect one person or several
persons, the offense (criminal negligence) remains one and the same, and cannot be split
into different crimes and prosecutions.
3. Article 48 Does Not Apply to Acts Penalized Under Article 365 of the Revised Penal Code
Prosecutions under Article 365 should proceed from a single charge regardless of the
number or severity of the consequences. In short, there shall be no splitting of charges under
Article 365, and only one information shall be filed in the same first level court.

28
CRIMINAL LAW I – DIGESTED CASES BY: BOAQUINA, MARY CRISCHAEL
29

CALIMUTAN V. PEOPLE
G.R. No. 152133. February 9, 2006

FACTS: Cantre and Sanano were on their way home when they crossed paths with Bulalacao and
Calimutan. Cantre was harboring a grudge against Bulalacao, suspecting the latter as the culprit
responsible for throwing stones at the his house on a previous night. Thus, upon seeing him,
Cantre suddenly punched him. While Bulalacao ran away, petitioner Calimutan dashed towards
the backs of victim Cantre and Sañano. threw a stone at victim Cantre, hitting him at the left side
of his back. Sañano attempted to pacify the two, even convincing both to just go home.
The stoning incident resulted in laceration of spleen due to impact which caused his death a day
after.

ISSUE: Whether or not Rollie Calimutan is guilty beyond reasonable doubt of the crime of
homicide

HELD: Yes, Rollie Calimutan is guilty of reckless imprudence resulting in homicide, Since it is
irrefragable that the stone thrown by petitioner Calimutan at the victim Cantre was the
proximate cause of the latter’s death, despite being done with reckless imprudence rather than
with malicious intent, petitioner Calimutan remains civilly liable for such death.
Calimutan’s intention to drive away the attacker who was, at that point, the victim Cantre, and
to protect his helper Bulalacao.

29
CRIMINAL LAW I – DIGESTED CASES BY: BOAQUINA, MARY CRISCHAEL
30

DIEGO V. CASTILLO
A.M. No. RTJ-02-1673. August 11, 2004

FACTS: Escoto contracted marriage with Jorge de Perio. Jorge, few years after, filed a Divorce in
Texas. Subsequently, Escoto using the name Lucena, contracted marriage with Diego whose sister
filed a criminal case of bigamy against Escoto. Escoto was aquitted since state failed to prove guilt
beyond whisper of a doubt and gave credence to the defense of the accused that she acted
without any malicious intent for believing in good faith that her marriage was already annulled
by a foreign judgment. An administrative case is filed against Judge Castillo for knowingly
rendering an unjust judgment (felony)

ISSUE: Whether or not Castillo should be liable against Article 204[7] of the Revised Penal Code

HELD: No. Judge Castillo is FINED with a STERN WARNING that a repetition of the same or similar
acts will be dealt with more severely. Even assuming that a judge erred in acquitting an accused,
he still cannot be administratively charged lacking the element of bad faith, malice or corrupt
purpose. As a matter of public policy then, the acts of a judge in his official capacity are not
subject to disciplinary action, even though such acts are erroneous. Good faith and absence of
malice, corrupt motives or improper considerations are sufficient defenses in which a judge
charged with ignorance of the law can find refuge. Error committed by respondent Judge being
gross and patent, the same constitutes ignorance of the law of a nature sufficient to warrant
disciplinary action.

30
CRIMINAL LAW I – DIGESTED CASES BY: BOAQUINA, MARY CRISCHAEL
31

PEOPLE V. BAYONA
G.R. L-42288. February 16, 1935

FACTS: Bayona stopped his automobile in front of municipal building of Pilar for the purpose of
delivering to Mayor Agdamag a revolver that the defendant had taken that day from one Thomas
de Martin, who had no license therefor. According to him, he did not know there was a polling
place near where he parked his motor. He took with him the revolver because there were many
people in the road and he was afraid he might lose it. He was sixty-three meters from electoral
college when the revolver was taken from him by Jose E. Desiderio, a representative of the
Secretary of Interior.

ISSUE: Whether the defendant is liable even without criminal intent.

HELD: Yes. the law that Bayona violated is a statutory provision, and the intent with which he
violated it is immaterial. It may be conceded that the defendant did not intend to intimidate any
elector or to violate the law in any other way, but when he got out of his automobile and carried
his revolver inside of the fence surrounding the polling place, he committed the act complained
of, and he committed it willfully. The act prohibited by the Election Law was complete. The
intention to intimidate the voters or to interfere otherwise with the election is not made an
essential element of the offense. Unless such an offender actually makes use of his revolver, it
would be extremely difficult, if not impossible, to prove that he intended to intimidate the voters.

31
CRIMINAL LAW I – DIGESTED CASES BY: BOAQUINA, MARY CRISCHAEL
32

U.S. V. CHICO
G.R. No. 4963. September 15, 1909

FACTS: Chico displayed in one of the windows and one of the show cases of his store a number
of medallions, in the form of a small button, upon which were printed the miniature faces of
Emilio Aguinaldo and the flag or banner or device used during the late insurrection in the Phil.
Islands to designate and identify those in armed insurrection against the United States. The day
before, Go Chico had purchased the said medallion sold at a public sale under the authority of
the sheriff of the city of Manila.
Go Chico was arranging his stock of goods for the purpose of displaying them to the public, and
in doing so, he placed the medallions in his showcase and on one of the windows of his store. He
was ignorant of any law against the display of such medallions and had consequently no corrupt
intention. He argued that criminal intent on his part must be proved beyond a reasonable doubt
and that the prohibition of law is directed against the use of identical banners, devices or
emblems actually used during the Philippine insurrection by those in armed rebellion against the
United States.

ISSUE: Whether or not criminal intent is necessary in crimes punishable by special laws.

HELD: No. The court ruled that the act alone, irrespective of its motive, constitutes the crime.
The words “used during the late insurrection in the Philippine Islands to designate or identify
those in armed rebellion against the United States” mean not only the identical flags actually
used in the insurrection, but any flag which is of that type. The description refers not to a
particular flag, but to a type of flag. The literal interpretation of a statute may lead to an
absurdity, or evidently fail to give the real intent of the legislature.

32
CRIMINAL LAW I – DIGESTED CASES BY: BOAQUINA, MARY CRISCHAEL
33

ESTRADA V. SANDIGANBAYAN
G.R. No. 148560. November 19, 2001

FACTS: Joseph Ejercito Estrada (Estrada), the highest-ranking official to be prosecuted with the
crime of plunder. Estrada wishes to impress the Court that the assailed law is so defectively
fashioned that it crosses that thin but distinct line which divides the valid from the
constitutionality infirm. That there was a clear violations of the fundamental rights of the accused
to due process and to be informed of the nature and cause of the accusation.

ISSUE: Whether or not Plunder as defined in RA 7080 is a malum prohibitum.

HELD: No. It is Mala in Se. The legislative declaration in RA No. 7659 that plunder is a heinous
offense implies that it is a malum in se. For when the acts punished are inherently immoral or
inherently wrong, they are mala in se and it does not matter that such acts are punished in a
special law, especially since in the case of plunder that predicate crimes are mainly mala in se.
Its abomination lies in the significance and implications of the subject criminal acts in the scheme
of the larger socio-political and economic context in which the state finds itself to be struggling
to develop and provide for its poor and underprivileged masses. Reeling from decades of corrupt
tyrannical rule that bankrupted the government and impoverished the population, the Philippine
Government must muster the political will to dismantle the culture of corruption, dishonesty,
green and syndicated criminality that so deeply entrenched itself in the structures of society and
the psyche of the populace. [With the government] terribly lacking the money to provide even
the most basic services to its people, any form of misappropriation or misapplication of
government funds translates to an actual threat to the very existence of government, and in turn,
the very survival of people it governs over.

33
CRIMINAL LAW I – DIGESTED CASES BY: BOAQUINA, MARY CRISCHAEL
34

ARTICLE 4: Criminal Liability

PEOPLE V. TOMOTOGO
G.R. No. L-47941. April 30, 1985

FACTS: Tomotorgo left his home to work on his farm. Upon his return, he found his wife and his
three-month old baby already gone. He look for both of them and sometime later on, he finally
saw his wife carrying his infant son and bringing a bundle of clothes. He asked and pleaded with
his wife that she should return home with their child but she adamantly refused to do so. When
he sought to take the child from his wife, the latter threw the baby on the grassy portion of the
trail hereby causing the latter to cry.

Incensed with wrath and his anger beyond control, he picked lip a piece of wood nearby and
started hitting his wife with it until she fell to the ground complaining of severe pains on her
chest. Realizing what he had done, the accused picked his wife in his arms and brought her to
their home. He then returned to the place where the child was thrown and he likewise took this
infant home. The wife died despite the efforts of her husband to alleviate her pains. After the
accused changed the dress of his wife, he reported the tragic incident to the Barangay Captain of
their place who brought him to Policeman Arellosa to whom the accused surrendered. He also
brought with him the piece of wood he used in beating his wife.

ISSUE: Whether or not the accused should only be punished for the offense he intended to
commit which is serious physical injuries.

HELD: No. The fact that he intended to maltreat the victim only or inflict physical injuries does
not exempt him from liability for the resulting and more serious crime committed. His conviction
for the crime of Parricide is therefore correct in the light of the relevant provisions of law and
jurisprudence. Considering the circumstances which attended the commission of the offense, the
manifest repentant attitude of the accused and his remorse for his act which even the trial court
made particular mention of in its decision and the recommendation made by the Office of the
Solicitor General as well as number of years that the accused-appellant had been imprisoned,
this Court can do no less than recommend that executive clemency be extended to him.

34
CRIMINAL LAW I – DIGESTED CASES BY: BOAQUINA, MARY CRISCHAEL
35

PEOPLE V. AMIT
G.R. No. L-29066. Mach 25, 1970

FACTS: Amit, at the time of the commission of the crime, was 32 years of age, while his victim
was 25 years older than him. His victim resisted his attempt to rape. In order to subdue her,
appellant boxed her, held her on the neck and pressed it down while she was lying on her back
and he was on top of her.

Amit was charged in the trial court with the complex crime of rape with homicide described and
penalized in Article 335 of the Revised Penal Code, as amended. Arraigned with the assistance of
a counsel de officio, he pleaded guilty. He claims that the penalty of death imposed upon him
should be reduced to reclusion perpetua in view of the presence of three mitigating
circumstances namely: (1) plea of guilty; (2) voluntary surrender, and (3) lack of intention to
commit so grave a wrong as the one actually committed.
The Solicitor General admits that the mitigating circumstances of plea of guilty and voluntary
surrender have been proven, but denies that the mitigating circumstance of lack of intention to
commit so grave a wrong as the one actually committed was similarly established.

ISSUE: Whether or not there was a lack of intention to commit so grave a wrong as the one
actually committed.

HELD: No. The court ruled that his acts mentioned above were reasonably sufficient to produce
the result that they actually produced — the death of appellant's victim. Consequently, the lack
of intention to commit so grave a wrong as that committed cannot be appreciated in favor of an
accused who employed brute force, intention being gathered from and determined only by the
conduct and external acts of the offender, and the results of the acts themselves.

35
CRIMINAL LAW I – DIGESTED CASES BY: BOAQUINA, MARY CRISCHAEL
36

PEOPLE V. ULEP
G.R. No. L-36858. June 20, 1988

FACTS: Asuncion Pablo Ulep died as a result of physical injuries inflicted upon her by her husband,
accused Macario Ulep. The Chief of Police and the Rural Health Officer went to the house of the
deceased and there they saw the body on a bamboo bed surrounded by relatives, friends, and
the husband of the deceased, Macario. The Chief of Police suggested that an autopsy be
conducted but the husband refused to allow the same. However, the daughter of the deceased
by a previous marriage asked for a day or two to decide on her preference. At the behest of the
daughter, the request for an autopsy was made shortly before the burial. Accordingly, the police
chief and Dr. Eliseo Bonoan, a physician, caught up with the funeral Procession at the Catholic
cemetery and thereupon conducted an autopsy on the deceased.Two weeks after the burial, two
constabulary sergeants investigated Macario Ulep. He admitted that he caused the death of his
wife by elbowing her because his wife was then drunk and was uttering indecent words. Ulep
narrated that this elbowing and attack took place at their home at 5:3pm. She vomited and then
went to bed, The accused then left for the fields and returned at around 9:00 in the evening and
found his wife dead on her bed. He reported this death to their barrio captain.

Despite these statements, Ulep retracted his statement in court by narrating that more than a
year before that, and while his wife went to have their palay milled, their bullcart loaded with
sacks of rice turned upside down and pinned his wife on her breast. With the pain in her chest,
she was treated by a country quack doctor or "arbularyo."
From all the observations, findings, and an incisive study of the necropsy report, the cause of
death of the victim in this case is cardiac arrest and primary shock caused by the strong pressure
applied on the upper front chest bone. The man-size blows coming from the elbow of the
aggressor upon a thin-framed woman can only bring about fatal results.

ISSUE: Whether or not Ulep’s acts were the proximate cause of the death of his wife and is liable
for the death.

HELD: Yes. Even if the victim is suffering from an internal ailment, liver or heart disease, or
tuberculosis, if the blow delivered by the accused — (a) is the efficient cause of death; or (b)
accelerated his death; or (c) is the proximate cause of death; then there is criminal liability.
"He who is the cause of the cause is the cause of the evil caused," is the rationale in Article 4 of
the Revised Penal Code which provides that "criminal liability shall be incurred by a person
committing a felony (delito) although the wrongful act done be different from that which he
intended." Thus, even though a blow with the fist or a kick does not cause any external wound,
it may easily produce inflammation of the spleen and peritonitis and cause death, and even
though the victim may have been previously affected by some internal malady, yet if the blow
with the fist or foot accelerated death, he who caused such acceleration is responsible for the
death as the result of an injury willfully and unlawfully inflicted.
There is that clear and categorical showing that on the appellant fell the blame for these in human
acts on his wife. He should answer for her tragic death.

36
CRIMINAL LAW I – DIGESTED CASES BY: BOAQUINA, MARY CRISCHAEL
37

PEOPLE V. ILIGAN
G.R. No. 75369. November 26, 1990

FACTS: Fernando hacked Quinones, Jr. on his face, causing fatal injuries on the latter’s face who
then subsequently run over by a vehicle and died. The accused denied having perpetrated the
crime and stated that they were on their respective houses when the crime occurred. The lower
court found that Iligan’s group conspired to kill anyone or all members of the group of the victim
to vindicate the boxing on the face of Edmundo. Fernando and Edmundo were then charged with
the crime of murder with aggravating circumstances of evident premeditation and treachery.

ISSUE: Whether or not Fernando could be absolved of his criminal liability given that the victim
was subsequently run over by a vehicle

HELD: No. The essential requisites of Article 4 are:(a) that an intentional felony has been
committed, and (b) that the wrong done to the aggrieved party be the direct, natural and logical
consequence of the felony committed by the offender. These requisites are present in this case.
The intentional felony was the hacking by Fernando.
The second requisite was also met. Given that the incident happened on a national highway
where vehicles are expected to pass, Fernando’s hacking of Quinones’s head was the proximate,
might not be direct, cause of the latter’s death. The sequence of events from Fernando’s assault
to the time Quinones was run over by a vehicle is one unbroken chain of events. With that said,
it did not really matter if he directly caused Quinones’ death or if he actually meant it. Having
triggered such events, Iligan cannot escape liability even though the autopsy indicated that the
death was caused by a vehicular accident.

Since treachery and evident premeditation were not established, the lower court’s charge for
Fernando was modified from murder to homicide. However, it was not clearly established that
Edmundo took any direct part in the hacking incident since mere knowledge/approval of the act
without cooperation is not enough to charge him a co-principal. Therefore, he deserved
exoneration.

37
CRIMINAL LAW I – DIGESTED CASES BY: BOAQUINA, MARY CRISCHAEL
38

PEOPLE V. MANANQUIL
G.R. No. L-35574. September 28, 1984

FACTS: Mananquil went to the NAWASA Building at Pasay where her husband was then working
as a security guard. She was angry of her husband, Pablo because the latter had burned her
clothing, was maintaining a mistress and had been taking all the food from their house. Upon
reaching the NAWASA Building, she knocked at the door. Immediately, after the door was
opened, Pablo shouted at Mananquil. Mananquil then got the bottle of gasoline and poured the
contents thereof on the face of the victim, got a matchbox and set the polo shirt of the victim a
flame. She admitted having burned the victim, who later died. But when she took the witness
stand, 5 years after the incident, she disclaimed any responsibility for the burning of her husband.
ISSUE: Whether or not the accused caused the death of her husband and is guilty of parricide.

HELD: Yes. It concluded that with pneumonia having developed, the burns became as to the
cause of death, merely contributory. Art. 4, Par. 1 of the Revised Penal Code provides that
“Criminal liability shall be incurred. 1. By any person committing a felony (delito) although the
wrongful act done be different from that which he intended. the essential requisites of which
are: (a) that an intentional felony has been committed; and (b) that the wrong done to the
aggrieved party be the direct, natural and logical consequence of the felony committed by the
offender.” The fact that other causes contribute to the death does not relieve the actor of
responsibility. He would still be liable "even if the deceased might litem recovered if he had taken
proper care of himself, or submitted to surgical operation, or that unskilled or improper
treatment aggravated the wound and contributed to the death, or that death was men." caused
by a surgical operation rendered necessary by the condition of the wound. The principle on which
this rule is founded is one of universal application. It lies at the foundation of criminal
jurisprudence.

38
CRIMINAL LAW I – DIGESTED CASES BY: BOAQUINA, MARY CRISCHAEL
39

QUINTO V. ANDRES
G.R. No. 155791. March 16, 2005

FACTS: Eleven-year-old Edison Garcia, a Grade 4 elementary school pupil, and his playmate,
Wilson Quinto, who was also about eleven years old, were at Barangay San Rafael, Tarlac, Tarlac.
They saw respondents Dante, Andres and Randyver Pacheco by the mouth of a drainage culvert.
Andres and Pacheco invited Wilson to go fishing with them inside the drainage culvert. Wilson
assented. When Garcia saw that it was dark inside, he opted to remain seated in a grassy area
about two meters from the entrance of the drainage system.

Respondent Pacheco had a flashlight. He entered the drainage system which was covered by
concrete. After a while, respondent Pacheco, who was holding a fish, came out of the drainage
system and left without saying a word. Respondent Andres also came out, went back inside, and
emerged again, this time, carrying Wilson who was already dead. Respondent Andres laid the
boy's lifeless body down in the grassy area. Shocked at the sudden turn of events, Garcia fled
from the scene. For his part, respondent Andres went to the house of petitioner Melba Quinto,
Wilson's mother, and informed her that her son had died. Melba Quinto rushed to the drainage
culvert while respondent Andres followed her. According to the autopsy, the cause of death was
Asphyxia by drowning and traumatic head injuries, as contributory.
ISSUE: Whether or not respondents are liable for the death of the victim

HELD: No. The trial court gave credence to the testimony of Dr. Aguda that the deceased might
have slipped, causing the latter to fall hard and hit his head on the pavement. However, the
absence of any ill-motive to kill the deceased is relevant and admissible in evidence to prove that
no violence was perpetrated on the person of the deceased. In this case, the petitioner failed to
adduce proof of any ill-motive on the part of either respondent to kill the deceased before or
after the latter was invited to join them in fishing. Indeed, the petitioner testified that
respondent Andres used to go to their house and play with her son before the latter's death.

39
CRIMINAL LAW I – DIGESTED CASES BY: BOAQUINA, MARY CRISCHAEL
40

PEOPLE V. QUIANZON
G.R. No. 42607. September 28, 1935

FACTS: Andres Aribuabo, one of the persons present, went to ask for food of Juan Quianzon, then
in the kitchen, who, to all appearances, had the victuals in his care. It was the second or third
time that Aribuabo approached Quianzon with the same purpose whereupon the latter, greatly
peeved, took hold of a firebrand and applied ran to the place where the people were gathered
exclaiming that he is wounded and was dying. Raising his shirt, he showed to those present a
wound in his abdomen below the navel. Aribuabo died as a result of this wound on the tenth day
after the incident.

ISSUE: The question to be determined is who wounded Aribuabo.

HELD: The defense of the accused consisted simply in denying that he had wounded the deceased
and that he had confessed his guilt to the witnesses Bagabay, Dumlao and Llaguno. But such
denial cannot prevail against the adverse testimony of these three veracious and disinterested
witnesses, all the more because neither the accused nor any other witness for the defense has
stated or insinuated that another person, not the accused, might be the author of the wound
which resulted in Aribuabo's death, and because it is admitted by the defense that it was the
accused, whom Aribuabo had been pestering with request for food, who attacked the latter,
burning his neck with a firebrand. It is contended by the defense that even granting that it was
the accused who inflicted the wound which resulted in Aribuabo's death, he should not be
convicted of homicide but only of serious physical injuries because said wound was not
necessarily fatal and the deceased would have survived it had he not twice removed the drainage
which Dr. Mendoza had placed to control or isolate the infection. This contention is without
merit. The infection was caused by the fecal matter from the large intestine which had been
perforated. The possibility that the patient might have survived said wound had he not removed
the drainage, does not mean that that act of the patient was the real cause of his death.
"One who inflicts an injury on another is deemed by the law to be guilty of homicide if the injury
contributes mediately or immediately to the death of such other. The fact that other causes
contribute to the death does not relieve the actor of responsibility

The settled rule is that, one who inflicts an injury on another will be held responsible for his death,
although it may appear that the deceased might have recovered if he had taken proper care of
himself, or submitted to a surgical operation, or that unskilled or improper treatment aggravated
the wound and contributed to the death, or that death was immediately caused by a surgical
operation rendered necessary by the condition of the wound.
The principle on which this rule is founded is one of universal application, and lies at the
foundation of all criminal jurisprudence. It is, that every person is to be held to contemplate and
to be responsible for the natural consequences of his own acts.

40
CRIMINAL LAW I – DIGESTED CASES BY: BOAQUINA, MARY CRISCHAEL
41

URBANO V. IAC
G.R. No. 72964. January 7, 1988

FACTS: Urbano was on his way to his rice field. He found the place where he stored palay flooded
with water coming from the irrigation canal. Urbano went to the elevated portion to see what
happened, and there he saw Marcelino Javier and Emilio Efre cutting grass. Javier admitted that
he was the one who opened the canal. A quarrel ensued, and Urbano hit Javier on the right palm
with his bolo, and again on the leg with the back of the bolo. 5 days after, Urbano and Javier had
an amicable settlement. Urbano paid P700 for the medical expenses of Javier. On November,
Urbano was rushed to the hospital where he had lockjaw and convulsions. The doctor found the
condition to be caused by tetanus toxin which infected the healing wound in his palm. He died
the following day. Urbano was charged with homicide and was found guilty both by the trial court
and on appeal by the Court of Appeals. Urbano filed a motion for new trial based on the affidavit
of the Barangay Captain who stated that he saw the deceased catching fish in the shallow
irrigation canals on November 5. The motion was denied; hence, this petition.

ISSUE: Whether the wound inflicted by Urbano to Javier was the proximate cause of the latter’s
death

HELD: No. Proximate cause is "that cause, which, in natural and continuous sequence, unbroken
by any efficient intervening cause, produces the injury, and without which the result would not
have occurred."
If the wound of Javier inflicted by the appellant was already infected by tetanus germs at the
time, it is more medically probable that Javier should have been infected with only a mild cause
of tetanus because the symptoms of tetanus appeared on the 22nd day after the hacking incident
or more than 14 days after the infliction of the wound. But at the time Javier's wound was
inflicted, the severe form of tetanus that killed him was not yet present.
The rule is that the death of the victim must be the direct, natural, and logical consequence of
the wounds inflicted upon him by the accused. And since we are dealing with a criminal
conviction, the proof that the accused caused the victim's death must convince a rational mind
beyond reasonable doubt. The medical findings, however, lead us to a distinct possibility that the
infection of the wound by tetanus was an efficient intervening cause later or between the time
Javier was wounded to the time of his death. The infection was, therefore, distinct and foreign
to the crime.
At the very least, the records show he is guilty of inflicting slight physical injuries. However, the
petitioner's criminal liability in this respect was wiped out by the victim's own act. There is a
likelihood that the wound was but the remote cause and its subsequent infection, for failure to
take necessary precautions, with tetanus may have been the proximate cause of Javier's death
with which the petitioner had nothing to do. If no danger existed in the condition except because
of the independent cause, such condition was not the proximate cause. And if an independent
negligent act or defective condition sets into operation the instances which result in injury
because of the prior defective condition, such subsequent act or condition is the proximate
cause.

41
CRIMINAL LAW I – DIGESTED CASES BY: BOAQUINA, MARY CRISCHAEL
42

INTOD V. CA
G.R. No. 103119. October 21, 1992

FACTS: Intod, Pangasian, Tubio and Daligdig went to Mandaya's house and asked him to go with
them to the house of Bernardina Palangpangan. Thereafter, Mandaya and Intod, Pangasian,
Tubio and Daligdig had a meeting with Aniceto Dumalagan. He told Mandaya that he wanted
Palangpangan to be killed because of a land dispute between them and that Mandaya should
accompany the four (4) men, otherwise, he would also be killed. At about 10pm of the same day,
Intod, Mandaya, Pangasian, Tubio and Daligdig, all armed with firearms, arrived at
Palangpangan's house. Mandaya pointed the location of Palangpangan's bedroom. Thereafter,
fired at said room.

It turned out, however, that Palangpangan was in another city and her home was then occupied
by her son-in-law and his family. No one was in the room when the accused fired the shots. No
one was hit by the gun fire. Petitioner and his companions were positively identified by witnesses.

ISSUE: Whether or not the crime committed was attempted murder.

HELD: No. Court ruled that the an Impossible Crime was committed. Article 4(2) of the RPC
provides that “Criminal responsibility shall be incurred … By any person performing an act which
would be an offense against persons or property, where it not for the inherent impossibility of
its accomplishment or on account of the employment of inadequate or ineffectual means.”

The offense cannot be produced because the commission of the offense is inherently impossible
of accomplishment is the focus of this petition. To be impossible under this clause, the act
intended by the offender must be by its nature one impossible of accomplishment. There must
be either (1) legal impossibility, or (2) physical impossibility of accomplishing the intended act in
order to qualify the act as an impossible crime.

42
CRIMINAL LAW I – DIGESTED CASES BY: BOAQUINA, MARY CRISCHAEL
43

PEOPLE V. DOMASIAN
G.R. No. 95322. March 1, 1993

FACTS: Enrico was walking with a classmate along Roque street, he was approached by Domasian
who requested his assistance in getting his father's signature on a medical certificate. Enrico
agreed to help and rode with the man in a tricycle to Calantipayan. The two then boarded a
tricycle headed for San Vicente, with the man still firmly holding Enrico, who continued crying.
This aroused the suspicion of the driver, Alexander Grate, who asked the man about his
relationship with the boy. The man said he and the boy were brothers, making Grate doubly
suspicious because of the physical differences between the two and the wide gap between their
ages. Grate immediately reported the matter to two barangay tanods when his passengers
alighted from the tricycle. Grate and the tanods went after the two and saw the man dragging
the boy.

Domasian managed to escape, leaving Enrico behind. After Enrico's return, Enrico’s father
received an envelope containing a ransom note. The note demanded P1 million for the release
of Enrico and warned that otherwise the boy would be killed. Enrico’s father thought the
handwriting in the note was familiar. After comparing it with some records in the hospital, he
gave the note to the police, which referred it to the NBI for examination. The test showed that it
had been written by Dr. Samson Tan. Enrico was shown a folder of pictures in the police station
so he could identify the man who had detained him, and he pointed to the picture of Pablito
Domasian. Domasian and Tan were subsequently charged with the crime of kidnaping with
serious illegal detention.

ISSUE: Whether or not that the sending of the ransom note was an impossible crime which Tan
says he is not punishable.

HELD: No, it is not an impossible crime. Even before the ransom note was received, the crime of
kidnaping with serious illegal detention had already been committed. The act cannot be
considered an impossible crime because there was no inherent improbability of its
accomplishment or the employment of inadequate or ineffective means. The delivery of the
ransom note after the rescue of the victim did not extinguish the offense, which had already been
consummated when Domasian deprived Enrico of his liberty. The sending of the ransom note
would have had the effect only of increasing the penalty to death under the last paragraph of
Article 267 although this too would not have been possible under the new Constitution.

43
CRIMINAL LAW I – DIGESTED CASES BY: BOAQUINA, MARY CRISCHAEL
44

ARTICLE 6: Stages of Felonies

US VS. EDUAVE
G.R. No. L-12155. February 22, 1917

FACTS: Eduave attacked the victim from behind using a bolo creating a gash of 8 and a half inches
long and 2 inches deep because the latter accused the defendant of raping her. Knowing that he
has already killed the victim, he threw the body into the bushes and left. Then, he gave himself
up and declared that he had killed the complainant.

ISSUE: In what stage of the crime of murder is committed by Eduave

HELD: The Accused is guilty of frustrated murder. The fact that Eduave attacked the victim from
behind, in a vital portion of the body, the crime, however, is not consummated by reason of the
intervention of causes independent of the will of the offender. Edueva did all that was necessary
to commit the crime but it did not result as a consequence due to something beyond his control.

The crime was not consummated because the elements of the crime’s execution and
accomplishment were not complete, as the victim did not die. Neither was the crime an
attempted one because the accused’s actions has already passed the subjective phase, that is,
there was no external force or intervention of a foreign or extraneous cause or agency between
the beginning of the commission of the crime and the moment when all of the acts have been
performed preventing defendant from performing all the acts of execution necessary to commit
the felony. That external force is the essential element which distinguishes attempted from
frustrated felony. Consequently, the victim did not die because an external element has
prevented such death after Eduave has performed all the necessary acts of execution that would
have caused the death of the victim.

44
CRIMINAL LAW I – DIGESTED CASES BY: BOAQUINA, MARY CRISCHAEL
45

PEOPLE V. ORITA
G.R. No. 88724. April 3, 1990

FACTS: ICristina Abayan arrived at her boarding house coming from a party. While she was
knocking at the door of her boarding house, someone held her and poked a knife at her neck.
She recognized the person as Lito Orita because he is a frequent visitor of another boarder. Lito
dragged the complainant and ordered her to look for a room. After entering the room, the
accused pushed the complainant and ordered her to undress herself. He mounted the victim and
tried to penetrate her but only a small portion was inserted because of the constant moving of
the victim. The accused lay down and ordered the victim to mount him and the victim used this
chance to escape. She dashed to the next room while the accused pursued her until she was able
to jump out through the window. She ran towards the municipal building and knocked on the
front door. When there was no answer she rushed to the back of the building. When the
policemen went out, they found her sitting naked while crying.

The accused contends that there is no crime of frustrated rape. The trial court erred in
disregarding the substantial inconsistencies in the testimonies of the witnesses; and the trial
court erred in declaring that the crime of frustrated rape was committed by the accused. He was
not able to fully penetrate in her. The accused also questions also the failure of the prosecution
to present other witnesses to corroborate the allegations in the complaint. The accused used the
Article 266 of the RPC to show that he is not guilty of frustrated rape, and Article 6 to stress the
difference of consummated, frustrated, and attempted felonies.

The victim's testimony from the time she knocked on the door of the municipal building up to
the time she was brought to the hospital was corroborated by Pat. Donceras. Rather than
discredit the testimonies of the prosecution witnesses, discrepancies on minor details must be
viewed as adding credence and veracity to such spontaneous testimonies. The accused
committed rape.

ISSUE: Whether or not the frustrated stage applies to the crime of rape

HELD: No. In the crime of rape, from the moment the offender has carnal knowledge of his victim
he actually attains his purpose and, from that moment also all the essential elements of the
offense have been accomplished. Nothing more is left to be done by the offender, because he
has performed the last act necessary to produce the crime. Thus, the felony is consummated.
We have set the uniform rule that for the consummation of rape, perfect penetration is not
essential. Any penetration of the female organ by the male organ is sufficient. Entry of the labia
or lips of the female organ, without rupture of the hymen or laceration of the vagina is sufficient
to warrant conviction.

45
CRIMINAL LAW I – DIGESTED CASES BY: BOAQUINA, MARY CRISCHAEL
46

PEOPLE V. CAMPUHAN
G.R. No. 129433. March 30, 2000

FACTS: Campuhan was a helper in the business of the family of the victim, a 4-year-old girl. One
time, the mother of the victim heard the latter cry, “Ayoko!”, prompting her to rush upstairs.
There, she saw Campuhan kneeling before the victim, whose pajamas and panty were already
removed, while his short pants were down to his knees. Campuhan was apprehended. Physical
examination of the victim yielded negative results. No evident sign of extra-genital physical injury
was noted. Her hymen was intact and its orifice was only .5 cm in diameter. Trial court found him
guilty of statutory rape and sentenced him to death.

ISSUE: Whether or not Campuhan is guilty of statutory rape.

HELD: No. The gravamen of the offense of statutory rape is carnal knowledge of woman below
12 as provided in RPC 335(3). The victim was only 4 years old when the molestation took place,
thus raising the penalty from “reclusion perpetua to death” to the single indivisible penalty of
death under RA 7659 Sec. 11, the offended party being below 7 years old.
In concluding that carnal knowledge took place, full penetration of the vaginal orifice is not an
essential ingredient, nor is the rupture of hymen necessary; the mere touching of external
genitalia by the penis capable of consummating the sexual act is sufficient to constitute carnal
knowledge. But the act of touching should be understood as inherently part of the entry of penis
into the labia of the female organ, and not mere touching alone of the mons pubis or the
pudendum (the part instantly visible within the surface). Absent any showing of the slightest
penetration of the female organ, i.e., touching of either labia by the penis, there can be no
consummated rape; at most, it can only be attempted rape, if not acts of lasciviousness. Here,
the prosecution failed to discharge its onus of proving that Campuhan’s penis was able to
penetrate the victim’s vagina however slight. Also, there were no external signs of physical
injuries on the victim’s body to conclude that penetration had taken place.

Crime committed was attempted rape. Under RPC 6 in relation to RPC 335, rape is attempted
when the offender commences the commission of rape directly by overt acts, and does not
perform all acts of execution which should produce the crime of rape by reason of some cause
or accident other than his own spontaneous desistance.. For attempted rape, there was no
penetration of the female organ because not all acts of execution were performed or the
offender merely commenced the commission of the felony directly by overt acts. All the elements
of attempted rape are present in this case

46
CRIMINAL LAW I – DIGESTED CASES BY: BOAQUINA, MARY CRISCHAEL
47

BALEROS V. PEOPLE
G.R. No. 138033. February 22, 2006

FACTS: Renato Baleros went to the dormitory room of Malou, placed himself on top of her, and
pressed a handkerchief soaked in chloroform. Malou struggled to free herself in the hands of
Baleros and succeeded by grabbing his sexual organ and squeezing it. Malou went straight to
Marvilou and said “may pumasok sa kuarto ko pinagtangkaan ako”.
The morning after, the police said to the tenants of the dormitory to grab the things that are
theirs. The room was left with an unclaimed bag which Christian, one of the tenants, knew right
away that was Renato’s. Among the contents of the bag was a handkerchief with a volatile
substance. They later found out that Renato was a suitor of Malou which she rejected a week
ago.

ISSUE: Whether or not Renato is guilty of attempted rape.

HELD: No. There is absolutely no dispute about the absence of sexual intercourse or carnal
knowledge in the present case. The next question that thus comes to the fore is whether or not
the act of the petitioner, i.e., the pressing of a chemical-soaked cloth while on top of Malou,
constitutes an overt act of rape.
Overt or external act has been defined as some physical activity or deed, indicating the intention
to commit a particular crime, more than a mere planning or preparation, which if carried out to
its complete termination following its natural course, without being frustrated by external
obstacles nor by the voluntary desistance of the perpetrator, will logically and necessarily ripen
into a concrete offense.

It cannot be overemphasized that petitioner was fully clothed and that there was no attempt on
his part to undress Malou, let alone touch her private part. For what reason petitioner wanted
the complainant unconscious, if that was really his immediate intention, is anybody’s guess.
Renato was acquitted of the charge of attempted rape. However, he, is adjudged guilty of light
coercion.

47
CRIMINAL LAW I – DIGESTED CASES BY: BOAQUINA, MARY CRISCHAEL
48

CRUZ V. PEOPLE
G.R. No. 166441. October 8, 2014

FACTS: Norberto Cruz and his wife employed AAA and BBB to help them in selling their plastic
wares and glass wares in La Union. Upon reaching the place, they set up their tents to have a
place to sleep. Petitioner’s wife and their driver went back to Manila to get more goods. While
sleeping, AAA felt that somebody was on top of her mashing her breast and touching her private
part. Norberto ordered her not to scream or she will be killed. AAA fought back and Norberto
was not able to pursue his lustful desires. AAA left the tent to seek for help. When she returned
to their tent, she saw Norberto touching the private parts of BBB. This prompted Norberto to
leave the tent.
Norberto denies the commission of the crime alleging that he could not possibly do the acts
imputed out in the open as there were many people preparing for the “simbang gabi”. He further
assails the credibility AAA for the crime of rape, alleging that the complaints were filed only for
the purpose of extorting money from him.

ISSUE: Whether or not petitioner guilty of attempted rape against AAA

HELD: NO. He is guilty only of acts of lasciviousness. The basic element of rape is carnal
knowledge of a female. Rape is consummated once the penis capable of consummating the
sexual act touches the external genitalia of the female. There must be sufficient and convincing
proof that the penis indeed touched the labias or slid into the female organ, and not merely
stroked the external surface thereof, for an accused to be convicted of consummated rape.
Rape in its frustrated stage is a physical impossibility. Nonetheless, rape admits of an attempted
stage. In attempted rape, the concrete felony is rape, but the offender does not perform all the
acts of execution of having carnal knowledge. The petitioner climbed on top of the naked victim
and was already touching her genitalia with his hands and mashing her breasts when she freed
herself from his clutches and effectively ended his designs on her. Yet, inferring from such
circumstances that rape, and no other, was his intended felony would be highly unwarranted.
Such circumstances remained equivocal, or “susceptible of double interpretation,” such that it
was not permissible to directly infer from them the intention to cause rape as the particular
injury.

48
CRIMINAL LAW I – DIGESTED CASES BY: BOAQUINA, MARY CRISCHAEL
49

VALENZUELA V. PEOPLE
G.R. No. 160188. June 21, 2007

FACTS: Petitioner was sited outside the Super Sale Club by a SG hauling a push cart with cases of
detergent. These were loaded in a taxi. Upon leaving the parking area, the SG proceed to stop
the taxi. He asked for the receipt but petitioner reacted by fleeing on foot. He was then
apprehended and the stolen merchandise recovered. RTC convicted him. On appeal, he argued
that he should only be convicted of frustrated theft since at the time he was apprehended, he
was never placed in a position to freely dispose of the articles stolen. CA rejected such
contention. Hence, this case.

ISSUE: Whether or not the only crime committed was frustrated theft

HELD: No. The moment petitioner obtained physical possession of the cases of detergent and
loaded them in the pushcart, such seizure motivated by intent to gain, completed without need
to inflict violence or intimidation against persons nor force upon things, and accomplished
without the consent of the SM Super Sales Club, petitioner forfeited the extenuating benefit a
conviction for only attempted theft would have afforded him.
There is no language in Article 308 that expressly or impliedly allows that the free disposition of
the items stolen is in any way determinative of whether the crime of theft has been produced.
We thus conclude that under the Revised Penal Code, there is no crime of frustrated theft. As
petitioner has latched the success of his appeal on our acceptance of the Dio and Flores rulings,
his petition must be denied, for we decline to adopt said rulings in our jurisdiction.

49
CRIMINAL LAW I – DIGESTED CASES BY: BOAQUINA, MARY CRISCHAEL
50

PEOPLE V. LAMAHANG
G.R. No. L-43530. August 3, 1935

FACTS: Policeman Jose Tomambing was patrolling his beat on Delgado and CR Funetes streets in
Iloilo. He caught Lamahang in the act of making an opening on the wall of a store of cheap goods,
using an iron bar. At that time, the owner of the store was sleeping inside with another Chinaman.
The accused only succeeded in breaking and in unfastening another from the wall, when the
policeman showed up, who instantly arrested him and placed him under custody.

ISSUE: Whether or not the accused was erroneously declared guilty of attempted robbery

HELD: YES.The accused is then held guilty of attempted trespass to dwelling, committed by
means of force, with the aforesaid aggravating and mitigating circumstances and sentenced to
three months and one day of arresto mayor.

It is necessary to prove that said beginning of execution, if carried to its complete termination
following its natural course, without being frustrated by external obstacles nor by the voluntary
desistance of the perpetrator, will logically and necessarily ripen into a concrete offense. In the
case of robbery, it must be shown that the offender clearly intended to take possession, for the
purpose of gain, of some personal property belonging to another. In the instant case, it may only
be inferred as a logical conclusion that his evident intention was to enter by means of force said
store against the will of its owner. That his final objective, once he succeeded in entering the
store, was to rob, to cause physical injury to the inmates, or to commit any other offense, there
is nothing in the record to justify a concrete finding.

It must be borne in mind (I Groizard, p. 99) that in offenses not consummated, as the material
damage is wanting, the nature of the action intended (accion fin) cannot exactly be ascertained,
but the same must be inferred from the nature of the acts executed (accion medio). The relation
existing between the facts submitted for appreciation and the offense which said facts are
supposed to produce must be direct; the intention must be ascertained from the facts and
therefore it is necessary, in order to avoid regrettable instances of injustice.

50
CRIMINAL LAW I – DIGESTED CASES BY: BOAQUINA, MARY CRISCHAEL
51

PEOPLE V. SALVILLA
G.R. No. 86163. April 26, 1990

FACTS: Salvilla together with his co-accused armed with homemade guns and hand grenade
robbed Rodita Habiero in the latter’s office. In the office of Rodita; her two daughters Mary and
Mimmie were also inside. One of the accused asks Mary to get the paper bag which contained
money. All accused held victims as hostage when the police and military authorities had
surrounded the lumber yard. After the negotiation fails to proceed, the police makes their move
in assaulting the robbers thus Mary and Mimmie are injured as well the accused also got an injury.

ISSUE: Whether or not the crime of robbery was consummated.

HELD: Yes. Robbery shall be deemed consummated if the unlawful “taking” is complete. Unlawful
taking of personal property of another is an essential part of the crime of robbery. The
respondent claimed that none of the items (money, watches and wallet) were recovered from
them. However, based on the evidence, the money demanded, the wallet and the wristwatch
were within the dominion and control of the appellant and his co-accused and thus the taking
was completed.
It is not necessary that the property be taken into the hands of the robber or that he should have
actually carried the property away, out of the physical presence of the lawful possessor, or that
he should have made his escape with it.

51
CRIMINAL LAW I – DIGESTED CASES BY: BOAQUINA, MARY CRISCHAEL
52

EPIFANO V. PEOPLE
G.R. No. 157057. June 26, 2007

FACTS: Crisaldo Alberto and his cousin, Allan were walking to their respective homes in a narrow
pathway. Allan walked ahead of Crisaldo. Suddenly, Crisaldo felt the piercing thrust of a bladed
weapon on his back, which caused him to cry out inpain. He made a quick turnaround and saw
his attacker, Epifano, also known as "Iyo (Uncle) Kingkoy." Epifano stabbed Crisaldo again but
only hit the latter's left arm. When Allan heard Crisaldo's outcry, he rushed to Crisaldo's side
which caused petitioner to run away. Allan then brought Crisaldo to his father's house where
Crisaldo's wounds were wrapped in a blanket. Crisaldo was then brought to the Peñaplata
Hospital where he was given first aid and then transferred to the Davao Medical Center where
he stayed for three weeks to recuperate from his wounds. Subsequently, Epifano was charged
with Frustrated Murder.

ISSUE: Whether or not the accused was guilty of frustrated murder.

HELD: No. Petitioner failed to perform all the acts of execution, because Allan came to the aid of
Crisaldo and petitioner was forced to scamper away. He did not voluntarily desist from stabbing
Crisaldo, but he had to stop stabbing when Allan rushed to help Crisaldo and recognized
petitioner. Thus, the subjective phase of the crime had not been completed. It is well-settled that
where there is nothing in the evidence to show that the wound would be fatal if not medically
attended to, the character of the wound is doubtful; hence, the doubt should be resolved in favor
of the accused and the crime committed by him may be declared as attempted, not frustrated
murder.

52
CRIMINAL LAW I – DIGESTED CASES BY: BOAQUINA, MARY CRISCHAEL
53

PEOPLE V. SY PIO
G.R. Nos. L-3827-28. November 23, 1951

FACTS: Sy Pio shot three people early in the morning of September 3, 1949. Tan Siong Kiap, Ong
Pian and Jose Sy. Sy Pio entered the store at 511 Misericordia Sta Cruz Manila and started firing
with a .45 caliber pistol. First to be shot was Jose Sy. Upon seeing Sy Pio fire at Jose Sy, Tan asked,
“what is the idea?” Thereupon, Sy Pio turned around and fired at him as well. Tan was shot at his
right shoulder and it passed through his back. He ran to a room behind the store to hide. He was
still able to hear gunshots from Sy Pio‘s pistol, but afterwards, Sy Pio ran away.

Tan Siong Kiap was brought to the Chinese General Hospital where his wound was treated.
According to Sy Pio‘s declaration, some months prior to the incident, he was employed in a
restaurant owned by Ong Pian. Sy Pio‘s wife, Vicenta was also employed by Ong Pian‘s partner.
When he tried to borrow money from Ong Pian fpr his wife‘s sick father, Ong Pian only lent him
P1. Ong Pian presented a list of Sy Pio‘s debts and these were deducted from his wife‘s monthly
salary. Sy Pio could not remember incurring such debts. As such, he was resentful of Ong Pian‘s
conduct.

In Tan Siong Kiap‘s case, a few months before Sept 3, Sy Pio was able to realize the sum of P70
and he put his money in a place in his room. The next day, Sy Pio found that his money was gone.
Tan told Sy Pio that he had probably given the money to his wife. Early in the morning of Sept 3,
while Ngo Cho, a Chinaman who has a pistol was away, he got his pistol and went to a restaurant
in Ongpin where Ong Pian worked and shot him. Afterwards he went to Sta Cruz and shot Jose
Sy and Tan.

ISSUE: Whether or not the defendant-appellant perform all the acts of execution necessary to
produce the death of his victim

HELD: No. While the intent to kill is conclusively proved the wound inflicted was not necessarily
fatal, because it did not touch any of the vital organs of the body. As a matter of fact, the medical
certification issued by the physician who examined the wound of the offended party at the time
he went to the hospital, states that the wound was to heal within a period of fourteen days. The
fact that he was able to escape which appellant must have seen, must have produced in the mind
of the defendant-appellant that he was not able to hit his victim at a vital part of the body. The
defendant appellant knew that he had not actually performed all acts of execution necessary to
kill his victim. Under these circumstances, it cannot be said that the subjective phase of the acts
of execution had been completed.
We only find him guilty of attempted murder, because he did not perform all the acts of
execution, actual and subjective, in order that the purpose and intention that he had to kill his
victim might be carried out.

53
CRIMINAL LAW I – DIGESTED CASES BY: BOAQUINA, MARY CRISCHAEL
54

PEOPLE V. RAVELO
G.R. No. 78781-82. October 15, 1991

FACTS: Accused-appellants assaulted, attacked, and burned Reynaldo Gaurano and latter die as
consequence thereof. The next day, the accused-appellants kidnapped by means of force Joey
Lugatiman and was brought to Ravelo's house where he was tortured. Lugatiman was able to
escape.

Lugatiman reported what happened to him and to Gaurano to the police authorities. RTC
convicted the accused-appellants of murder of Gaurano and frustrated murder of Lugatiman.
In this appeal, counsel contends that there can be no frustrated murder absent any proof of
intent to kill, which is an essential element of the offense of frustrated murder.

ISSUE: Whether the statement by the accused stating that “Lugatiman” would be killed is
sufficient proof of intent to convict a person of frustrated murder.

HELD: No. The offender must have the intent or the actual design to kill which must be
manifested by external acts. A verbal expression is not sufficient to show an actual design to
perpetrate the act. Intent must be shown not only by a statement of the aggressor, but also by
the execution of all acts and the use of means necessary to deliver a fatal blow while the victim
is not placed in a position to defend himself.

54
CRIMINAL LAW I – DIGESTED CASES BY: BOAQUINA, MARY CRISCHAEL
55

VELASCO V. PEOPLE
G.R. No. 166479. February 28, 2006

FACTS: Frederick Maramba was cleaning and washing his owner type jeep in front of his house
when a motorized tricycle stopped near him. Rodolfo C. Velasco dashed out of the tricycle,
approached the complainant and fired at him several times. Velasco missed his first shot but the
second one hit the complainant at the upper arm, causing him to stumble on the ground. But,
Frederick stood up and ran, while Velasco fired 6 more but missed. After being reported as
wearing a vest or a “chaleco”, the police, composed of SPO4 Romulo Villamil, PO3 Rolando
Alvendo, and SPO1 Soliven pursued and caught Velasco who was on board a motorized tricycle
to the highway going to Barangay Banaoang in Calasiao town with a firearm protruding from the
waistline. RTC charged the accused of attempted murder appreciating treachery in the
commission of the crime.

ISSUE: Whether or not Velasco is guilty of attempted murder

HELD: YES. It was not physically impossible for Velasco to be at the crime scene when the crime
was committed since it only takes a 10-minute ride from the place where he allegedly alighted
from the car of one Berting Soriano to the crime scene. Even without a ballistic report, the
positive identification by prosecution witnesses is more than sufficient to prove accused’s guilt
beyond reasonable doubt. It must be stressed that motive is a state of (one’s) mind which others
cannot discern. It is not an element of the crime, and as such does not have to be proved.
In fact, lack of motive for committing a crime does not preclude conviction. It is judicial
knowledge that persons have been killed or assaulted for no reason at all. Even in the absence of
a known motive, the time-honored rule is that motive is not essential to convict when there is no
doubt as to the identity of the culprit. Motive assumes significance only where there is no
showing of who the perpetrator of the crime was. Since petitioner has been positively
identified the lack of motive is no longer of consequence. The relationship could strengthen the
witnesses’ credibility, for it is unnatural for an aggrieved relative to falsely accuse someone other
than the actual culprit. The fact that the shooting occurred in broad daylight does not render its
commission impossible. The fact that petitioner was a navy man, a protector of the people, does
not mean that he is innocent of the crime charged or that he is incapable of doing it.

The suddenness of the shooting and the fact that he was unarmed left private complainant with
no option but to run for his life. – treachery. Having commenced the criminal act by overt acts
but failing to perform all acts of execution as to produce the felony by reason of some cause
other than his own desistance, petitioner committed an attempted felony. Petitioner already
commenced his attack with a manifest intent to kill by shooting private complainant seven times,
but failed to perform all the acts of execution by reason of causes independent of his will, that is,
poor aim and the swiftness of the latter. Private complainant sustained a wound on the left arm
that is not sufficient to cause his death. The settled rule is that where the wound inflicted on the
victim is not sufficient to cause his death, the crime is only attempted murder, since the accused
did not perform all the acts of execution that would have brought about death.

55
CRIMINAL LAW I – DIGESTED CASES BY: BOAQUINA, MARY CRISCHAEL
56

PEOPLE V. ALMAZAN
G.R. Nos. 138943-44. September 17, 2001

FACTS: Henry Almazan went home accompanied by his friend Johnald Molina. His wife informed
him upon his return that his twelve fighting cocks had been stolen. He then proceeded to search
for them and ended up in Vicente Madriaga’s house where the latter was playing chess with a
certain Allan. The spectators were Vicente’s son Noli carrying his 2-year old daughter, his
grandson Noel, and neighbor Angel Soliva.

Henry Almazan brandished a revolver in front of the group since he suspected Angel to be the
culprit behind the theft of his fighting cocks. He aimed at Angel and fired twice but to no avail.
Vicente tried to calm Henry down while Angel ran away but the accused-appellant aimed instead
at Noli, fired his gun, and killed him. He then turned to Noel and shot him on the thigh. Both Noli
and Noel were rushed to the hospital; the former dying along the way and latter surviving from
a mere minor injury from the gunshot.-Henry contested that his acts were in self-defense.
According to him, upon arrival at Vicente’s house, the group was drinking liquor, mocking him,
and threatening him of physical violence. He claims that Angel was the one that pulled out a .38
caliber revolver and aimed at him but misfired twice that ensued in a struggle for the weapon
between the two men. During the struggle, accused-appellant claims that Noli was accidentally
shot. After successfully wrestling the weapon away from Angel, Henry claims to have received a
blow from behind which caused him to fall and see Noel poised to attack him with a broken bottle
so he fired at the latter’s lower part of the body. His friend Johnald Molina corroborated his
story.-Accused-appellant was charged with murder qualified by treachery for the death of Noli
and frustrated murder for shooting Noel based on a statement of Noel’s attending physician, Dr.
Misael Jonathan Ticman, who said that if not medically treated, the wound might get infected or
lead to the victim’s death. He did send the victim home after undergoing treatment. Accused-
appellant went into hiding after the shooting incident and was caught eight months later.

ISSUE: Whether or not the accused is liable for attempted murder.

HELD: Yes. Testimonial evidence to be credible should not only come from the mouth of a
credible witness but should also be credible, reasonable and in accord with human experience,
failing which, it should be rejected.
For the claim of self-defense to the charge of murder aggravated by treachery, Almazan failed to
discharge the burden of proof to prove by clear and convincing evidence the elements thereof:
(a) that there was unlawful aggression on the part of the victim; (b) that there was reasonable
necessity for the means employed to prevent or repel it; and (c) that there was lack of sufficient
provocation on the part of the defendant. Almazan should be held liable for attempted murder,
not frustrated murder since the victim did not sustain a fatal wound that could have caused his
death were it not for timely medical assistance.
Henry Almazanis found guilty of Murder. However, his conviction for Frustrated Murder is
modified by lowering the crime to Attempted Murder.

56
CRIMINAL LAW I – DIGESTED CASES BY: BOAQUINA, MARY CRISCHAEL
57

PEOPLE V. KALALO
G.R. Nos. L-39303-05. March 17, 1934

FACTS: The appellants namely, Felipe Kalalo, Marcelo Kalalo, Juan Kalalo, and Gregorio Ramos,
were tried in the Batangas jointly with Alejandro Garcia, Fausta Abrenica and Alipia Abrenica in
criminal cases Nos. 6858,6859 and 6860, the first two for murder, and the last for frustrated
murder. Upon agreement of the parties said three cases were tried together and after the
presentation of their respective evidence, the said court acquitted Alejandro Garcia, Fausta
Abrenica and Alipia Abrenica, and sentenced the other appellants.

ISSUE: Whether or not accused-appellants are liable of the crimes of murder and discharge of
firearms

HELD: The first case is, for the alleged murder of Marcelino Panaligan, to seventeen years, four
months and one day of reclusion temporal, with the corresponding accessory penalties, and to
indemnify the heirs of the said deceased Marcelino Panaligan in the sum of P1,000, with the
costs. The second case is, for the alleged murder of Arcadio Holgado, to seventeen years, four
months and one day of reclusion temporal, with the corresponding accessory penalties, and to
indemnify the heirs of the aforesaid victim, the deceased Arcadio Holgado, in the sum of P1,000,
with the costs. In the third case, that is, the court held that the crime committed was simply that
of discharge of firearm, not frustrated murder, the appellant Marcelo Kalalo was sentenced to
one year, eight months and twenty-one days of prision correccional and to pay the proportionate
part of the costs of the proceedings. Felipe Kalalo and Juan Kalalo, as well as their co-accused
Fausta and Alipia Abrenica, Gregorio Ramos and Alejandro Garcia, were acquitted of the charges
therein. In all other respects, the appealed sentences in the said three cases are hereby affirmed
without prejudice to crediting the appellants therein with one-half of the time during which they
have undergone preventive imprisonment, in accordance with article 29 of the Revised Penal
Code.

57
CRIMINAL LAW I – DIGESTED CASES BY: BOAQUINA, MARY CRISCHAEL
58

PEOPLE V. LISTERIO
G.R. No. 122099. July 5, 2000

FACTS: Marlon and Jeonito Araque went to collect some of money from certain Tino. However,
they failed to collect anything. A group blocked their path and attacked them with lead pipes and
bladed weapons. They stabbed Jeonito from behind and Marlon was hit on the head with lead
pipes and momentarily lost consciousness. When Marlon regained his senses after 3minutes, he
saw that his brother was already dead. Marlon was brought to a hospital for treatment.

The accused were charged of Murder and Frustrated Homicide. Upon arraignment, accused
Listerio and Samson pleaded not guilty to the crimes charged. Their other co-accused have
remained at large. Samson escaped during the presentation of the prosecution’s evidence and
he was not tried.

ISSUE: Whether or not Listerio should be charged with Attempted Homicide for the reason that
none of the wounds sustained by Marlon Araque were fatal.

HELD: No. It is not the gravity of the wounds inflicted which determines whether a felony is
attempted or frustrated but whether or not the subjective phase in the commission of an offense
has been passed.

The subjective phase is that portion of the acts constituting the crime included between the act
which begins the commission of the crime and the last act performed by the offender which, with
the prior acts, should result in the consummated crime. It may also be said to be that period
occupied by the acts of the offender over which he has control – that period between the point
where he begins and the point where he voluntarily desists. If between these two points the
offender is stopped by reason of any cause outside of his own voluntary desistance, the
subjective phase has not been passed and it is an attempt. If he is not so stopped but continues
until he performs the last act, it is frustrated.

Subjectively the crime is complete. Nothing interrupted the offender while he was passing
through the subjective phase. The crime, however, is not consummated by reason of the
intervention of causes independent of the will of the offender. He did all that was necessary to
commit the crime. If the crime did not result as a consequence it was due to something beyond
his control. Accused-Appellant is found guilty of Frustrated Homicide

58
CRIMINAL LAW I – DIGESTED CASES BY: BOAQUINA, MARY CRISCHAEL
59

U.S. V. DOMINGUEZ
G.R. No. L-17021. February 23, 1921

FACTS: Dominguez, sold five copies of Sams' "Practical Business Letters," of the value P7.50 each.
The accused alleges that he did not deliver the money immediately after the sale, because the
cash boys were very busy as well as the cashier, while he had to go to the toilet for some
necessity, and upon coming out, the cashier caught him by the arm and asked him for the money,
and then he delivered the sum of pesos 7.50 to him; and that it was not his intention to make
use of said money. Such claim, nevertheless, does not exempt him from the criminal
responsibility which he had incurred, for the evidence before us shows clearly that he attempted
to defraud the "Philippine Education Co., Inc."

Upon being asked for the money, he first said that a woman, whom he did not know, bought
books, without having paid, for the reason that she was, according to herself, in a hurry; and,
latter, he went out of the store to talk to a friend who was employed in the Pacific Mail Steamship
Co. to tell him that if anyone should ask him if he (the employee of the Pacific Mail Steamship
Co.) bought books that morning in the store of the "Philippine Education Company" he should
answer affirmatively. Furthermore, he had also declared to the manager of the bookstore that
he used part of the money in purchasing postage stamps. There can be no doubt as to the injury
which the accused would have caused to the interests of the company in retaining for himself
the proceeds of the sale in question.

ISSUE: Whether or not the fact that the accused retained in his possession the proceeds of the
sale, delivering them to the cashier only after the deceit had been discovered is merely a
frustrated offense of estafa.

HELD: Yes. Court, applying Spanish doctrine to similar cases held that the appellant is guilty of
the frustrated offense of estafa of 37 ½ pesetas, inasmuch as he performed all the acts of
execution which should produce the crime as a consequence, but which, by reason of causes
independent of his will, did not produce it, no appreciable damage having been caused to the
offended party, such damage being one of the essential elements of the crime, due to the timely
discovery of the acts prosecuted.

The offense is frustrated when the accused performs all the acts of execution which would have
produced the crime, and, nevertheless, do not produce it by reason of causes independent of the
will of the actor, and that in said case the appellant, together with his co-accused attempted to
take possession of the two bundles which they believed were at the station, by going there and
presenting the tag, and they did not succeed because these bundles had already been taken,
which constitutes the frustrated crime.

59
CRIMINAL LAW I – DIGESTED CASES BY: BOAQUINA, MARY CRISCHAEL
60

POZAR V. CA
G.R. No. L-62439. October 23, 1984

FACTS: Manalo, Clerk at the Probation Office of Angeles City, declared that he started working at
the Probation Office since May 2, 1978 and came to know appellant because the latter had gone
to said office in connection with his application for probation. At about noon time of December
17, 1979, appellant came to the office looking for Probation Officer Danilo Ocampo and since the
latter was out at the time, appellant gave him a closed envelope bearing the name of Ocampo
for delivery to the latter. Two days later, he gave the envelope to Ocampo who opened the same
in his presence. The envelope contained some official papers connected with appellant's
application for probation and attached thereto was a hundred-peso bill. Ocampo then remarked:
“This is something bad that the opening of the envelope was done on December 19, 1979.”
Ocampo kept the envelope and its contents, including the one hundred-peso bill, but within a
week's time gave them to him with instructions to give the same to appellant but the latter never
came to the office and so he returned them to Ocampo. Although he later saw appellant about
two weeks after December 17, 1979, when the latter came to the office to sign some papers, he
never mentioned to appellant the one hundred-peso bill.

ISSUE: Whether or not the accused is guilty of direct bribery.

HELD: No. Petitioner was required by the Assistant Probation Officer, Primitive Francisco, to
submit in connection with his probation application the Court Information(complaint) Court
decision, Custody Status (recognizance or bail bond), clearances from the Police, the Court,
Barangay Certificate, I.D. pictures (3 copies), residence certificate, and told to report once a week
on Mondays. when the latter was asking permission to go to Baguio to submit to the office a copy
of his visa and passport. During all the time he was applying for probation, he made more or less
12 visits in the office as he was directed to report every Monday at 10:00 o'clock in the morning.
He reported for 6 to 7 consecutive weeks and there were times that he went there unscheduled
for conference and clarification of the various requirements he needed. Pozar was acquitted.

60
CRIMINAL LAW I – DIGESTED CASES BY: BOAQUINA, MARY CRISCHAEL
61

PEOPLE V. HERNANDEZ
G.R. No. L-31770. December 5, 1929

FACTS: Dayrit noticed that his house was on fire. Upon looking out of the window, he saw the
appellant beside the house with a stick. The appellant knew of the fact that Dayrit and his children
lived and were in the house that night. Upon investigation, it was found out that it was really the
appellant who set the fire to the house. The stick was found to have to have burnt end and a rag
soaked with petroleum dangling from it. The trial court held that the crime was only on the
frustrated stage.

ISSUE: Whether or not the trial court erred in ruling that the crime committed was only
frustrated.

HELD: Yes. The appellant did in fact, set fire to the roof of the house, and said house was in fact
partially burned. With this, the crime of arson was consummated, notwithstanding the fact that
the fire was afterwards extinguished, for, once the fire has been started, the consummation of
the crime of arson does not depend upon the extent of the damage cause.

61
CRIMINAL LAW I – DIGESTED CASES BY: BOAQUINA, MARY CRISCHAEL
62

U.S V. VALDEZ
GR No. L-14128 . December 10, 2018

FACTS: Mrs. Lewis’s house was seen to have smoke issuing from its lower floor. When she
ordered Banal to look for the source, it was found out that it came between a post of the house
and a partition of the entresol, a piece of a jute sack and a rag which were burning. At that
moment the defendant Valdes was in the entresol, engaged in his work of cleaning. He was later
on arrested and made a statement that he had set the fire to the said rag and piece of sack under
the house due to the inducement of other prisoners and that he had started the several other
fires which had occurred in said house on previous days.

ISSUE: Whether or not the accused has committed the crime of frustrated arson?

HELD: Yes. Setting fire to a jute sack and a rag, soaked with kerosene oil and placed beside an
upright of the house and a partition of the entresol of the building, thus endangering the burning
of the latter, constitutes the crime of frustrated arson of an inhabited house, on an occasion
when some of its inmates were inside of it.

The crime is classified only as frustrated arson, inasmuch as the defendant performed all the acts
conceive to the burning of said house, but nevertheless., owing to causes independent of his will,
the criminal act which he intended was not produced. The offense committed cannot be
classified as consummated arson by the burning of said inhabited house, for the reason that no
part of the building had yet commenced to burn, although, as the piece of sack and the rag,
soaked in kerosene oil, had been placed near partition of the entresol, the partition might have
started to burn, had the fire not been put out on time.

62
CRIMINAL LAW I – DIGESTED CASES BY: BOAQUINA, MARY CRISCHAEL
63

ARTICLE 8: Conspiracy

ASTUDILLO V. PEOPLE
G.R. No. 159734. November 29, 2006

DOCTRINE: Conspiracy exists when two or more persons come to an agreement concerning the
commission of the crime and decide to commit it. To effectively serve as basis for conviction,
conspiracy must be proved as convincingly as the criminal act. Direct proof is not absolutely
required for the purpose.

FACTS: Petitioners were collectively charged with Qualified Theft as follows: being then
employed at the Western Marketing Corporation and as such had free access to the company
premises, materials, supplies and item store, thereat, conspiring, confederating together and
mutually helping one another, feloniously take, steal and carry away two (2) booklets of Sales
Invoices and thereafter use it in preparation of fictitious sales and withdrawals of merchandise
belonging to said corporation.

In the course of preparing the January monthly sales report, Accountant noticed that the
computer printout of the monthly sales report revealed a belated entry for a certain cash sales
invoice. He then confirmed that the booklet of sales invoice was missing. He noted that the daily
cash collection report did not reflect any remittance of payments from the transactions covered
by the said invoices. In an inventory of stocks conducted at the branch office of Western, several
other appliances were found missing as were unauthorized deductions from the cash collections.

ISSUE: Whether or not conspiracy may be proved simply on the ground that all accused are co-
employees and working in one company.

HELD: No. Mere circumstance that petitioners were employees of Western does not suffice to
create the relation of confidence and intimacy that the law requires. A review of the inference
drawn from petitioners’ acts before, during and after the commission of the crime to indubitably
indicate a joint purpose, concert of action and community of interest is thus in order.

Mere companionship does not establish conspiracy. As indicated early on, there were two
different sets of imputed acts, one individual and the other collective. Rosario’s admission was
material only to her individual guilt as she referred only to the “short-over”. The wording of her
cannot be construed to extend to the other offense charging conspiracy under which no overt
act was established to prove that Rosario share with, and concurred in, the criminal design of
taking away Western’s merchandise.

63
CRIMINAL LAW I – DIGESTED CASES BY: BOAQUINA, MARY CRISCHAEL
64

PEOPLE V. COMADRE
G.R. No. 153559, June 8, 2004

DOCTRINE: The mere presence at the scene of the crime as well as their close relationship with
convict are insufficient to establish conspiracy considering that they performed no positive act in
furtherance of the crime.

FACTS: The victims Robert, Jimmy, Gerry, Rey, and Lorenzo were having a drinking spree on the
terrace of the house of Robert’s father. As the drinking session went on, Robert and the others
noticed appellants Antonio Comadre, George Comadre and Danilo Lozano walking. The three
stopped in the house. While his companions looked on, Antonio suddenly lobbed an object which
fell on the roof of the terrace. Appellants immediately fled by scaling the fence of a nearby school.
The object, which turned out to be a hand grenade, exploded. The victims were hit by shrapnel
and slumped unconscious on the floor. However, Robert Agbanlog died.

The appellants denied all the allegations. The trial court disregarded appellants’ defense of alibi
and denial.

ISSUE: Whether or not there is a conspiracy committed by herein appellants.

HELD: No. Evidence of actual cooperation rather than mere cognizance or approval of an illegal
act is required. The undisputed facts show that when Antonio Comadre was in the act of throwing
the hand grenade, George Comadre and Danilo Lozano merely looked on without uttering a single
words of encouragement or performed any act to assist him. The trial court erred in ruling that
the mere presence of George Comadre and Danilo Lozano provided encouragement and a sense
of security to Antonio Comadre constitutes conspiracy.

Neither was it proven that their act of running away with running away with Antonio was an act
of giving moral assistance to his criminal act. There being no conspiracy, only Antonio Comadre
must answer for the crime.

64
CRIMINAL LAW I – DIGESTED CASES BY: BOAQUINA, MARY CRISCHAEL
65

LI V. PEOPLE
G.R. No. 127962. April 14, 2004

DOCTRINE: It is difficult to presume conspiracy in extemporaneous outburst of violence; hence,


the demand that it may be established by positive evidence. A conviction premised on a finding
of conspiracy must be founded in facts, not on mere inferences and presumption.

FACTS: As dela Camara and Tan heard a noise outside they peered through the window, they saw
Li and Sangalang taking a bath completely naked. The two were facing the house of the Arugays.
Li and Sangalang were involved in a street brawl against Christopher Arugay and Ronaldo Tan.
There were different stories regarding the facts of the case. The SC assessed the facts of the case:
first blow was struck by Li who was armed with baseball bat which hit Arugay, whom retaliated
by inflicting a wound on Li’s head using a bolo. After Li had fallen, Sangalan, armed with a knife,
fatally stabbed Arugay. Tan picked up the baseball bat and struck Li on the head. Arugay died
while Li sustained incised wounds and a contusion. RTC found Li guilty of conspiring with
Sangalang for the murder of Arugay. The CA ruled that both are guilty of homicide, regardless of
conspiracy. Li appealed.

ISSUE: Whether or not a conspiracy exists between Li and Sangalang

HELD: No. The fact that both Li and Sangalang were in the same house at the same time is not
sufficient to establish conspiracy. Before the fight, it was not proven that Li has had asked for, or
received, any assistance from Sangalang and based on the evidence it is hard to conclude that
Sangalang and Li had acted in concert to commit the crime. Stabbing Arugay can be construed as
a spur of the moment reaction by Sangalang upon seeing his friend was struck on the head. A
conspiracy cannot arise from such spontaneous reaction.

Conspiracy transcends companionship, and mere presence at the scene of the crime does not in
itself amount to conspiracy.

65
CRIMINAL LAW I – DIGESTED CASES BY: BOAQUINA, MARY CRISCHAEL
66

GARCIA V. CA
G.R. No. 124036, October 23, 2001

DOCTRINE: Conspiracy denotes an intentional participation in a criminal transaction, with a view


to the furtherance of common design and purpose. The act of one becomes the act of all and the
particular act of an accused becomes of secondary relevance.

FACTS: Francisco Rollera was on his way to mail a letter. He was waiting at the crossing near the
police outpost, when he saw the petitioner, Wilfredo and Leopoldo, ganging up on Paulino
Rodolfo. While Leopoldo held the victim petitioner hit him with an empty bottle. Wilfredo then
stabbed the victim once with a stainless steel fan knife. The knife got struck in Paulino’s body.
Paulino succeeded in wrestling free from Leopoldo’s grasp and pulling out the knife from his
body. He used the knife to stab petitioner in his stomach. Two policemen arrived and pacified
the antagonists. A third responding policeman grabbed and caught petitioner chasing Rollera
around the parked vehicle. Paulino died.

ISSUE: Whether or not the appellate court erred in convicting petitioner as a conspirator in the
killing of Paulino Rodolfo y Olgena.

HELD: No. Petitioner Fidelino Garcia cannot be convicted as a conspirator in the killing of Paulino
Rodolfo, for the simple reason that the information against the accused contained no clear and
definite allegation of conspiracy.

In criminal prosecutions, the accused shall first be informed of the nature and cause of the
accusation against him. To ensure that the due process rights of an accused are observed, every
indictment must embody the essential elements of the crime charged with reasonable
particularity as to the name of the accused, the time and place of commission of the offense, and
the circumstances thereof.

66
CRIMINAL LAW I – DIGESTED CASES BY: BOAQUINA, MARY CRISCHAEL
67

PEOPLE V. TABUSO
G.R. No. 113708, October 26, 1999

DOCTRINE: The mere presence of a person at the scene of the crime does not make him a co-
conspirator.

FACTS: Renato Datingginoo passed by the group of Arnold Mendoza, accused Arquillos Tabuso
and some other companions in the alley to buy food. He heards Tabuso utter “nandiyan na si
Dagul”. Referred to as Dagul was the deceased Roberto Bugarin. When Renato was near the
store, he heard 2 gunshots coming from the direction of the said alley. Thereafter, he saw Arnold
Mendoza, Banong, Arquillos Tabuso and another person hurriedly coming out of the alley, and
proceeding to their house. Renato went to the place where the incident happened, near his
house, and saw Roberto Bugarin lying prostate on the ground. Accused put up the defense of
alibi.

ISSUE: Whether or not the trial court erred in holding that accused-appellant conspired with
Arnold Mendoza in the murder of Roberto Bugarin.

HELD: Yes. The people placed heavy reliance on Renato Datingginoo’s testimony that Tabuso
acted as a lookout, which conclusion must have been arrived at when Tabuso uttered “Nandiyan
na si Dagul” and from the fact that the assailants (including Tabuso) fled.

After careful examination of the evidence, the Court is not convinced that Tabuso acted as a
lookout when he uttered “Nandyan na si Dagul.”. The mere utterance did not evince
commonality in criminal intent. There is a scant scintilla of proof of Tabuso’s alleged role as a
lookout. It was never proven by the People. Obviously, that Tabuso acted as a lookout is just a
conclusion arrived at by Renato. It is barren of any factual or legal basis. Absent enough evidence
to establish conspiracy, acquittal of accused is in order since his guilt has not been established
beyond reasonable doubt.

67
CRIMINAL LAW I – DIGESTED CASES BY: BOAQUINA, MARY CRISCHAEL
68

PEOPLE V. PUGAY
G.R. L-74324, November 17, 1988

DOCTRINE: Animosity and collective acts of the conspirators should be proven to constitute
conspiracy.

FACTS: The deceased victim Miranda, a 25-year old retardate, and the accused Pugay were
friends. During a town fiesta, Gabion, the witness, was sitting in the ferris wheel and reading a
comic book. He then saw Pugay and Samson with several companions making fun of Miranda.
Pugay suddenly took a can of gasoline from under the engine of the ferris wheel and poured its
contents on the body of the Miranda. The victim died due to the incident. Gabion, Pugay, Samson
and 5 others were brought to the municipal building for interrogation. Pugay and Samson gave
statements to the police. Pugay admitted in his statement that he poured a can of gasoline on
the deceased believing that the contents thereof was water and Samson set the deceased on fire.
Samson alleged in his statement that he saw Pugay pour gasoline on Miranda but did not see the
person who set him on fire.

ISSUE: Whether or not there was a conspiracy.

HELD: No. There is nothing in the records showing that there was previous conspiracy or unity of
criminal purpose and intention between the two accused-appellants immediately before the
commission of the crime. There was no animosity between the deceased and the accused Pugay
or Samson. Their meeting at the scene of the incidental was accidental. It is also clear that the
accused Pugay and his group merely wanted to merely wanted to make dun of the deceased.
Hence, the respective criminal responsibility of Pugay and Samson arising from different acts
directed against the deceased is individual and not collective , and each of them is liable only for
the act committed by him.

68
CRIMINAL LAW I – DIGESTED CASES BY: BOAQUINA, MARY CRISCHAEL
69

ARTICLE 10: Offenses Not Subject to the Provisions of this Code

TOLEDO V. PEOPLE
439 SCRA 94

DOCTRINE: It is a matter of law that when a part adopts a particular theory and the case is tried
and decided upon that theory in the court below, he will not be permitted to change his theory
on appeal. To permit a party to change his theory on appeal will be unfair to the adverse party.

FACTS: Appellant requested the group of Ricky to refrain from making any noise. Thereupon,
appellant proceeded inside his house and went to sleep. Ricky heard stones being hurled at the
roof of the house. It was made 3 times. He saw the appellant stoning their house. Ricky went out
of the house and proceeded to appellant’s house. Ricky asked appellant, his uncle, why he was
stoning their house. Appellant did not answer but met Ricky at the doorstep of the appellant’s
house. Ricky asked appellant, his uncle, why he was stoning their house, and without any
warning, stabbed Ricky on the abdomen with a bolo.

Appellant said that the bolo accidentally hit Ricky on the stomach. He asserted in the RTC and in
the CA that he is exempt from criminal liability under Art (4) of the RPC. However, the petitioner
now alleged that he acted in self-defense when he stabbed the victim. As such, he contends, he
is not criminally liable under Art 11(1) of the RPC.

ISSUE: Whether or not the appellant may change his defense which is different from what he
adopted in the trial court and foisted in the CA.

HELD: No. The petitioner is proscribed from changing in this Court, his theory of self-defense
which he adopted in the trial court and foisted in the CA-by claiming that he stabbed and killed
the victim in complete self-defense. The petitioner relied on Art 12(4) of the RPC in the trial and
appellate courts, but adopted two divergent theories – (1) that he killed the victim to defend
himself against unlawful aggression; hence, is justified under Art 11(1) of the RPC; (2) that his
bolo accidentally hit the victim and is, thus exempt from criminal liability under Art 12(4).

It is an aberration for the petitioner to invoke two defenses at the same time because the said
defenses are intrinsically antithetical. There is no such defense in the realm of criminal law.

69
CRIMINAL LAW I – DIGESTED CASES BY: BOAQUINA, MARY CRISCHAEL
70

LADONGA V. PEOPLE
G.R. No. 141066. February 17, 2005

DOCTRINE: Offenses which are or in the future may be punishable under special laws are not
subject to the provisions of this Code. This Code shall be supplementary to such laws, unless the
latter should specially provide the contrary.

FACTS: The accused, conspiring, confederating and mutually helping with one another, knowing
fully well that they did not have sufficient funds deposited with UCPB, did then and there willfully,
unlawfully, and feloniously, draw and issue postdated check in the amount of P9,075.55, payable
to Alfredo Oculam, and thereafter, without informing the latter that they did not have sufficient
funds deposited with the bank to cover up the amount of the check, did then and there willfully,
unlawfully, and feloniously pass on, indorse, give and deliver the said check to Alfredo Oculam
by way of rediscounting of the aforementioned check; however, upon the presentation of the
check to the drawee bank for encashment, the same was dishonored for it had been already
closed, to the damage and prejudice of the said Alfredo Oculam in the aforestated amount.

ISSUE: Whether or not conspiracy is applicable in violations of BP 22 by invoking the last sentence
of Article 10 of the RPC.

HELD: Yes. BP 22 does not expressly proscribe the suppletory application of the provisions of the
RPC. Thus, in the absence of the contrary provisions in the BP 22, the general provisions of the
RPC which, by their nature, are necessarily applicable, may be applied suppletorily.

70
CRIMINAL LAW I – DIGESTED CASES BY: BOAQUINA, MARY CRISCHAEL
71

PEOPLE V. SIMON
234 SCRA 555

FACTS: Accused Martin Simon was charged with a violation of Section 4, Article II of Republic Act
No. 6425 or the Dangerous Drugs Act of 1972. He sold tea bags of marijuana to a Narcotics
Command (NARCOM) poseur-buyer. The confiscated 4 tea bags, weighing a total of 3.8 grams,
when subjected to laboratory examination, were found positive for marijuana.

Simon denied the accusation. As his defense, he claimed that on the day of question, he was
picked up by the police at their house while watching TV. He was told that he was a pusher so he
attempted to alight from the jeep but he was handcuffed instead. When they finally reached the
camp, he was ordered to sign some papers and, when he refused, he was boxed in the stomach
eight or nine times by Sgt. Pejoro. He was then compelled to affix his signature and fingerprints
on the documents presented to him. He denied knowledge of the marked money or the 4 teabags
of dried marijuana leaves, and insisted that the marked money came from the pocket of Pejoro.
Moreover, the reason why he vomited blood was because of the blows he suffered at the hands
of Pejoro.

Dr. Evelyn Gomez-Aguas, a resident physician of Romana Pangan District Hospital, declared that
she treated appellant for three days due to abdominal pain, but her examination revealed that
the cause for this ailment was appellant’s peptic ulcer. She did not see any sign of slight or serious
external injury, abrasion or contusion on his body.

Simon was sentenced to suffer the penalty of life imprisonment, to pay a fine of twenty thousand
pesos and to pay the costs. Simon then seek the reversal of the judgement

ISSUE: Whether or not the conviction of Simon correct?

HELD: No. To sustain a conviction for selling prohibited drugs, the sale must be clearly and
unmistakably established. To sell means to give, whether for money or any other material
consideration. It must, therefore, be established beyond doubt that appellant actually sold and
delivered two tea bags of marijuana dried leaves to Sgt. Lopez, who acted as the poseur-buyer,
in exchange for two twenty-peso bills.

After careful review, the Court held that there were 2 tea bags of marijuana that was sold and
there were 2 other tea bags of marijuana confiscated. Thus, Simon should be charged of selling
for the 2 tea bags of marijuana only.

However, there is an overlapping error in the provisions on the penalty of reclusion perpetua by
reason of its dual imposition, that is, as the maximum of the penalty where the marijuana is less
than 750 grams, and also as the minimum of the penalty where the marijuana involved is 750
grams or more. The same error has been committed with respect to the other prohibited and
regulated drugs provided in said Section 20. To harmonize such conflicting provisions in order to

71
CRIMINAL LAW I – DIGESTED CASES BY: BOAQUINA, MARY CRISCHAEL
72

give effect to the whole law, the court hereby hold that the penalty to be imposed where the
quantity of the drugs involved is less than the quantities stated in the first paragraph shall range
from prision correccional to reclusion temporal, and not reclusion perpetua. This is also
concordant with the fundamental rule in criminal law that all doubts should be construed in a
manner favorable to the accused.

The court held that Republic Act No. 6425, as now amended by Republic Act No. 7659, has
unqualifiedly adopted the penalties under the Revised Penal Code in their technical terms, hence
with their technical signification and effects. In fact, for purposes of determining the maximum
of said sentence, the court have applied the provisions of the amended Section 20 of said law to
arrive at prision correccional and Article 64 of the Code to impose the same in the medium
period. Such offense, although provided for in a special law, is now in effect punished by and
under the Revised Penal Code. Correlatively, to determine the minimum, the court applied first
part of the aforesaid Section 1 which directs that “in imposing a prison sentence for an offense
punished by the Revised Penal Code, or its amendments, the court shall sentence the accused to
an indeterminate sentence the maximum term of which shall be that which, in view of the
attending circumstances, could be properly imposed under the rules of said Code, and the
minimum which shall be within the range of the penalty next lower to that prescribed by the
Code for the offense.”

Thus, in the case at bar, appellant should be begrudged the benefit of a minimum sentence within
the range of arresto mayor, the penalty next lower to prision correccional which is the maximum
range have fixed through the application of Articles 61 and 71 of the Revised Penal Code. For,
with fealty to the law, the court may set the minimum sentence at 6 months of arresto mayor,
instead of 6 months and 1 day of prision correccional.

72
CRIMINAL LAW I – DIGESTED CASES BY: BOAQUINA, MARY CRISCHAEL
73

ARTICLE 11: Justifying Circumstances

MANABAN V. PEOPLE
G.R. 150 723. July 11, 2006

FACTS: Bautista is a member of the UP Police Force, took his daughter, who complained of
difficulty in breathing to the UP Health center. The doctor gave him prescriptions and so he went
to BPI Kalayaan to withdraw money. He could not withdraw money so he started kicking the
machine. The accused Manaban saw him and said that the PIN entered was incorrect that is why
the card was captured. Bautista got angry and continued kicking the machine. The former advised
to call the customer service but the latter continued kicking. Failing to pacify the victim, petitioner
fired a warning shot and then fired another hitting and killing Bautista. Manaban said that he
feared that Bautista would pull his gun first and might kill him. Trial Court found petitioner guilty
beyond reasonable doubt of the crime of Homicide. Affirmed by CA.

Issue: Whether or not the justifying circumstance is applicable

Held: No. There was no unlawful aggression on part of the victim. The allegation of Manaban that
Bautista was about to draw the gun is mere speculation. Aggression presupposes that the person
attacked must face a real threat to his life and the peril sought to be avoided is imminent and
actual, not imaginary. Absent of such actual or imminent peril to one’s life or limb. There is
nothing to repel and there is no justification for inflicting injuries.

73
CRIMINAL LAW I – DIGESTED CASES BY: BOAQUINA, MARY CRISCHAEL
74

PEOPLE V. ALCONGA
78 Phil 366

FACTS: Barion, the banker of the card game, was playing black jack against De Raposo. De Raposo
and Alconga were partners in the game. Alconga seated behind Barrion and gave signs to De
Raposo. Barion, who suffered losses, found out and expressed anger.

They met again when Alconga was on his job at home guard. Barion suddenly appeared and
swinged his bamboo stick. The former avoided the blow. While Barion was about to fire the 3rd
blow, Alconga fired at him with his revolver. The deceased stood up, drew his dagger and ran
away about 200m. Alconga ran after him and gave a mortal bolo blow.

Issue: Whether or not self-defense can be used as a defense by Alconga

Held: No. Alconga guilty of Homicide. It is apparent that it is Alconga who is the superior fighter
and his safety was already secured after the first fight . There was no more reason for him to
further chase Barion. In the first fight, self-defense would have been valid. Provocation must be
sufficient and immediately preceding the act.

74
CRIMINAL LAW I – DIGESTED CASES BY: BOAQUINA, MARY CRISCHAEL
75

PEOPLE V. GENOSA (Battered Woman Syndrome)


419 SCRA 537

DOCTRINE: To use battered woman syndrome, one must prove that: First, each of the phases of
the cycle of violence must be proven to have characterized at least two battering episodes
between the appellant and her intimate partner. Second, the final acute episode preceding the
killing of the batterer must have produced in the battered person’s mind an actual fear of an
imminent harm from the batterer and an honest belief that she needed to use force in order to
save her life. Third, at the time of the killing, the batterer must have posed probable-not
necessarily immediate and actual-grave harm to the accused, based on the history of violence
perpetrated by the former against the latter.

FACTS: Marivic Genosa, the appellant, on November 15, 1995, attacked and wounded his
husband which ultimately led to his death. According to the appellant, she did not provoke her
husband when she got home that night and it was her husband who began the provocation. The
appellant said she was frightened that her husband would hurt her and she wanted to make sure
she would deliver her baby safely.

The appellant testified that during her marriage she had tried to leave her husband at least five
times, but that Ben would always follow her and they would reconcile. The appellant said that
the reason why Ben was violent and abusive towards her that night was because he was crazy
about his recent girlfriend, Lulu Rubillos. The appellant, after being interviewed by specialist, has
been shown to be suffering from Battered Woman Syndrome. The appellant with a plea of self-
defense admitted the killing of her husband. She was found guilty of the crime of parricide, with
the aggravating circumstance of treachery, for the husband was attacked while asleep.

ISSUES: Whether or not appellant acted in self-defense.

HELD: The SC held that the defense failed to establish all the elements of self-defense arising
from battered woman syndrome, to wit: (a) Each of the phases of the cycle of violence must be
proven to have characterized at least two battering episodes between the appellant and her
intimated partner; (b) The final acute battering episode preceding the killing of the batterer must
have produced in the battered person’s mind an actual fear of an imminent harm from her
batterer and an honest belief that she needed to use force in order to save her life, and; (c) At
the time of the killing, the batterer must have posed probable – not necessarily immediate and
actual – grave harm to the accused based on the history of violence perpetuated by the former
against the latter.

75
CRIMINAL LAW I – DIGESTED CASES BY: BOAQUINA, MARY CRISCHAEL
76

SENOJA V. PEOPLE
G.R. No. 160341, October 19, 2004

FACTS: Petitioner Exequiel Senoja, Fidel Senoja, Jose Calica, and Miguel Lumasac were drinking
gin in the hut of Crisanto Reguyal. Leon Lumasac, who was furious, suddenly arrived at the said
place, holding a bolo in his right hand and looking for his brother Miguel. Leon hacked the wall
of the house in anger so petitioner and Jose tried to pacify him but when petitioner approached
Leon, the latter tried to hack him so he embraced Leon while Jose took Leon's bolo and threw it
away. Then, Leon and petitioner talked things out and later reconciled. Senoja took the
petitioner's knife because Leon was already pacified and he later gave it back to the petitioner.

Subsequently, Leon demanded that Calica return his bolo as he wanted to go home already.
Because he had thrown away the victim's bolo, Calica was, thus, impelled to give his own. The
victim then warned the petitioner three times, "May mangyayari sa iyo, kung hindi ngayon,
bukas," and left the hut. Leon walked out of Crisanto's hut followed by petitioner. When Leon
had already gone about ten meters from the hut, the petitioner followed him.

According to the petitioner, Leon turned around and told him, "Kung hindi lang kita inaanak."
Leon then hacked the petitioner, hitting the latter on the left side of his head and thigh. Believing
that Leon would attack him anew, the petitioner stabbed him frontally several times. Then,
petitioner ran towards the barangay road and threw away the "kolonial" knife he used in
stabbing Leon.
The petitioner was charged with homicide so he appealed the decision to the Court of Appeals
invoking the defense of self-defense.

ISSUE: Whether or not the petitioner could invoke self-defense.

HELD: No, the testimony of the petitioner is belied by the physical evidence on record, which
has proven that Leon must’ve been stabbed as when he was walking home as evinced by the left
buttock stab he sustained. Then, Leon received the front stab wounds when he faced his assailant
while all the injuries that the petitioner received were a result of the victim’s attempt to defend
himself. The settled rule is that physical evidence is evidence of the highest order; it speaks more
eloquently than a hundred witnesses. The petitioner even failed to inform the doctor that he
sustained the wounds to defend himself.

The petitioner also threw away his knife and failed to surrender it to the policemen; neither did
he inform the policemen that he killed the victim in self-defence. The petitioner's claim that the
victim was armed with a bolo is hard to believe because he even failed to surrender the bolo.

Lastly, the petitioner's version of the events that transpired immediately before he stabbed the
victim does not inspire belief. He claims that when he saw the victim emerge from the hut, the
victim walked towards the petitioner saying, "Kung hindi lang kita inaanak," but hit and hacked

76
CRIMINAL LAW I – DIGESTED CASES BY: BOAQUINA, MARY CRISCHAEL
77

the latter on the left buttock. As gleaned from his statement, the victim was not disposed, much
less determined to assault the petitioner. And yet, the petitioner insists that without much ado,
the victim, nevertheless, hit him on the head and on the thigh with his bolo.

It is the well-considered finding of this Court that while Leon Lumasac had ceased being the
aggressor after he left the hut to go home, accused Exequiel Senoja was now the unlawful
aggressor in this second phase of their confrontation. It bears mentioning that appellant
contradicted himself with respect for the reason why he left the hut. First, it was to pacify Leon
and the second reason was that he was going home.

As for appellant's injuries, it is clear that they were sustained in the course of the victim's attempt
to defend himself as shown by the lacerated wound on the victim's left palm, a defensive wound.

The affirmative defense of self-defense may be complete or incomplete. It is complete when all
the three essential requisites are present; it is incomplete if only unlawful aggression on the part
of the victim and any of the two essential requisites were present. In fine, unlawful aggression
on the part of the victim is a conditionsine qua non to self-defense, complete or incomplete.
Whether or not the accused acted in self-defense is a question of fact. Like alibi, the affirmative
defense of self-defense is inherently weak because, as experience has demonstrated, it is easy to
fabricate and difficult to disprove.

The right of self-defense proceeds from necessity and limited by it. The right begins where
necessity does, and ends where it ends. There is, however, a perceptible difference between
necessity and self-defense, which is that, self-defense excuses the repulse of a wrong; necessity
justifies the invasion of a right. Hence, it is essential to self-defense that it should be a defense
against a present unlawful attack.

Unlawful aggression presupposes an actual, sudden, unexpected attack or imminent danger


thereof, not merely a threatening or intimidating attitude. Hence, when an inceptual/unlawful
aggression ceases to exist, the one making a defense has no right to kill or injure the former
aggressor. After the danger has passed, one is not justified in following up his adversary to take
his life. The conflict for blood should be avoided if possible. An assault on his person, he cannot
punish when the danger or peril is over. When the danger is over, the right of self-defense ceases.
His right is defense, not retribution.

77
CRIMINAL LAW I – DIGESTED CASES BY: BOAQUINA, MARY CRISCHAEL
78

RAZON V. PEOPLE
GR 158053. June 21, 2007

FACTS: 3 men boarded the taxi cab or Razon from the bus station. Upon reaching the destination,
Gonzalo, who was seated behind the driver’s seat, declared a hold-up and poked a Batangas knife
at the right side of Razon’s neck. However, Razon was able to grab the knife. Gonzalo’s
companions went out and picked up stones. Gonzalo followed and Razon ran after them. Gonzalo
was swinging his can and it hit Razon on his leg. Razon though of his knife inside the cab and he
went to get it and confronted the three by swinging his knife.

ISSUE: Whether or not the accused can claim that he acted in self-defense.

HELD: No. It is settled that the moment the first aggressor run away, unlawful aggression on the
part of the first aggressor ceases to exists; and when unlawful aggression ceases, the defender
no longer has any right to kill or wound the former aggressor; otherwise, retaliation and not self-
defense is committed. Retaliation is not the same as self-defense. In retaliation, the aggression
that was begun by the injured party already ceased when the accused attacked him, while in self-
defense the aggression was still existing when the aggressor was injured by the accused.

78
CRIMINAL LAW I – DIGESTED CASES BY: BOAQUINA, MARY CRISCHAEL
79

ESLABON V. PEOPLE
G.R. No. L-66202. February 24, 1984

FACTS: Noli Eslabon, who is the first cousin of Francisco, came to Francisco’s rescue and stabbed
Harder at the left arm which was holding the scythe; that as the stab was made sideways,
Eslabon’s knife penetrated the whole arm of Harder and reached his chest; that despite the stab
wound, Harder did not release his hold on Francisco so Eslabon struck Harder with his hand.
Francisco and Harder were taken to the hospital for treatment but Harder died.

ISSUE: Whether or not Eslabon used the reasonable necessity of the means employed to repel
the unlawful aggression of the deceased against the petitioner’s first cousin.

HELD: No. Eslabon failed to discharge the burden of proving the element of reasonable necessity.
The stab wounds inflicted by petitioner on the deceased were all directed at the left forearm of
the deceased, sustaining petitioner's statement that he did not intend to seriously injure Harder
but merely wanted to release the latter's hold on Francisco because the scythe was stuck under
the latter's armpit. In view of the fact that Francisco and Harder were grappling, it is entirely
credible that the same knife thrust at Harder's left arm caused the wound on Harder's left chest
as testified by the attending doctor, which wound unfortunately proved fatal. Under the
emergency situation confronting the petitioner, who feared for the life of Francisco, it would
have been rash and unnatural on his part, as rationalized by respondent court, if he were yet to
look for a police officer instead of rushing to the defense of Francisco who was under serious
attack and in grave danger.

79
CRIMINAL LAW I – DIGESTED CASES BY: BOAQUINA, MARY CRISCHAEL
80

PEOPLE V. LUAGUE
62 Phil 344

FACTS: Paulino Disuadido came and began to make love to Natividad Luague; she cannot
dissuade him from his purpose, Paulino drew and opened a knife and threatened her with death,
began to embrace her and touch her breasts; that in preparing to lie with her, Paulino had to
leave the knife to the floor and the accused picked up the weapon and stabbed him. The accused
immediately surrender herself to the authorities and report the incident.

ISSUE: Whether or not the act committed by Natividad is a reasonable necessity to defend
herself.

HELD: Yes. The accused acted in legitimate self-defense. It is evident that a woman who,
imperiled, wounds, nay kills the offender should be afforded exemption from criminal liability
provided by Article 11 (1) of the RPC since such killing cannot be considered a crime from the
moment it became the only means left for her to protect her honor from so great an outrage.

80
CRIMINAL LAW I – DIGESTED CASES BY: BOAQUINA, MARY CRISCHAEL
81

PEOPLE VS. DE LA CRUZ


61 Phil 344

DOCTRINE: A person is not criminally responsible when, by reason of a mistake of facts, he does
an act for which he would be exempt if the facts were as he supposed them to be, but would
constitute murder if he had known the true state of facts at the time, provided that the ignorance
or mistake of fact was not due to negligence or bad faith

FACTS: One evening, the defendant Remedios de la Cruz, with Francisco Ramos and his wife,
Vrigida Vistada; his sister Baltazara Ramos; and a woman named Consuelo or Natividad Santoyo
went to a wake in honor of one Sion. At about 9 pm, the defendant and her friends went home.
They were followed about 5 minutes later by the deceased Francisco Rivera who was
accompanied by Enrique Bautista. Rivera and Bautista overtook defendant‘s party. When they
reached a narrow part of the path, Rivera went ahead of Bautista. At that time, the members of
the defendant‘s party were walking in single pile. Baltazara Ramos was in the lead and the
defendant was the hindmost. The defendant was about 2 brazas from the person immediately
ahead of her.

Defendant‘s testimony: a man suddenly threw his arms behind, caught hold of her breasts and
kissed her, and seized her in her private parts; that she tried to free herself, but he held her and
tried to throw her down; she got a knife from her pocket (she was engaged in selling fruits),
opened it and stabbed him in defense of her honor. It should be noted that the deceased had
been making love to the defendant and also to another girl.

Issue: Whether or not De la Cruz‘ killing of Rivera may be justified by defense of honor.

Held: Yes. She was justified in making use of the pocket-knife in repelling what she believed to
be an attack upon her honor since she had no other means of defending herself.

81
CRIMINAL LAW I – DIGESTED CASES BY: BOAQUINA, MARY CRISCHAEL
82

PEOPLE V. JAURIGE
76 Phil 174

FACTS: Defendant and the appellant Capino lived in the same barrio. Her family went to the local
church where it was quite bright. When Jaurigue was left alone, Capino sat beside her and place
his hand on top of her thigh. Jaurige suddenly stabbed Capino in the neck, fatally causing a single
wound from which he died.

ISSUE: Whether or not defendant acted in self-defense.

HELD: No. She is not absolved form criminal liability. When the deceased sat beside her on the
same bench in a well-lit chapel with several people there is no possibility of being raped. She
cannot be legally declared completely exempt from criminal liability for fatally wounding the
deceased since the means employed by her was evidently excessive.

82
CRIMINAL LAW I – DIGESTED CASES BY: BOAQUINA, MARY CRISCHAEL
83

PEOPLE V. ORIENTE
GR 155094. Januarty 30, 2007

FACTS: Manuel Oriente’s appeal of his conviction for the crime of homicide. The appellant was
with other persons, attacked and assaulted Romulo Vallo, hitting him with a lead pipe on
different parts of the body, thereby inflicting him serious and mortal wounds which were the
immediate cause of his death.

ISSUE: Whether or not the lower courts erred in not appreciating that there was unlawful
aggression on the part of the victim and the means employed by the appellant to prevent the
same was reasonable necessity.

HELD: No. Since self-defense was invoked, the burden of evidence shifts to the accused. The fact
that the deceased was not able to make use of his gun after being hit in the forehead by the
weapon of the appellant as alleged by the defense makes their claim of self-defense unusual. The
appellant failed to establish the existence of gun, that was alleged to have constitute the unlawful
aggression.

83
CRIMINAL LAW I – DIGESTED CASES BY: BOAQUINA, MARY CRISCHAEL
84

US V. BUMANGLAG
41 Phil 644

FACTS: Rafael Bumanglag missed 4 baares or 40 bundles of palay which were kept in his granary.
He found it in an inclosed field which planted with sugar cane. For the purpose of ascertaining
who had done it, he left the palay and that night, accompanied by his friends waited near the
said field for the person who migh return to get the palay. Guillermo Ribis made his appearance
and approaching the palay, and attempted to carry it with him but Bumanglag and his friends
assaulted the presumed thief which caused the victim’s death.

ISSUE: Whether or not the act committed may be justified as defense of property.

HELD: In the commission of the crime we should take into account the mitigating circumstance
No. 7 of article 9 of the Penal Code, because the defendant acted with loss of reason and self-
control on seeing that Guillermo Ribis was taking material possession of the palay seized and
hidden by him on the previous night, thus committing one of the numerous unlawful acts
perpetrated at that place, to the damage and prejudice of those who, by their labor endeavor to
provide themselves with the necessary elements for their subsistence and that of their families.
The special circumstance established by article 11 of the same code should be also considered in
favor of the accused, in view of the erroneous and quite general belief that it is legal to punish,
even to excess the thief who, in defiance of law and justice, while refusing to work, devotes
himself to depriving his neighbors of the fruits of their arduous labors.

A person may repel force by force in defense of his habitation or property, as well as in defense
of his person, against one who manifestly intends and endeavors by violence or surprise to
commit a known felony upon either, if need be, may kill his adversary

84
CRIMINAL LAW I – DIGESTED CASES BY: BOAQUINA, MARY CRISCHAEL
85

PEOPLE V. NARVAEZ
121 SCRA 339

FACTS: Graciano Juan, Jesus Verano and Cesar Ibañez, together with the two deceased Davis
Fleischer and Flaviano Rubia, were fencing the land of George Fleischer, father of deceased Davis
Fleischer. The place was in the boundary of the highway and the hacienda owned by George
Fleischer. This is located in the municipality of Maitum, South Cotabato. At the place of the
fencing is the house and rice drier of appellant Mamerto Narvaez. At that time, appellant was
taking his rest, but when he heard that the walls of his house were being chiseled, he arose and
there he saw the fencing going on. If the fencing would go on, appellant would be prevented
from getting into his house and the bodega of his rice mill. So he addressed the group, saying -
'Pare, if possible, stop destroying my house and if possible we will talk it over - what is good,'
addressing the deceased Rubia, who is appellant's compadre. The deceased Fleischer, however,
answered: 'No, god damn it, proceed, go ahead.' Appellant apparently lost his equilibrium and
he got his gun and shot Fleischer, hitting him. As Fleischer fell down, Rubia ran towards the jeep,
and knowing there is a gun on the jeep, appellant fired at Rubia, likewise hitting him. Both
Fleischer and Rubia died as a result of the shooting.
It appears, however, that this incident is intertwined with the long drawn out legal battle
between the Fleischer and Co., Inc. of which deceased Fleischer was the secretary-treasurer and
deceased Rubia the assistant manager, on the one hand, and the land settlers of Cotabato,
among whom was appellant.

During the pendency of the case, appellant on February 21, 1967 entered into a contract of lease
with the company whereby he agreed to lease an area. Narvaez signed the contract although the
ownership of the land was still uncertain, in order to avoid trouble, until the question of
ownership could be decided. He never paid the agreed rental, although he alleges that the milling
job they did for Rubia was considered payment. On June 25, 1968, deceased Fleischer wrote him
a letter stating the termination of their contract because of Narvaez’s failure to pay rent and
would consequently start fencing as well as demolishing Narvaez’s property.
The act of killing of the two deceased by appellant is not disputed. Appellant admitted having
shot them from the window of his house with the shotgun which he surrendered to the police
authorities when he willingly gave himself up. He claims, however, that he did so in defense of
his person and of his rights, and therefore he should be exempt from criminal liability.

ISSUE: Whether or not Narvaez could invoke self-defense.

HELD: The aforementioned verbal exchange took place while the two deceased were on the
ground doing the fencing and the appellant was up in his house looking out of his window.
According to appellant, Fleischer's remarks caused this reaction in him: "As if, I lost my senses
and unknowingly I took the gun on the bed and unknowingly also I shot Mr. Fleischer, without
realizing it, I shot Mr. Fleischer." While Rubia was shot because he was clearly going after the gun
on the jeep. Therefore, obfuscation was clearly present in the situation.

85
CRIMINAL LAW I – DIGESTED CASES BY: BOAQUINA, MARY CRISCHAEL
86

In any case, after the termination of the contract, Fleischer had given Narvaez up to December
31, 1968 within which to vacate the land. He should have allowed appellant the peaceful
enjoyment of his properties up to that time, instead of chiseling the walls of his house and closing
appellant's entrance and exit to the highway.

Conformably to Articles 536 and 539 of the Civil Code, the deceased had no right to destroy or
cause damage to appellant's house, nor to close his accessibility to the highway while he was
pleading with them to stop and talk things over with him. The assault on appellant's property,
therefore, amounts to unlawful aggression as contemplated by law.

In the case at bar, there was an actual physical invasion of appellant's property which he had the
right to resist, pursuant to Art. 429 of the Civil Code and defense of one's person or rights is
treated as a justifying circumstance under Art. 11, par. 1 of the Revised Penal Code.

However, there was no provocation at all on Narvaez’s part, since he was asleep at first and was
only awakened by the noise produced by the victims and their laborers. His plea for the deceased
and their men to stop and talk things over with him was no provocation at all.
Be that as it may, appellant's act in killing the deceased was not justifiable, since not all the
elements for justification are present. He should therefore be held responsible for the death of
his victims, but he could be credited with the special mitigating circumstance of incomplete
defense, pursuant to paragraph 6, Article 13 of the Revised Penal Code.
The court ruled that Narvaez was guilty of two counts of homicide, mitigated by the privileged
extenuating circumstance of incomplete self-defense as well as by two generic mitigating
circumstances of voluntary surrender and obfuscation.

Note:
First, to have incomplete self-defense, the offended party must be guilty of unlawful aggression.
Without this, there can be no incomplete self-defense, defense of relative, or defense of stranger.
Second, if only the element of unlawful aggression is present, the other requisites being absent,
the offender shall be given only the benefit of an ordinary mitigating circumstance.
Third, if aside from the element of unlawful aggression another requisite, but not all, are present,
the offender shall be given the benefit of a privileged mitigating circumstance. In such a case, the
imposable penalty shall be reduced by one or two degrees depending upon how the court regards
the importance of the requisites present.

86
CRIMINAL LAW I – DIGESTED CASES BY: BOAQUINA, MARY CRISCHAEL
87

PEOPLE VS. PELAYO


64 OG 1991

FACTS: Pelayo told Atty. Clapano in his office and within hearing distance of three other people
that upon his investigation about the existence of gambling in the community, a Chinese operator
named Lim Peng told him that then Gov. Alejandro Almendras (now Senator) used to receive
from him P500 protection money. The following day, Pelayo delivered a privileged speech in city
council session wherein he did not directly mention but insinuated through his interpellations
that the governor was receiving ―tongs. Pelayo admits having the said conversation with
Clapano.

Issue: WON words were uttered in Self Defense to what the governor had said about him
previously

Held: NO. Even if on a previous occasion the governor made derogatory remarks against Pelayo,
the retaliation with scurrilous words cannot be self-defense. It will only exist of the defendant
did not go beyond explaining what was previously said of him for the purpose of repairing the
effect of the damage caused to him. There is no justification for him to hit back of make the same
imputation of accusation because this is not an act of defense but an aggression itself.

87
CRIMINAL LAW I – DIGESTED CASES BY: BOAQUINA, MARY CRISCHAEL
88

TAN V. STANDARD VACUUM OIL CO.,


91 Phil. 672

FACTS: The driver of the truck abandoned the truck which continued moving to the opposite side
of the street causing the buildings on the side to be burned and destroyed. The house of Anita
Tan was among those destroyed. The driver cannot be held responsible since he took all the
necessary precautions against the contingency as he was confronted with, so the RTC acquitted
them.

ISSUE: Whether or not the driver can be held liable.

HELD: No. This company is sued not precisely because of supposed negligent acts of its two
employees Julito Sto. Domingo and Igmidio Rico but because of acts of its own which might have
contributed to the fire that destroyed the house of the plaintiff. The complaint contains definite
allegations of negligent acts properly attributable to the company which if proven and not
refuted may serve as basis of its civil liability. Thus, in paragraph 5 of the first cause of action, it
is expressly alleged that this company, through its employees, failed to take the necessary
precautions or measures to insure safety and avoid harm to persons and damage to property as
well as to observe that degree of care, precaution and vigilance which the circumstances justly
demanded, thereby causing the gasoline they were unloading to catch fire. The precautions or
measures which this company has allegedly failed to take to prevent fire are not clearly stated,
but they are matters of evidence which need not now be determined. Suffice it to say that such
allegation furnishes enough basis for a cause of action against this company. There is no need for
the plaintiff to make a reservation of her right to file a separate civil action, for as this court
already held in a number of cases, such reservation is not necessary when the civil action
contemplated is not derived from the criminal liability but one based on culpa aquiliana under
the old Civil Code (articles 1902 to 1910). These two acts are separate and distinct and should
not be confused one with the other. Plaintiff can chose either.

88
CRIMINAL LAW I – DIGESTED CASES BY: BOAQUINA, MARY CRISCHAEL
89

PEOPLE V. OANIS
74 Phil 257

FACTS: The police were instructed to arrest Balagtas and to follow the instruction in the telegram
(dead or alive). The group of Oanis and Galanta went to the house where Irene lived. They went
to the room of Irene and seeing a man sleeping with his back toward the door where they are,
simultaneously or successively fired at him. Later, it was found that the person shot was not the
notorious criminal Balagtas but Tecson, Irene’s paramour.

ISSUE: Whether or not Oanis and Galanta may be held responsible for the death of Tecson.

HELD: Yes. Although an officer in making a lawful arrest is justifies in using such force as is
reasonably necessary to secure and detain the offender, he is never justified in using unnecessary
force or in resorting to dangerous means when the arrest could be effected otherwise. It may be
true that Balagtas was a notorious criminal, a life-termer, but these facts alone constitute no
justification for killing him when in effecting his arrest, he offers no resistance or in fact no
resistance can be offered, as when he is asleep.

89
CRIMINAL LAW I – DIGESTED CASES BY: BOAQUINA, MARY CRISCHAEL
90

MAMANGUN V. PEOPLE
G.R. No. 149152. February 2, 2007

FACTS:Rufino Mamangun y Silverio was a public officer, Police Officer (PO2), armed with a gun,
with intent to kill, assault and shoot Gener Contreras, thereby inflicting him serious physical
injuries which cause his death.

One evening in Brgy. Calvario, Meycauayan, Bulacan, someone shouted that there was a robber
and several residents chased the suspect who proceeded to the rooftop of Abacan’s house. The
petitioner and other policemen went searched the rooftop whereat the suspect was allegedly
taking refuge. They saw a man whom they thought was the robbery suspect. Mamangun, who
was walking ahead of the group, hit a man who turned out to be Contreras who was not the
robbery suspect.

The policemen corroborating one another testified that they think that Contreras was the suspect
they were looking for. They announced that they were police but the person continued to run
until Mamangun caught up with him and the person suddenly stopped, raised a stainless steel
pipe towards the latter’s head but Mamangun was able to evade the attack.

ISSUE: Whether or not the petitioner Mamangun can claim self-defense.

HELD: No. The Court rejected the petitioner’s claim that the shooting was justified by self-
defense, it ruled that the crime of Homicide was attended by an incomplete justifying
circumstance of the petitioner having acted in the performance of his duty as a policeman.

Petitioner could have first fired a warning shot before pulling the trigger against Contrerqas who
was one of the residents chasing the suspected robber.

The justifying circumstance of fulfillment of duty under par. 5, Article 11 of the RPC may be
invoked only after the defense proves that (1) the accused acted in the performance of a duty;
(2) the injury inflicted or offense committed is the necessary consequence of the due
performance or lawful exercise of such duty.

Self-defense, whether complete or incomplete, cannot be appreciated as a valid justifying


circumstance in this case. The most important element of unlawful aggression on the part of the
victim to justify a claim of self-defense was absent. Lacking this essential and primary element of
unlawful aggression, petitioner’s plea of self-defense, complete or incomplete, must have to fail.
Mamangun found guilty beyond reasonable doubt of the crime of Homicide.

90
CRIMINAL LAW I – DIGESTED CASES BY: BOAQUINA, MARY CRISCHAEL
91

PEOPLE V. BERONILLA
96 Phil. 566

FACTS: The accused was a military major of La Paz, Abra. He received an order form the regional
commander of an infantry of the Philippine Army, operating as a guerrilla unit, to prosecute
Arsenio Borjal for treason and to appoint a jury of 12 bolomen. The jury found Borjal guilty of the
charge and the recommendation of the jury was approved by the Headquarters of the guerrilla
unit. For the execution of Borjal, the accused was prosecuted for murder. The accused acted
upon orders of superior officers which turned out to be illegal.

ISSUE: Whether or not Beronilla can be considered to be acting upon a call of duty and thus,
covered by justifying circumstances which would warrant acquittal

HELD: Yes. Criminal intent not established; he was acting pursuant to orders of his superiors in
the City (orders given through letters/telegrams). As a military subordinate, he could not
question the orders of his superior officers. He obeyed the orders in good faith without being
aware of their illegality, without any fault or negligence on his part.

91
CRIMINAL LAW I – DIGESTED CASES BY: BOAQUINA, MARY CRISCHAEL
92

PEOPLE V. BARROGA
54 Phil 247

FACTS: Barroga freely admits that he prepared the falsified documents with full knowledge of
their falsity; but he alleges that he did so from data furnished by his immediate chief, the now
deceased Baldomero Fernandez, and only in obedience to instructions from him.

ISSUE: Whether or not Barroga may be justified in committing the crime since it is from an order.

HELD: No. With respect to the alleged instructions given by said Baldomero Fernandez, even
supposing that he did in fact give them, and that the defendant committed the crime charged by
virtue thereof, inasmuch as such instructions were not lawful, they do not legally shield the
appellant, nor relieve him from criminal liability. In order to exempt from guilt, obedience must
be due, or as Viada lucidly states, it must be a compliance with "a lawful order not opposed to a
higher positive duty of a subaltern, and that the person commanding, act within the scope of his
authority. As a general rule, an inferior should obey his superior but, as an illustrious
commentator has said, "between a general law which enjoins obedience to a superior giving just
orders, etc., and a prohibitive law which plaintiff forbids what that superior commands, the
choice is not doubtful."

92
CRIMINAL LAW I – DIGESTED CASES BY: BOAQUINA, MARY CRISCHAEL
93

ARTICLE 12: Exempting Circumstances

PEOPLE V. AMBAL
100 SCRA 325

DOCTRINE: In order that insanity may be taken as an exempting circumstance, there must be
complete deprivation of intelligence in the commission of the act or that the accused acted
without the least discernment. Mere abnormality of his mental faculties does not exclude
imputability.

FACTS: Honorato Ambal went to the house of the barangay captain and made an oral confession
that he had killed his wife, Felicula. After that, he went to the municipal hall and surrendered
himself to the policeman.

The immediate provocation was a quarrel induced by Felicula’s failure to buy medicine for Ambal
who was afflicted with influenza. They two engaged in a heated alteration. Felicula told her
husband that it would be better if he were dead. That remark infuriated Ambal and impelled him
to attack his wife.

ISSUE: Whether or not Ambal can use insanity as his defense.

HELD: NO. The doctor concluded that Ambal was not insane. He was normal but nervous, no
mental disorder. The law presumes that every person is of sound mind, in the absence of proof
to the contrary. The defendant who interposes the defense of mental capacity, has the burden
of establishing that he was insane at the very moment when the crime was committed.

In the instant case, the alleged insanity was not substantiated by any sufficient evidence. The
presumption of sanity was not overthrown. He was not completely bereft of reason or
discernment and freedom of will when he mortally wounded his wife. He was not suffering from
mental disease. The fact that after the incident he thought of surrendering to the law enforcing
authorities is a proof that he knew that what had done was wrong.

Ambal is GUILTY of Parricide.

93
CRIMINAL LAW I – DIGESTED CASES BY: BOAQUINA, MARY CRISCHAEL
94

PEOPLE V. PUNO
105 SCRA 151

DOCTRINE: Persons who recover from an acute episode of mental illness like schizophrenia may
retain some residual symptoms impairing their judgment but necessarily their discernment of
right from wrong of the offense committed.

FACTS: Puno entered the room of Aling Kikay and insulted her by saying: “Mangkukulam ka
mambabarang mayroon kang bubuyog.” Then, he repeatedly slapped her and struck her several
times on the head with hammer until she was dead. Witnesses testified that Puno’s eyes were
reddish. His look was baleful and menacing. The witnesses were threatened by Puno not to go to
the police. Disregarding Puno’s threat, Lina notified the police of the killing.

Puno’s father surrendered him to the police. Puno, on his testimony, pretended that he did not
remember having killed Aling Kikay. The defense presented three psychiatrist. However, instead
of proving that Puno was insane when he killed Aling Kikay, the medical experts testified that
Puno acted with discernment.

ISSUE: Whether or not Puno is exempted from criminal liability due to insanity.

HELD: NO. Puno was not legally insane when he killed the hapless and helpless victim. The facts
and the findings of the psychiatrists reveal that in that tragic occasion he was not completely
deprived of reason and freedom of will.

Insanity exists when there is a complete deprivation of intelligence in committing the act, that is,
the accused is deprived of reason, he acts without the least discernment because there is
complete absence of the power to discern, or that there is total deprivation of freedom of the
will. Mere abnormality of the mental faculties will not exclude imputability.

It results that the medium period of the penalty for murder should be imposed. The death penalty
is set aside. The accused is accused to reclusion perpetua.

94
CRIMINAL LAW I – DIGESTED CASES BY: BOAQUINA, MARY CRISCHAEL
95

PEOPLE V. DUNGO
199 SCRA 860

DOCTRINE: One who suffers from insanity at the time of the commission of the offense charged
cannot in a legal sense entertain a criminal intent and cannot be held criminally responsible for
his acts.

FACTS: Rosalino Dungo stabbed Mrs. Sigua, with a knife from the envelope he was carrying, inside
the field office of the Department of Agrarian Reform. Mrs. Sigua died and an information for
murder was filed against Dungo. The accused raised the defense of insanity. During the trial, the
prosecution presented the victim’s husband, Atty. Sigua, to testify that the accused visited their
house to confront him on why his wife was making it difficult for the accused to transfer the
landholding his father to him. The trial court convicted him because the act of concealing a fatal
weapon and the act of taking flight in order to evade arrest indicates that accused was sane
during the time he committed the stabbing.

The case went up to the Supreme Court for automatic review.

ISSUE: Whether it is permissible to receive evidence of the accused’s mental condition for a
reasonable period both before and after the time of the act in question.

HELD: YES. The Court held that “Evidence of insanity must have reference to the mental condition
of the person whose sanity is in issue, at the very time of doing the act which is the subject of
inquiry. However, it is permissible to receive evidence of his mental condition for a reasonable
period both before and after the time of the act in question. Direct testimony is not required nor
the specific acts of derangement essential to establish insanity as a defense.”

95
CRIMINAL LAW I – DIGESTED CASES BY: BOAQUINA, MARY CRISCHAEL
96

PEOPLE V. TANEO
58 Phil. 255

FACTS: Potenciano Taneo while sleeping, got up, left his room with a bolo and while meeting his
wife wounded her in the abdomen. Other people were also attacked such as his father and his
guests. He claims that he was sleeping as the crime was committed.

ISSUE: Whether or not Taneo can be exempt from criminal liability

HELD: Yes. The defendant acted while in a dream & his acts, therefore, weren‘t voluntary in the
sense of entailing
criminal liability. The apparent lack of motive for committing a criminal act does not necessarily
mean that there are none, but that simply they are not known to us. Although an extreme moral
perversion may lead a man to commit a crime without a real motive but just for the sake of
committing it. In the case at hand, the court found not only lack of motives for the defendant to
voluntarily commit the acts complained of (read: he loved his wife dearly, he tried to attack his
father in whose house the lived and the guests whom he invited), but also motives for not
committing the acts. Dr. Serafica, an expert witness in the case, stated that considering the
circumstances of the case, the defendant acted while in a dream, under the influence of a
hallucination and not in his right mind. The wife's wound may have been inflicted accidentally.
The defendant did not dream that he was assaulting his wife, but that he was defending himself
from his enemies.

Judgment: defendant not criminally liable for the offense. It was also ordered that he be confined
in the government insane asylum and will not be released until the director thereof finds that his
liberty would no longer constitute a menace.

96
CRIMINAL LAW I – DIGESTED CASES BY: BOAQUINA, MARY CRISCHAEL
97

PEOPLE V. ESTEPANO
307 SCRA 701

FACTS: Balinas stabbed and hacked to death by a bolo and cane butter by Dominador, Rodrigo,
Rube, Rodney, Dante and Rene all with the last name Estepano who were charged with Murder.
Rodrigo died during the trial, Dante was never apprehended and Dominador was acquitted while
the other three Rube, Rodney, and Rene were found guilty. Rene at that time the crime was
committed was 13 years of age

ISSUE: Whether or not minority can be used as a defense by Rene

HELD: Rene was only 13 at the that the crime was committed. He was over 9 and under 15, which
means he could be exempted from criminal liability when it is proven that he did not act with
discernment. The prosecution failed to prove that he acted with discernment, therefore, he is
exempted.

97
CRIMINAL LAW I – DIGESTED CASES BY: BOAQUINA, MARY CRISCHAEL
98

LLAVE V. PEOPLE
G.R. 166040. April 26, 2006

FACTS:Neil Llave is a minor of 12 years of age had carnal knowledge with his victim who was 7
years old where he dragged the victim behind a group of hollow blocks to ensure that people
would not discover the rape. While committing the crime, he was discovered and subsequently
fled, but eventually caught.

ISSUE: Whether or not Llave is exempted of crime of rape

HELD: No. Accused is a minor over the age of 9 and under 15, he could be exempted if he acted
without discernment. In this case, it was seen that the act of hiding his victim in a secluded are
shows that he had discernment and had an idea that what he was doing was wrong.

98
CRIMINAL LAW I – DIGESTED CASES BY: BOAQUINA, MARY CRISCHAEL
99

JARCO V. CA
321 SCRA 375

FACTS: Crisel and Zhieneth (6 years old) were at the department store. Criselda was signing her
credit card slip when she heard a loud thud. She looked behind her and beheld her daughter
pinned beneath the gift-wrapping counter structure. She was crying and shouting for help. She
was brought to Makati Medical, where she died after 14 days.

The Aguilars demanded upon Jarco Marketing the reimbursement of the hospitalization and
medical bills but Jarco refused to pay. Jarco’s defense that it observed diligence of a good father
of a family in the selection, supervision and control of its employees. Also, the mother was
negligent in exercising care and diligence by allowing her daughter to freely roam around in the
store. Zhieneth was guilty of negligence since she climbed the counter, triggering its eventual
collapse on her.

ISSUE: Whether or not the same was attributable to private respondents for maintaining a
defective counter or to the mother and Zhieneth for failing to exercise due and reasonable care
while inside the store premises.

HELD: No. Conclusive presumption that favors children below 9 years old in that they are
incapable of contributory negligence. Even if we substitute negligence to Zhieneth and assume
that she climbed over the counter, no injury should have occurred if we accept petitioners’ theory
that the counter was stable and sturdy. If that was the truth, a 6-year old could not have caused
the counter to collapse.

99
CRIMINAL LAW I – DIGESTED CASES BY: BOAQUINA, MARY CRISCHAEL
100

GUEVARA V. ALMODOVAR
G.R. 75256. January 26, 1989

FACTS: Petitioner John Philip Guevarra, then 11 years old, was playing with his best friend
Teodoro Almine, Jr. and three other children in their backyard in the morning of 29 October 1984.
They were target-shooting a bottle cap (tansan) placed around fifteen (15) to twenty (20) meters
away with an air rifle borrowed from a neighbor. In the course of their game, Teodoro was hit by
a pellet on his left collar bone which caused his unfortunate death.

ISSUE: Whether or not an eleven (11) year old boy could be charged with a crime of homicide
thru reckless imprudence

HELD: Intent and discernment convey two distinct thoughts. Both are products of mental
processes within a person, the former refers to the desired of one’s act while the latter relates
to the moral significance that person ascribes to the said act. Hence, a person may not intend to
shoot another but may be aware of the consequences of his negligent act which may cause injury
to the same person in negligently handling an air rifle. It is not connect, therefore, to argue, as
petitioner does, that since a minor above 9 years of age but below 15 years acted with
discernment, then he intended such act to be done. He may negligently shoot his friend, thus,
did not intend to shoot him, and at the same time recognize undesirable result of his negligence.

100
CRIMINAL LAW I – DIGESTED CASES BY: BOAQUINA, MARY CRISCHAEL
101

PEOPLE V. SARCIA
G.R. No. 169641. September 10, 2009

FACTS: AAA, five year old, together with her cousin and two other playmates were playing in the
yard. Richard Sarcia, 18 or 19 years old, at the time of the time appeared and invited AAA to go
with him to the backyard. Sarcia removed AAA’s shorts and underwear, made her lie on her back
and then he laid on the top of AAA and made up-and-down movement which caused AAA to feel
pain in her genital are. AAA’s cousin saw the whole incident and reported it to AAA’s mother but
was rebuffed. AAA’s father filed a complaint for acts of lasciviousness .

ISSUE: Whether or not the accused is guilty beyond reasonable doubt of the crime of rape.

HELD: Yes. Inability of the victim, AAA, to recall the exact date of the incident cannot discredit
credibility of the victim since it is not an essential element of the crime. The employment of force,
threat, intimidation are not element of statutory rape, only carnal knowledge must be proven to
have taken place. Alibi or denial is the weakest defense.

101
CRIMINAL LAW I – DIGESTED CASES BY: BOAQUINA, MARY CRISCHAEL
102

PEOPLE V. MANTALABA
G.R. 186227. July 20, 2011

FACTS: Allen Mantalaba, 17 years old, is selling shabu. A buy-bust operation was organized. The
two poseur-buyers approached Allen who was said to be in the act of selling shabu. Appellant
handed a sachet of shabu to one of the buyers and the latter gave the marked money to the
appellant. Once the transaction was completed, the police handcuffed the appellant.

ISSUE: Whether or not there should have been a suspension of sentence by reason of minority.

HELD: Yes. The appellant was 17 years old when the buy-bust operation took place or when the
said offense was committed, but was no longer a minor at the time of the promulgation of the
RTC’s decision. It may be argued that the appellant should have been entitled to a suspension of
his sentence under Section 38 and 68 of RA 6344 which provide for its retroactive application. If
said child in conflict with the law has reached eighteen (18) years of age while under suspended
sentence, the court shall determine whether to discharge the child in accordance with this Act,
to order execution of sentence, or to extend the suspended sentence for a certain specified
period or until the child reaches the maximum age of twenty-one (21) years.

Hence, the appellant, who is now beyond the age of twenty-one (21) years can no longer avail of
the provisions of Sections 38 and 40 of RA 9344 as to his suspension of sentence, because such
is already moot and academic.

102
CRIMINAL LAW I – DIGESTED CASES BY: BOAQUINA, MARY CRISCHAEL
103

US VS. CABALLEROS
4 Phil 350

FACTS: Four American school teachers were murdered and buried. Robert Baculi and Apolonio
Caballeros were convicted as accessories to the crime of assassination or murder, having buried
the corpses of the victims to conceal the crime. They were allegedly coerced. Roberto Baculi, one
of the accused and it appears that he took part in the burial of the teachers because he was
compelled to do so by the murderers. According to a witness named Teodoro Sabate, Baculi was
not a member of the group and that he was in the banana plantation gathering some bananas
when Damaso and Isodoro called Baculi and by by striking him with the butts of their guns forced
him to bury the corpses. Another accused by the name of Apolonio Caballeros confessed by the
promise that nothing would be done to them.

ISSUE: Whether or not the defense under Art12(5) is tenable

HELD: Yes. Not only is Baculi‘s confession that he only assisted in the burial of the corpses
because he was compelled by the murderers, but this was corroborated by the only eyewitness
to the crime, Sabate. Sabate said that he was present when the Americans were killed; that Baculi
was not a member of the group of murderers but he was in the banana plantation gathering
some bananas; that when he heard the shots he began to run; that he was, however, seen by
Damaso and Isidro, the leaders of the band; that the latter called to him and striking him with the
butts of their guns forced him to bury the corpses. As for Caballeros, there was no proof that he
took any part in the execution of the crime; there was conclusive proof to the contrary. Sabate
and Baculi declared that Caballeros did not take any part in the burial of the aforesaid corpses,
nor was he even in the place of the occurrence when the burial took place. Their failure to report
the crime is not an offense punished by the Penal Code

103
CRIMINAL LAW I – DIGESTED CASES BY: BOAQUINA, MARY CRISCHAEL
104

US VS. EXALTACION
3 Phil 339

FACTS: Liberate Exaltacion and Buenaventura Tanchinco were charged with rebellion, willfully
and illegally bound themselves to take part in a rebellion against the government of the US,
swearing allegiance to the Katipunan Society (whose purpose was to overthrow the government
by force of arms). Exaltacion and Tanchinco claim that they were captured by armed bandits and
were compelled to sign documents (containing oath taken in the name of God and a covenant to
carry out superior orders of the Katipunan Society and never disobey them until their death in
the defense of the mother country) under threat of death. Exaltacion and Tanchinco reported
the incident to the governor, lieutenant of volunteers and the president of Meycauayan.
Witnesses testified to this fact as well.

ISSUE: Having signed the said documents, are the defendants guilty of the crime of rebellion? Or
did defendants incur criminal liability when they signed the documents?

HELD: No. The evidence for the prosecution and the documents signed by the accused is not
sufficient to prove the guilt of the latter or to justify the imposition upon them of the penalty
inflicted by the judgment of the court below. The facts, established by the evidence, that the
defendants were kidnapped by brigands who belonged to the Contreras Band, and that they
signed the said documents under compulsion and while in captivity, relieve them from all criminal
liability from the crime of rebellion of which they are charged. The conduct of the defendants in
presenting themselves to the authorities as soon as they were released is corroborative of their
innocence. Guilt of defendants was not established beyond reasonable doubt.
Judgment:
Decision of the lower court REVERSED. Defendants ACQUITTED

104
CRIMINAL LAW I – DIGESTED CASES BY: BOAQUINA, MARY CRISCHAEL
105

POMOY V. PEOPLE
439 SCRA 439

FACTS: Tomas Balboa was a teacher in Concepcion College of Science and Fisheries inIloilo. On
January 4, 1990, about 7:30 in the morning, some policemen arrived at and arrested Balboa,
allegedly in connection with a robbery which took place in the municipality in December
1989.Balboa was taken to the Headquarters of the already defunct 321stPhilippine Constabulary
Company at CampJalandoni, Sara,Iloilo.He was detained along with another suspect, Edgar
Samudio. At about 2 o’clock in the afternoon, petitioner, a police sergeant, directed Balboa to
come out from the jail where he is detained, purportedly for tactical interrogation at the
investigation room. At that time, petitioner had a gun, a .45 caliber pistol, tucked in a holster
which was hanging by the side of his belt. When petitioner and Balboa were near the
investigation room, two (2) gunshots were heard. When the source of the shots was verified,
petitioner was seen still holding a .45 caliber pistol, facing Balboa, who was lying in a pool of
blood, about two (2) feet away. Certain Dr. Palma, who happened to be at the crime scene as he
was visiting his brother in the Philippine Constabulary, examined Balboa, he (Dr. Palma) said that
it was unnecessary to bring Balboa to the hospital for he was dead

ISSUE: Whether or not the accused was in the lawful performance of his duties as an enforcer of
the law

HELD: Yes. The act of the petitioner to prevent the snatching of his service weapon by anyone,
especially by a detained person in his custody, is a lawful performance of his duty as a law
enforcer. Aside from the fact that all the elements of accident as an Exempting Circumstance
under Article 12 of the Revised Penal Code were present in this case, which exonerate the
accused from criminal liability, the accused was also in the lawful performance of his duties as
investigating officer at that time of the incident. He was a member, specifically one of the
investigators of the PNP stationed at the Iloilo Provincial Mobile Force Company, and that under
the instructions of his superior, he fetched the victim from the latter’s cell for a routine
interrogation. It was in the lawful performance of his duty as a law enforcer that petitioner tried
to defend his possession of the weapon when the victim suddenly tried to remove it from his
holster. He was duty-bound to prevent the snatching of his service weapon by anyone, especially
by a detained person in his custody. Such weapon was likely to be used to facilitate escape and
to kill or maim persons in the vicinity, including petitioner himself. The participation of petitioner,
if any, in the victim’s death was limited only to acts committed in the course of the lawful
performance of his duties as an enforcer of the law. The removal of the gun from its holster, the
release of the safety lock, and the firing of the two successive shots --all of which led to the death
of the victim --were sufficiently demonstrated to have been consequences of circumstances
beyond the control of petitioner.

105
CRIMINAL LAW I – DIGESTED CASES BY: BOAQUINA, MARY CRISCHAEL
106

TOLEDO V. PEOPLE
G.R. No. 158057. September 24, 2004

FACTS: Appellant requested the group of Ricky to refrain from making any noise so he could go
to sleep. On the other hand, Ricky rose from bed when they heard stones being hurled at the
roof of the house. He saw appellant stoning their house. Ricky asked appellant why he was
stoning their house. Appellant did not answer, and, without a warning, stabbed Ricky on the
abdomen with a bolo. Appellant as his defense, said that he pointed his bolo, which was on his
right hand, towards Ricky. The bolo accidentally hit Ricky on the stomach, and the latter lst his
balance and fell to the floor. Appellant surrendered to barangay captain.

ISSUE: Whether or not accused-appellant can be criminally held liable for the accidental death or
Ricky Guarte.

HELD: Yes. The petitioner failed to prove that the victim was killed by accident, without fault or
intention on his part to cause it. The petitioner was burdened to prove with clear and convincing
evidence, the essential requisites for exempting circumstance under Art 12 (4) of the RPC which
are: (1) A person is performing a lawful act; (2) With due care; (3) He causes an injury to another
by mere accident; (4) Without fault or intention to cause.

106
CRIMINAL LAW I – DIGESTED CASES BY: BOAQUINA, MARY CRISCHAEL
107

PEOPLE V. DORIA
301 SCRA 668

FACTS: Members of the PNP Narcotics Command (NarCom) received information that one “Jun”
was engaged in illegal drug activities in Mandaluyong. NarCom decided to entrap and arrest “Jun”
in buy-bust operation. At the operation, 7:20am, Jun appeared. PO3 Manlangit gave the marked
bills to Jun worth P1,600. Jun asked PO3 to wait for him to get the marijuana. When Jun was
about to give the marijuana, Manlangit arrested Jun. They went to house of Jun and found a box,
which contained 10 bricks of what appeared to be dried marijuana.

ISSUE: Whether or not the arrest of Doria in entrapment was lawful.

HELD: Yes. Jun or Doria was caught in the act of committing an offense. When an accused is
apprehended in flagrante delicto as a result of a buy-bust operation, the police are not only
authorized but duty-bound to arrest him even without warrant.

107
CRIMINAL LAW I – DIGESTED CASES BY: BOAQUINA, MARY CRISCHAEL
108

ARANETA V. CA
142 SCRA 532

FACTS: Complainant Yoyongco approached the appellant, Atty. Araneta, to inquire about the
procedure for filing a claim for death of her husband compensation. She was told by the
appellant that she had to pay PhP100 so that her claim would be acted upon. Yoyongco went to
her bro-in-law, Col. Yoyongco; the latter then gave her 2 PhP50 bills and instructed her to go to
Col Laureaga. The latter concocted a plan to entrap the appellant. The 2 PhP50 bills were marked
w/ notations “CC-NE-1” and “CC-NE-2”, photographed and dusted w/ ultra-violet powder.
She again approached Araneta the officer to process her claim but was again asked if she already
had P100. Yoyongco brought out the 2 P50 bills & handed them to the appellant. As she took
hold of the money, officer Balcos grabbed her hand & arrested her. In the PC headquarters, Atty
Araneta’s hands were examined for the presence of UV powder & result was positive.

ISSUE: Whether or not the accused was caught through entrapment

HELD: Yes. There is entrapment when law officers employ ruses and schemes to ensure the
apprehension of the criminal while in the actual commission of the crime. Wherefore, appellant
is guilty of the crime of bribery, a violation of Sec 3 RA No 3019 known as the “Anti-Graft and
Corrupt Practices Act.”

108
CRIMINAL LAW I – DIGESTED CASES BY: BOAQUINA, MARY CRISCHAEL
109

PEOPLE V. CASIO
G.R. No. 211465. December 3, 2014

FACTS: International Justice Mission (IJM), a non-governmental organization, coordinated with


the police to initiate an entrapment for those who are engaged in human trafficking in Cebu City.
5 polices formed the team of police operatives, Luardo and Veloso were designated as decoys,
pretending to be tour guides looking for girls to entertain their guests. IJM provided them with
marked money, which was recorded in the police blotter. The team went to Queensland Motel
and rented adjacent Rooms 24 and 25. Room 24 was designated for the transaction while Room
25 was for the rest of the police team. PO1 Luardo and PO1 Veloso proceeded to D. Jakosalem
Street in Barangay Kamagayan, Cebu City’s red light district where the accused noticed them and
called their attention. Negotiation occurred and upon the signal, the accused was arrested and
the two minors were taken into custody by the DSWD officials.

ISSUE: Whether or not accused is liable for human trafficking.

HELD: Yes. Under Republic Act No. 10364, the elements of trafficking in persons have been
expanded to include the following acts:

1. The act of “recruitment, obtaining, hiring, providing, offering, transportation, transfer,


maintaining, harboring, or receipt of persons with or without the victim’s consent or
knowledge, within or across national borders;”
2. The means used include “by means of threat, or use of force, or other forms of coercion,
abduction, fraud, deception, abuse of power or of position, taking advantage of the
vulnerability of the person, or, the giving or receiving of payments or benefits to achieve
the consent of a person having control over another person”
3. The purpose of trafficking includes “the exploitation or the prostitution of others or other
forms of sexual exploitation, forced labor or services, slavery, servitude or the removal or
sale of organs”

The Court of Appeals found that AAA and BBB were recruited by accused when their services
were peddled to the police who acted as decoys. AAA was a child at the time that accused
peddled her services.66 to work as a prostitute because she needed money. AAA also stated that
she agreed Accused took advantage of AAA’s vulnerability as a child and as one who need money,
as proven by the testimonies of the witnesses.

Knowledge or consent of the minor is not a defense under Republic Act No. 9208.

“The recruitment, transportation, transfer, harboring, adoption or receipt of a child for the
purpose of exploitation or when the adoption is induced by any form of consideration for
exploitative purposes shall also be considered as ‘trafficking in persons’ even if it does not involve
any of the means set forth in the preceding paragraph.”

109
CRIMINAL LAW I – DIGESTED CASES BY: BOAQUINA, MARY CRISCHAEL
110

ARTICLE 13: Mitigating Circumstances

PEOPLE V. URAL
56 SCRA 138

FACTS: Brigido Alberto, who just got out of jail, saw Policeman Ural inside the jail. Ural as boxing
the detention prisoner, Felix Napola. Napola collapsed on the floor then Ural stepped on his body.
After a short interval, Ural returned with a bottle. He poured its contents on Napola and ignited
it with a match and left the cell. Napola shouted for help but nobody came to succor him. Napola
died.

ISSUE: Whether or not Ural should be held criminally liable.

HELD: Yes. There is a rule that “an individual who unlawfully inflicts rounds upon another person,
which result in the death of the latter, is guilty of the crime of homicide, and the fact that the
injured person did not receive proper medical attendance does not affect the criminal
responsibility.

The crime committed by Ural was murder by means of fire. The accused took advantage of his
public position. He could not have maltreated Napola if he was not a policeman on guard duty.
There is a mitigating circumstance “that the offender had no intention to commit so grave a
wrong as that committed”. It is manifest from the fact that Ural had no intent to kill Napola. His
design was only to maltreat him may be because of his drunken condition he was making a
nuisance of himself inside the detention cell. When Ural realized the fearful consequences of his
felonious act, he allowed Napola to secure medical treatment

110
CRIMINAL LAW I – DIGESTED CASES BY: BOAQUINA, MARY CRISCHAEL
111

PEOPLE V. CALLET
G.R. 135701. May 9, 2002

FACTS: Alfredo Senado, his 12-year old son, Lecpoy Senador and Eduardo Perater were watching
a cara y cruz game at the flea market. Out of nowhere, the accused, Elbert Callet, appeared
behind Alfredo and stabbed the latter on the left shoulder near the base of the neck with a 9-
inch hunting knife. Alfredo died.

Defense gave a different account of stabbing incident: the accused, while watching the volleyball
game, was hit on the left side of the body of Alfredo’s elbow. Next, Alfredo grabbed his left arm
and tried to twist it. As Alfredo was pulling out a hunting knife from his waist, the accused
managed to stab him first. Thereafter, he ran towards the municipal hall to surrender.

ISSUE: Whether or not there is a mitigating circumstance present in this case.

HELD: Yes. The trial court correctly credited the accused with voluntary surrender to mitigate his
liability. Voluntary surrender requires that the offender had not been actually arrested; that he
surrendered himself to a person in authority or to the latter’s agent; and that the surrender was
voluntary. The record reveal that the accused ran toward the municipal building after the
stabbing incident. On his way to the municipal building, he admitted to the barangay tanods that
he stabbed the victim. Although he did not immediately turn over his weapon to them for fear of
retaliation from victim’s relatives, he did so as soon as they reached the municipal building.

The accused cannot claim that his liability should be mitigated by the fact that he had no intention
to commit so grave a wrong. It is weighed based on the weapon used, the part of the body
injured, the injury inflicted and the manner it is inflicted. The fact that the accused used a 9-inch
hunting knife in attacking the victim from behind, without giving him an opportunity to defend
himself, clearly shows that he intended to do what he actually did, and he must be held
responsible, without the benefit of this mitigating circumstance.

111
CRIMINAL LAW I – DIGESTED CASES BY: BOAQUINA, MARY CRISCHAEL
112

PEOPLE V. LEONOR
305 SCRA 285

FACTS: Christopher Leonor entered and inquired about the cost of tooth extraction. Dr. Tarlengco
said the price and Christopher said that he would be back. Minutes later, Leonor came back and
Dr. Tarlengco told him to take a seat and wait. Dr. Tarlengco was preparing her dental intruments
when Leonor barged in and demanded money. Dr. Tarlengco told Leonor that the money was on
the table. On hearing this, Leonor stabbed Dr. Tarlengco and grabbed her watch and ran away
Dr. Tarlengco struggled out of the clinic and saw the man running out of the building, Dr.
Tarlengco shouted for help. Dr. Tarlengco died.

According to Christopher, he asked Dr. Tarlengco how much an extraction cost, and was told that
the fee was P150 per tooth. Christopher negotiated a charge of P100 per tooth. Just as she was
about to inject anesthesia, she remarked that she would charge P150 per tooth pulled.
Christopher pushed away Dr. Tarlengco’s hand, which angered her. She cursed. As Christopher
was making his way out of the clinic, Dr. Tarlengco cursed and pushed him, at which moment he
blacked out. He realized that he had stabbed the dentist. He ran out of the building. He came
across a police and admitted that he had stabbed Dr. Tarlengco, but denied that he had taken
P900 and a Titus wristwatch from the victim.

ISSUE: Whether or not there is a mitigating circumstance present in this case.

HELD: No. We find no mitigating circumstance in this case. Lack of intent to commit so grave a
wrong does not mitigate in homicide cases where the accused used a deadly weapon in inflicting
mortal wounds on vital organs of the victim as in this case. The provocation sufficient to mitigate
an offense must be proportionate to the gravity of the retaliatory act.

Christopher is thus claiming that a push and bad words justify retaliation with a knife. Such claim
is undeserving of belief and does not entitle him to the benefit of the mitigating circumstance
prior provocation by the offended party. He could not have been provoked by passion or
obfuscation as, according to him, he momentarily blacked out and instantly found his fan knife
embedded in Dr. Tarlengcos chest. Courts cannot appreciate passion and obfuscation unless
there is clear showing that there were causes naturally tending to produce such powerful
excitement as to deprive the accused of reason and self-control.

112
CRIMINAL LAW I – DIGESTED CASES BY: BOAQUINA, MARY CRISCHAEL
113

PEOPLE V. ESPINA
G.R. 132325. July 16, 2002

FACTS: The members of “ripa-ripa” went to the house of Eufronia Pagas for their scheduled
contribution to a fund intended for a wedding celebration. Among those present thereat were
Romeo Bulicatin, Rogelio Espina, Samson Abuloc who were having a drinking spree and playing a
card game. When accused-appellant arrived, Romeon asked 3 bottles of “kulafu” wine. Later on,
Romeo again demanded another bottle “kulafu” wine but this time, the latter refused to give in
to the demand. The accused-appellant got angry. Later that evening, the accused-appellant called
Romeo. Romeo was still at the stairway and when he turned his back towards the accused
appellant, the latter shot him, hitting him at the back. Romeo ran away but he was chased by
accused-appellant who fired 2 more shots at him.

Testimonies of accused-appellant and Maximiano Dormal may be summarized as follows:


accused-appellant was in the house of Eufronia Pagas to represent his father in a meeting to
prepare for a wedding celebration. Bulicatin asked him to buy 3 bottles of “kulafu” wine to which
he acceded. Bulicatin again demanded another bottle “kulafu” wine from accused-appellant. The
latter, however, refused to obey prompting Bulicatin to urinate on accused appellant. Thie
infuriated the accused-appellant, but walked away. Bulicatin pursued him. Accused-appellant
tried to evade Bulicatin but the latter caught up with him and stabbed him on his side.

ISSUE: Whether or not Romeo Espina acted in immediate vindication of a grave offense.

HELD: Yes. The trial court correctly appreciated the mitigating circumstance of having acted in
immediate vindication of a grave offense. The accused-appellant was urinated on by the victim
in front of the guests. The act of the victim, which undoubtedly insulted and humiliated accused-
appellant, came within the purview of a “grave offense” under Art 13, paragraph 5, of the RPC>
thus, this mitigating should be appreciated in favor of accused-appellant. The accused-appellant
Romeo Espino is guilty beyond reasonable doubt of the crime of murder.

113
CRIMINAL LAW I – DIGESTED CASES BY: BOAQUINA, MARY CRISCHAEL
114

PEOPLE V. DIOKNO
63 Phil. 601

FACTS: Salome Diokno, to whom Yu Hiong was engaged for about a year, invited the latter to go
with her. Yu Hiong accepted the invitation. Roman Diokno telegraphed his father that Salome
had eloped with Yu Hiong. Roman Diokno and Epifanio went to search for the elopers. When Yu
Hiong saw them, he ran upstairs and they pursued him. As the Chinese found the door of the
house locked, he shouted that it be opened for him. He was overtaken by the accused who carried
balisong of different sizes. Yu Hiong fell on his knees and implored pardon. Roman Diokno
stabbed him with the knife in the back and the later in the left side. Epifanio Diokno stabbed him
once.

Upon hearing the shots, municipal policeman appeared and found Yu Hiong pale and lying on the
landing of the stairs. He asked who had wounded the Chinese and the accused Epifanio Diokno
answered it was he.

ISSUE: Whether or not there is mitigating circumstances present in this case.

HELD: Yes. The immediate vindication of a grave offense to said accused, may be taken into
consideration in favor of two accused, because although the elopement took place on January 4,
1935 and the aggression on the 7th of the said month and year, the offense did not cease while
Salome’s whereabouts remained unknown and her marriage to the deceased unlegalized.
Therefore, there was no interruption from the time the offense was committed to the vindication
thereof. It based on the fact that the herein accused belong to a family of old customs to whom
the elopement of a daughter with a man constitutes a grave offense to their honor and causes
disturbance of the peace and tranquility of the home and at the same time spreads uneasiness
and anxiety in the minds of the members thereof.

The accused having acted upon an impulse so powerful as naturally to have produced passion or
obfuscation, may also be considered, the fact that the accused saw the deceased run upstairs
when he became aware of their presence, as if he returned to deal with them after having gravely
offended them, was certainly a stimulus strong enough to produce in their mind a fit of passion
which blinded them and ed them to commit the crime with which they are charged.

The circumstance where Epifanio Diokno having surrendered himself immediately to the agents
of authority should also be taken into consideration.

114
CRIMINAL LAW I – DIGESTED CASES BY: BOAQUINA, MARY CRISCHAEL
115

US V. AMPAR
37 Phil 201

FACTS: Roast pig was being served d uring fiesta Province of Occidental Negros. The accused
Clemente Ampar, proceeded to the kitchen and asked Modesto Patobo for some of the delicacy.
Patobo answered, “There is no more. Come here and I will make roast pig of you”. With this
provocation, a little later while the said Modesto Patobo was squatting down, the accused came
up behind him and struck him on the head with an ax, causing death the following day.

ISSUE: Whether or not there is mitigating circumstance in this case.

HELD: Yes. The act was committed in the immediate vindication of a grave offense to the one
committing the felony. It is present when one may term the remarks of Patobo to the accused is
admitted. Whether these remarks can properly be classed as “a grave offense” is more uncertain.
The offense which the defendant was endeavoring to vindicate would to the average person be
considered as a mere trifle. But to this defendant, an old man, it evidently was a serious matter
to be made the butt of a joke in the presence of so many guests.

115
CRIMINAL LAW I – DIGESTED CASES BY: BOAQUINA, MARY CRISCHAEL
116

PEOPLE V. PAJARES
210 SCRA 237

FACTS: Renato Perez and Diosdado Viojan were on their way to a store when appellant Pajares
suddenly appeared from behind and hit Viojan with a baseball bat at the back of his head. When
Perez tried to help Viojan, he too, was attacked by Pajares hitting him at the back below the left
shoulder. Viojan died.

Leandro Pajares denied the allegations. He asserts that, at the time of the incident, he was inside
the store of Alex Blas watching television. Hence, he did not see who hit Viojan and Perez. After
the commotion, he went home and slept. At 3:30am the next day, he was arrested without asking
any question and went with arresting officers to police station. He points out that hours before
the incident, Roberto Pajares, appellant’s younger brother, was mauled by group of Diosdado
Viojan. The mauling of the latter is a big insult and truly offending to the appellant and his family.
Hence, the clubbing of Diosdado Viojan by Leandro Pajares was a vindication of the grave offense
committed against his family, a mitigating circumstance under par. 5 of Article 13 of the RPC.

ISSUE: Whether or not the mitigating circumstance of immediate vindication of a grave offense
can be appreciated in favor of Pajares.

HELD: No. While it may be true that appellant’s brother Roberto Pajares was mauled by
companions of the deceased (Viojan), it must be emphasized that there is a lapse of about ten
(10) hours between the said incident and the killing of Diosdado Viojan. Such interval of time was
more than sufficient to enable Pajares to recover his serenity. Hence, the mitigating circumstance
of immediate vindication of a grave offense cannot be appreciated in his favor. Pajares is
convicted of the crime of Murder.

116
CRIMINAL LAW I – DIGESTED CASES BY: BOAQUINA, MARY CRISCHAEL
117

PEOPLE V. ADLAWAN
G.R. 131839. January 30, 2002

FACTS: At dawn of November 15, 1992, the deceased(Nequito Ortizano), Benjamin Basubas and
Quirino Cinco and Olicer Bonayan were inside a fenced disco area. At 2am, Benjamin and Quirino
were alerted by commotion outside. They saw the deceased raising his hands in front of the
accused. The father of the accused was also lying unconsciously. All of a sudden, accused fired a
gun at the deceased. The latter fell on a shallow canal the accused followed the deceased and
delivered a fatal shot.

The defense averred that the accused was inside a fenced disco area. He heard somebody
shouting and when he turned to the source of the disturbance, he saw his father, lying
unconsciously. Accused ran to his father whom he thought was already dead. He saw the
deceased about 2 ½ arm’s length away, holding a gun and told him “Do you want to follow your
father?”. The accused lunged at the deceased, forcing the muzzle of gun to be pointed at
deceased’s chest. Accused was able to get hold of the fun and again fired at the deceased.
Thereafter, he fled and hid in Manila until January 23, 1997, when he finally decided to surrender.
Trial court found him guilty of the crime of murder. Hence, this appeal.

ISSUE: Whether or not the mitigating circumstance of passion or obfuscation should be


appreciated to mitigate the accused’s criminal liability:

HELD: Yes. The mitigating circumstance of passion or obfuscation should be appreciated. The
accused thought his father whose face was bloodied and lying unconscious on the ground was
dead. Surely, such scenario is sufficient to trigger an uncontrollable burst of legitimate passion.
His act, therefore, of shooting the deceased, right after learning that the latter was the one who
harmed his father, satisfies the requisite of mitigating circumstance of passion or obfuscation
under Par. 6, Art 13 of the RPC.

The requisites of this mitigating circumstance are: (1) that there be an act, both unlawful and
sufficient to produce such a condition of mind; and (2) said act which produced the obfuscation
was not far removed from the commission of the crime by a considerable length of time, during
which the perpetrator might recover his normal equanimity.

117
CRIMINAL LAW I – DIGESTED CASES BY: BOAQUINA, MARY CRISCHAEL
118

US V. HICKS
14 Phil 217

FACTS: Augustus Hicks and Agustina Sola illicitly lived together for 5 years until trouble arising
between them. Agustina left, live with her brother-in-law (Luis Corrales). She contracted new
relation with Wallace Current, a corporal in Army who live in the said house.

Augustus went to the said house and talked to Current. But Current said that Agustina does not
want to live with Augustus anymore. Current saw Hicks drawing revolver from his pocket, he
caught him by the hand, but the latter, snatching his hand roughly away, said “Don’t do that”,
Current hid himself behind the partition, just as Hicks drew his revolver and fired at Agustina who
was in the sala. Agustina died.

ISSUE: Whether or not Hicks can avail himself of mitigating circumstance of passion and
obfuscation.

HELD: No. No mitigating circumstance is present not even that mentioned in par 7 Art 9 of the
RPC, to wit loss of reason and self-control produced by jealousy as alleged by the defense,
inasmuch as the only causes which mitigate the criminal responsibility for the loss of self-control
are such as originate from legitimate feelings, not those which arise from vicious, unworthy, and
immoral passions.

All the foregoing circumstance conclusively proved that the accused, deliberately and after due
reflection had resolved to kill the woman who had left him for another man, and in order to
accomplish his intention safety, when he appeared in the house, he appeared to be in a proper
manner, doubtless in order to successfully accomplish his criminal design, behaving himself
properly as he had planned to do beforehand.

118
CRIMINAL LAW I – DIGESTED CASES BY: BOAQUINA, MARY CRISCHAEL
119

US V. DELA CRUZ
22 Phil 429

FACTS: The convict, in the heat of passion, killed the deceased, who had theretofore been his
querida (concubine or lover) upon discovering her in flagrante in carnal communication with a
mutual acquaintance. Trial Court ruled that its commission was not marked by either aggravating
or extenuating circumstances and sentenced the convict to 14 years 8 months and 1 day of
reclusion temporal.

ISSUE: Whether or not there is extenuating/mitigating present in this case.

HELD: Yes. The commission of the offense of which defendant was convicted was marked with
the extenuating circumstance defined in subsection 7 of article 9, in that defendant "acted upon
an impulse so powerful as naturally to have produced passion and obfuscation," the evidence
disclosing that in the heat of passion he killed the deceased, who had theretofore been his
querida (concubine or lover), upon discovering her in flagrante in carnal communication with a
mutual acquaintance. It was a “sufficient impulse” in the ordinary and natural course of things to
produce the passion and obfuscation which the law declares to be one of the mitigating
circumstances to be taken into consideration.

119
CRIMINAL LAW I – DIGESTED CASES BY: BOAQUINA, MARY CRISCHAEL
120

PEOPLE V. VENTURA
G.R. 148145-46. July 5, 2004

FACTS: Accused Ventura noticed that his wife (Johanna), who had been previously employed as
a house helper of the Bocateja spouses, was wearing a ring. When he confronted her, she said it
came from Jaime Bocateja who was courting her, and that it was because Jaime’s wife (Aileen
Bocateja), who had discovered their illicit relationship that she had been dismissed from Bocateja
household. At that same day, Accused Flores visited his uncle Ventura. He, who had previously
worked for a day at the meat shop of the Bocateja spouses confirmed that Johanna and Jaima
were having an affair.

Spouses Jaime and Aileen Bocateja were fast asleep in their room. At around 2am, Jaime was
roused from his sleep by accused Ventura who, together with his nephew accused Flores, entered
the couple’s room by cutting a hole in kitchen door. Ventura pointed the revolver at Jaime’s face
told him to stop the illicit relationship with his wife, but the latter almost succeeded in getting
the fun, Flores stabbed Jaime 3 times. Jaime threw a nearby plastic stool at the glass window
causing it to break and cried out for help. Aileen was awakened, began shouting as she saw her
husband in mortal danger. Flores stabbed her which multiple stab wounds caused her death.

ISSUE: Whether or not there is mitigating circumstance to offset the foregoing aggravating
circumstances.

HELD: No. The accused presented evidence to prove that Jaime Bocateja and Johanna Ventura,
wife of the accused Ventura, were maintaining an illicit relationship. The evidence on this point
is principally hearsay – the alleged admissions made by Johanna of the relationship. There is no
doubt that the accused Venture believes that his wife and Jaime are clandestine lovers. It is fairly
reasonable that it is Ventura’s belief of this illicit relationship which prompted him to confront
Jaime which may be ruled out as passion or obfuscation or immediate vindication of a grave
offense as mitigating circumstance.

While jealousy may give rise to passion or obfuscation, for the appreciation or this mitigating
circumstance it is necessary that the act which produced the obfuscation was not far removed
from the commission of the crim by a considerable length of time, during which the perpetrator
might recover his normal equanimity. Without question, sufficient time had passed for
appellants’ emotions to cool and for them to recover their equanimity.

120
CRIMINAL LAW I – DIGESTED CASES BY: BOAQUINA, MARY CRISCHAEL
121

PEOPLE V. CRISOSTOMO
G.R. L-32243, April 15, 1988

FACTS: On Christmas Day, Eugene Crisostomo was passing near the house of Romeo Geronimo
and invited him to have a drink in the place of a friend. Romeo declined. Suddenly, Eugene rushed
towards Romeo who was standing near a store facing the street with his back towards Eugene
and shot him. Romeo fell to the ground mortally wounded while Eugenio ran away.

ISSUES: a.) Whether or not the accused is entitled to the mitigating circumstance of drunkenness.
b.) Whether or not the mitigating circumstance of voluntary surrender should be appreciated.

HELD: a.) No. The accused asserts that he had been drinking from 1pm on that day and that he
had been drunk 5 times in his entire life so that it is not habitual. The allegation that he was drunk
in committing the offense is self-serving and uncorroborated. Besides, the appellant admitted
that he was only dizzy, and that he was in the way to another drinking spree. Obviously, he had
not drunk enough. He remembers the details of the shooting, the time it started and ended, how
much wine he imbibed and the persons who were with him. He realized the gravity of the offense
so he fled and hid from the authorities for 10 days. All these acts of a man whose has not been
impaired.

b.) Yes. Although he hid himself from the authorities for 10 days, he voluntarily surrendered to
the authorities thereafter upon the advice of his parents. Requisites of voluntary surrender: (a)
the offender had not actually been arrested; (b) the offender surrendered himself to a person in
authority or the latter’s agent; and (c) the surrender was voluntary.

121
CRIMINAL LAW I – DIGESTED CASES BY: BOAQUINA, MARY CRISCHAEL
122

PEOPLE V. JAVIER
311 SCRA 576

FACTS: Eduardo Javier and the victim Florentina Javier were legally married. Between 2-3am,
Consolacion Javier heard her mother shouting “Your father is going to killing me.” After she heard
her mother scream for help, Consolacion rushed to her sister Alma told her that their parents
were quarreling. The two proceeded to their brother Manuel’s house. The three then proceeded
to their parents’ house. They found a lifeless body of their mother and his father (accused),
wounded in the abdomen. Eduardo confessed to Manuel that he killed his wife and thereafter
allegedly stabbed himself.

Accused told the Court that he killed his wife because he could not sleep for almost a month. He
claimed that when the killing took place, his mind totally went blank and he did not know what
he was doing. He claims that he was insane at the time of the incident.

ISSUE: Whether or not the accused can claim mitigating circumstances of illness.

HELD: No. Aside from the testimony of the accused that his mind went blank due to loss of sleep,
no medical findings that presented regarding his mental condition at the time of the killing. The
Court can hardly rely on the bare allegations of accused, nor on mere presumptions and
conjectures. No clear and convincing evidence was shown that accused was suffering from illness
which diminished his exercise of will-power at the time of the killing.

It is clear that accused was aware of the acts he committed. He remembered killing his wife with
use of bolo, he remembered trying to commit suicide by wounding himself with the same bolo
he used in killing his wife; he remembered being brought to the hospital. Since he remembered
the vital circumstance surrounding the ghastly incident, it shows that he was in full control of his
mental faculties.

122
CRIMINAL LAW I – DIGESTED CASES BY: BOAQUINA, MARY CRISCHAEL
123

PEOPLE V. VILLANUEVA
G.R. 172697, September 25, 2007

FACTS: Accused consulted Dr. Dy of Psychiatric Department of BGH for follow-up check-up. Dr.
Dy prescribed medicine and she allowed accused to go him. From BGC, accused had several beers
and had videoke. Upon reaching home, he went to buy some hotdogs which he cooked since he
was hungry. When his mother asked if she could have some, accused got irked because he did
not have breakfast and lunch. His mother got scared and ran away. Accused was so peeved that
he wanted to give vent to his anger. He went to his mother’s room.

Accused killed his niece Angelica (8yo), by boxing her on the head and kicking her several times
on different parts of the body. He also mauled his nephews Rexie (5yo) and Enrique Jr. (2yo).
Angelica died of massive brain edema and dubdural hemorrhage due to mauling. Rexie sustained
injuries and almost died were it not for medical intervention. Enrique Jr. suffered broken mouth.

Accused-appellant pleaded insanity. He claimed that he did not know what he has done.

ISSUE: Whether or not the accused may claim mitigating circumstance of illness/insanity.

HELD: Yes. There is no dispute that accused has a history of mental illness. He was diagnosed to
be suffering from schizophrenia, paranoid, episodic with inter-episode residual symptoms which
characterized by intermittent episodes of psychotic signs and symptoms. However, the defense
failed to prove that accused was completely deprived of intelligence in committing the act. Proof
of existence of some abnormalities in the mental faculties will not exempt the accused from
culpability, if it was shown that he was not completely deprived of freedom and intelligence.

123
CRIMINAL LAW I – DIGESTED CASES BY: BOAQUINA, MARY CRISCHAEL
124

PEOPLE V. NAVASCA
76 SCRA 70

FACTS: Lorenzo Soberano saw his co-accused sitting on the road by the river: Manuel Marquez
holding a thompson, Tomas Navasca armed with pistol and Florencio Geraldes carrying a carbine.
He bought tuba as requested by Navasca and he was invited to go with the group to serenade.
He was slapped by Navasca and threatened with a pistol should he not consent to go with the
group to rob. Out of fear, he did not refuse and as they reached the house of intended victim, he
was given a knife by Navasca and told to stand guard outside the house. From where he stood,
he heard gunshots coming from the house, after which his companions went out of the same and
they altogether fled.

There was judicial confession made by Florencio Geraldes and extra-judicial admission and
testimony of Lorenzo Soberano in court. They said that these were prepared by police and they
were compelled to affix their signature thereto. The court cannot sustain that they were
compelled thereto after he had been maltreated and promised freedom because there are
certain characteristic features in his statement that refute his claim of involuntariness – Lorenzo
took the witness stand for the prosecution notwithstanding the severe admonition by trial judge
of consequences of testimony.

ISSUE: Whether or not accused Lorenzo Soberano’s act of testifying for the prosecution and
revealing voluntarily in Open Court, the details of the crime, considered as mitigating
circumstance of similar analogous nature.

HELD: Yes. The act of testifying for the prosecution, without previous discharge, by Lorenzo
should be considered in his favor as a mitigating circumstance analogous to a plea of guilty. The
judge painstakingly advised and informed the accused of his rights as an accused and could not
be indicted for dereliction of his duties as judge. He insisted that as in fact he did testify against
himself and his co-accused. Nor can it be suspected that his act of testifying was prompted by his
belief that, in doing so, he would be released.

124
CRIMINAL LAW I – DIGESTED CASES BY: BOAQUINA, MARY CRISCHAEL
125

CANTA V. PEOPLE
353 SCRA 250

FACTS: Petitioner Exuperancio Canta was found guilty of violating P.D. No. 533 or Anti-Cattle
Rustling Law of 1974. Narciso Gabriel acquired from his half-sister a cow, subject of the case,
upon its birth on March 10, 1984. Narciso gave the care and custody of the animal to three
persons in different persons. Agapay, one of the caretakers, took the cow to graze in the
mountain of Pilipogan in Barangay Candatag, about 40 meters from his hut. However, when he
came back for it, he found the cow gone. He found hoof prints which led to the house of Filomeno
Vallejos. He was told that petitioner Exuperancio Canta had taken the animal. Two of the other
caretakers went to recover the animal from petitioner's wife, but they were informed that
petitioner had delivered the cow to petitioner’s father. Thereafter, the two went to the father of
the petitioner’s house. On their way, they met petitioner who told them that if Narciso was the
owner, he should claim the cow himself. Nevertheless, petitioner accompanied the two to his
father's house, where Maria recognized the cow. As petitioner's father was not in the house,
petitioner told Gardenio and Maria he would call them the next day so that they could talk the
matter over with his father. However, petitioner never called them. Hence, Narciso Gabriel
reported the matter to the police of Malitbog, Southern Leyte. 6 As a result, Narciso and
petitioner Exuperancio were called to an investigation. Petitioner admitted taking the cow but
claimed that it was his and that it was lost before. He presented two certificates of ownership as
evidence.

Narciso presented a certificate of ownership signed by the municipal treasurer describing the
cow as a two-year old female. On the reverse side of the illustrates the physical features of the
cow. All four caretakers of the cow identified the cow as the same one they had taken care of,
based on the location of its cowlicks, its sex and its color.

On the other hand, petitioner claimed he acquired the animal under an agreement which he had
with Pat. Diosdado Villanueva, that petitioner take care of a female cow of Pat. Villanueva in
consideration for which petitioner would get a calf if the cow produced two offsprings. Petitioner
claimed that the cow in question was his share and that it was born on December 5, 1984. This
cow, however, was lost on December 2, 1985. Petitioner said he reported the loss to the police.

Petitioner said that on March 14, 1986, his uncle Meno told him that he had seen the cow under
the care of Gardenio Agapay. Thereafter, he went to Agapay’s place to see whether the cow
would suckle the mother cow. As the cow did, petitioner took it with him and brought it, together
with the mother cow, to his father Florentino Canta.
Petitioner presented a Certificate of Ownership of Large Cattle, and a statement executed by
Franklin Telen, janitor at the treasurer's office, to the effect that he issued a Certificate of
Ownership of Large Cattle in the name of petitioner Exuperancio Canta.

125
CRIMINAL LAW I – DIGESTED CASES BY: BOAQUINA, MARY CRISCHAEL
126

ISSUE: Whether or not Canta is guilty of violation of PD 533

HELD: Yes. P.D. No. 533 defines cattle-rustling as . . . the taking away by any means, methods or
scheme, without the consent of the owner/raiser, of any of the abovementioned animals
whether or not for profit or gain, or whether committed with or without violence against or
intimidation of any person or force upon things.

The crime is committed if the following elements concur: (1) a large cattle is taken; (2) it belongs
to another; (3) the taking is done without the consent of the owner; (4) the taking is done by any
means, methods or scheme; (5) the taking is with or without intent to gain; and (6) the taking is
accomplished with or without violence or intimidation against person or force upon things.
First, accused-appellant should be given the benefit of the mitigating circumstance analogous to
voluntary surrender. The circumstance of voluntary surrender has the following elements: (1) the
offender has not actually been arrested; (2) the offender surrenders to a person in authority or
to the latter's agent; and (3) the surrender is voluntary. Second, the trial court correctly found
petitioner guilty of violation of P. D. No. 533, otherwise known as the Anti-Cattle Rustling Law of
1974. The trial court apparently considered P. D. No. 533 as a special law and applied (1) of the
Indeterminate Sentence Law, which provides that "if the offense is punished by any other law,
the court shall sentence the accused to an indeterminate sentence, the maximum term of which
shall not exceed the maximum fixed by said law and the minimum shall not be less than the
minimum term prescribed by the same." However, P. D. No. 533 is not a special law. The penalty
for its violation is in terms of the classification and duration of penalties prescribed in the Revised
Penal Code, thus indicating that the intent of the lawmaker was to amend the Revised Penal Code
with respect to the offense of theft of large cattle.

126
CRIMINAL LAW I – DIGESTED CASES BY: BOAQUINA, MARY CRISCHAEL
127

Article 14: Aggravating Circumstances


Insult to Public Authorities

PEOPLE V. RODIL
G.R. No. L-35156. November 20, 1981

FACTS: The deceased, PC Lt. Masana saw the appellant outside the restaurant blowing his
whistle. Lt. Masana and PC soldier Fidel asked the appellant if the gun tucked in his waist had a
license. Instead of answering, the appellant attempted to draw his gun. PC soldier Fidel
immediately grabbed appellant’s gun and asked him to go inside the restaurant. The appellant
asked Lt. Masana to return the gun but the latter rejected the appellant’s plea and told that they
would talk the matter over in the municipal building Indang, Cavite. When Lt. Masana was about
to stand up, appellant suddenly pulled out a double-bladed dagger and with it he stabbed Lt.
Masana which caused his death.

ISSUE: Whether or not an aggravating circumstance is present.

HELD: YES. Aggravating circumstance of disregard of rank should be appreciated because it is


obvious that the victim, PC. Lt. Masana Identified himself as a PC officer to the accused who is
merely a member of the Anti-Smuggling Unit and therefore inferior both in rank and social status
to the victim.

Also, the aggravating circumstance of contempt of, or insult to, public authority under paragraph
2 of Article 14 of the RPC can likewise be appreciated in the case at bar.

The evidence of the prosecution clearly established that Chief of Police Primo Panaligan of Indang
was present as he was taking his lunch in the same restaurant when the incident occurred. As a
matter of fact, the said chief of police was the one who embraced or grabbed the accused from
behind, wrested the dagger from him and thereafter brought him to the municipal building of
Indang. And appellant admittedly knew him even then as the town chief of police, although he
now claims that he went to the municipal building to surrender to the chief of police who was
not allegedly in the restaurant during the incident.

127
CRIMINAL LAW I – DIGESTED CASES BY: BOAQUINA, MARY CRISCHAEL
128

PEOPLE V. TAC-AN
G.R. No. 76338-39. February 26, 1990

FACTS: Appellant Renato and the deceased Francis were friends and members of the same gang.
When Francis withdrew from the Bronx gang, their relationship turned sour. One day, during
their class, Renato found Francis sitting on his scrapbook. He was angered by what he saw and
Francis explained that he had not intentionally sat down on the scrapbook. It lead to a fistfight.
Suddenly, Renato slipped out of the classroom and went home to get a gun.
When he get back, he looked for Francis and, after several fires, he successfully shot Francis on
the head. Renato proceeded to the faculty room and ordered the teachers and students to lock
the door and close the windows, in effect holding them as hostages.

ISSUE: Whether or not the crime was committed in contempt of or with insult to public
authorities.

HELD: No. We believe the trial court erred in so finding the presence of a generic aggravating
circumstance. Article 152 of the Revised Penal Code, as amended by Republic Act No. 1978 and
Presidential Decree No. 299, provides as follows:

Art. 152. Persons in authority and agents of persons in authority. — Who shall be deemed
as such. — In applying the provisions of the preceding and other articles of this Code, any
person directly vested with jurisdiction, whether as an individual or as a member of some
court or government corporation, board, or commission, shall be deemed a person in
authority. A barrio captain and a barangay chairman shall also be deemed a person in
authority.

A person who by direct provision of law or by election or by appointment by competent


authority, is charged with the maintenance of public order and the protection and security
of life and property, such as a barrio councilman, barrio policeman and barangay leader
and any person who comes to the aid of persons in authority, shall be deemed an agent
of a person in authority.

In applying the provisions of Articles 148 and 151 of this Code, teachers, professors and
persons charged with the supervision of public or duly recognized private schools, colleges
and universities, and lawyers in the actual performance of their professional duties or on
the occasion of such performance, shall be deemed persons in authority. (As amended by
P.D. No. 299, September 19, 1973 and Batas Pambansa Blg. 873, June 12, 1985).

Careful reading of the last paragraph of Article 152 will show that while a teacher or professor of
a public or recognized private school is deemed to be a "person in authority," such teacher or

128
CRIMINAL LAW I – DIGESTED CASES BY: BOAQUINA, MARY CRISCHAEL
129

professor is so deemed only for purposes of application of Articles 148 (direct assault upon a
person in authority), and 151 (resistance and disobedience to a person in authority or the agents
of such person) of the Revised Penal Code. In marked contrast, the first paragraph of Article 152
does not identify specific articles of the Revised Penal Code for the application of which any
person "directly vested with jurisdiction, etc." is deemed "a person in authority." Because a penal
statute is not to be given a longer reach and broader scope than is called for by the ordinary
meaning of the ordinary words used by such statute, to the disadvantage of an accused, we do
not believe that a teacher or professor of a public or recognized private school may be regarded
as a "public authority" within the meaning of paragraph 2 of Article 14 of the Revised Penal Code,
31 the provision the trial court applied in the case at bar.

129
CRIMINAL LAW I – DIGESTED CASES BY: BOAQUINA, MARY CRISCHAEL
130

Dwelling

PEOPLE V. PACITO STO. TOMAS


GR Nos. 40367-69. Aug 22, 1985

FACTS: Pacito asked his wife to go with him, together with their children, and rushed the maid to
get their things packed for Legaspi City. Consolacion Grulla (Pacito’s mother-in-law) disagree with
his plan. So he answered that he was not talking to her but to her wife. While Natividad was
already in her room, she heard a series of gunshots. She went out and saw her mother
motionless. Frightened to death, she ran through the front door, shouting for help. Upon
reaching the corner of Calle Tres Marias, they met Sixto Grulla, a brother of Natividad, who had
also been alarmed by the shots he heard. Sixto went with them to the municipal building, and
on the way, Natividad unfolded to him the tragic incident.

ISSUE: Whether or not the trial court erred in taking into account the aggravating circumstance
of dwelling in the imposition of penalties.

HELD: No. Anent appellant's, submission that the trial court erred in considering dwelling as an
aggravating circumstance, we find the same bereft of any legal support. There is no dispute that
the place where the crimes herein involved were committed is the house of Consolacion Grulla.
It is there where she lives with her daughter, Natividad Grulla (the other victim) and where
Salvacion Grulla was temporarily staying in order to escape from the brutalities of the appellant
brought about by the latter's jealousy. The fact that Salvacion's stay in the said place may be
considered as a temporary sojourn adds no validity to appellant's stance on this point. As we
earlier held in People vs. Galapia, the aggravating circumstance of dwelling is present when the
appellant killed his wife in the house occupied by her other than the conjugal home. Similarly,
dwelling is aggravating where the offended party was raped in a boarding house rented by her.

130
CRIMINAL LAW I – DIGESTED CASES BY: BOAQUINA, MARY CRISCHAEL
131

PEOPLE V. APDUHAN
GR. No. L-19491. August 30, 1968

FACTS: The accused along with five other persons entered the house of the spouses Miano,
shooting Geronimo Miano and Norberto Aton that killed both and took money amounting to Php
322.00 belonging to Geronimo Miano. The accused pleaded not guilty initially and later advise
from counsel Atty. Tirol, pleaded guilty. The judge informed respondent that the penalty imposed
might be death and respondent insisted on pleading guilty with the condition that he be
sentenced to life imprisonment instead of death.

The accused then desisted from his plea of guilt and having it made on record, his counsel
conferred with him and later manifested that respondent will enter the plea of guilty with the
trial court’s ascertainment that he was not forced into pleading guilty. The aggravating
circumstances alleged by the prosecution were 1) band; 2) dwelling; 3) nighttime; and 4) abuse
of superior strength that was withdrawn.

ISSUE: Whether or not the aggravating circumstance of dwelling should be appreciated in this
case.

HELD: Yes. The settled rule is that dwelling is aggravating in robbery with violence or intimidation
of persons, like the offense at bar. The rationale behind this pronouncement is that this class of
robbery could be committed without the necessity of transgressing the sanctity of the home.
Morada is inherent only in crimes which could be committed in no other place than in the house
of another, such as trespass and robbery in an inhabited house. This Court in People vs. Pinca,
citing People vs. Valdez, ruled that the "circumstances (of dwelling and scaling) were certainly
not inherent in the crime committed, because, the crime being robbery with violence or
intimidation against persons (specifically, robbery with homicide) the authors thereof could have
committed it without the necessity of violating or scaling the domicile of their victim." Cuello
Calon opines that the commission of the crime in another's dwelling shows greater perversity in
the accused and produces greater alarm.

131
CRIMINAL LAW I – DIGESTED CASES BY: BOAQUINA, MARY CRISCHAEL
132

PEOPLE V. MONTESA
GR 181899. Novermber 27, 2008

FACTS: AAA went out of the house and watched a "Betamax" movie in the house of a certain
Emmy. She saw appellant and several other persons also watching it. After the show, she went
home arriving therein at around 10:00 p.m. She was alone in the room of their house because
BBB was still in Brgy. Damutan. While she was about to sleep, she saw appellant beside her bed.
Appellant was naked from the waist down to the feet and armed with a 14-inch jagged knife. She
also noticed that the cover of the room’s window was removed. Thereupon, appellant took the
room’s kerosene lamp and blew out the light. Appellant approached her, pointed the knife to her
neck, and warned her not to shout. Appellant soaked his penis with his saliva, removed AAA’s
shorts and panty, and placed himself on top of her. Appellant inserted his penis into her vagina
and made a push and pull movement. AAA felt pain in her vagina. She could not shout for help
because appellant pointed the knife to her neck and threatened to stab her. She tried to free
herself but appellant pinned her down strongly. Later, she felt a fluid in her vagina. Appellant
rested for a while beside her. Thereafter, appellant again placed himself on top of her, inserted
his penis into her vagina and made a pumping motion. Appellant then stood up, wiped his penis,
and warned her not to tell anyone of what happened or he would kill her. Appellant left her and
passed through the room’s window. The same thing happened again on September 21, 1997 at
about 10pm.

AAA went to the house of her classmate and stayed there for 6days because she was afraid that
appellant would rape her again. On Sept 27, 1997, BBB arrived at Maricel’s house to fetch her.
AAA told BBB that appellant raped her. Thereafter, she and BBB went to police to report the
incident.

ISSUE: Whether or not the lower court gravely erred in appreciating the aggravating
circumstance of dwelling thereby imposing the supreme penalty of death.

HELD: No. The information also alleged that appellant raped AAA in the latter’s dwelling and such
circumstance was duly proven during the trial. Under Article 14(3) of the Revised Penal Code,
dwelling is an aggravating circumstance where the crime is committed in the dwelling of the
offended party and the latter has not given provocation. Hence, we have steadfastly held that
dwelling is an aggravating circumstance in the crime of rape.74 Dwelling is considered as an
aggravating circumstance primarily because of the sanctity of privacy the law accords to human
abode.75

Article 63 of the Revised Penal Code provides that if the penalty is composed of two indivisible
penalties, as in this case, and there is one aggravating circumstance, the greater penalty shall be
applied. Since the aggravating circumstance of dwelling was present in these cases, the penalty
of death should be imposed on appellant.

132
CRIMINAL LAW I – DIGESTED CASES BY: BOAQUINA, MARY CRISCHAEL
133

PEOPLE V. GAYETA
GR No. 171564. December 17, 2008

FACTS: The spouses Benjamin and Conchita were drinking tuba when two armed men barged
into their house. One of the armen men, was Reano and the other was the appellant. The duo
announced a hold-up and ordered the spouses to lie down on the floor. When the duo fled, the
spouses Nicer reported the incident to the barangay officials who immediately sought police
assistance.

Meanwhile, spouses BBB and AAA were watching television in their living room when two armed
men, also later identified as Reano and appellant, entered their house. They likewise ordered the
spouses to lie down and asked them to produce their money. BBB asked AAA to get the money
from their store, which was located some twenty (20) meters away from their house. Appellant
accompanied AAA to the store while Reano stayed with BBB. Upon reaching the store, AAA gave
the money to appellant and the latter forcibly had sexual intercourse with AAA.

ISSUE: Whether or not the aggravating circumstance of dwelling is present.

HELD: Yes. The CA correctly appreciated the aggravating circumstance of dwelling. When the
crime is committed in the dwelling of the offended party and the latter has not given provocation,
dwelling may be appreciated as an aggravating circumstance. Applying Article 63(1) of the
Revised Penal Code, the penalty of death is rightfully imposed in Criminal Case No. P-5420.
However, pursuant to Republic Act (R.A.) No. 9346, the penalty of death should be commuted to
reclusion perpetua with no eligibility for parole.

133
CRIMINAL LAW I – DIGESTED CASES BY: BOAQUINA, MARY CRISCHAEL
134

Nighttime

PEOPLE V. GARCIA
GR No. L-30449. October 31, 1979

FACTS: Corazon’s husband informed her that he saw her brother Apolonio engaged in a drinking
spree with his gang in front of Bill’s Place at M. de la Cruz Street, Pasay City. Corazon obtained
permission to leave the house at 3:00am so she could fetch her brother. She went to fetch him
because she wanted him to escape the untoward influence of his gang. In explaining the rationale
for her noctural mission, she employed in her sworn statement the following language: "Dahil
itong si Junior ay meron na kaming nabalitaan na naaakay ng barkada niya sa paggawa ng hindi
mabuti." On her way, Corazon saw her brother fleeing a group of about seven persons, including
the two accused. About 20 meters away, she saw the group catch up with her brother and
maltreat him. Some beat him with pieces of wood, others boxed him. Immediately afterwards,
the group scampered away in different directions. Antonio was left behind. He was sitting astride
the prostrate figure of Apolonio, stabbing the latter in the back with his long knife. Corazon was
not able to observe where Antonio later fled, for she could hardly bear to witness the scene.

ISSUE: Whether or not nighttime was purposely sought to insure the execution of the crime.

HELD: No. There is no showing that the accused purposely sought the cover of night time. The
offense took place at 3:00 o'clock in the morning. It may therefore be said that it was committed
at night, which covers the period from sunset to sunrise, according to the New Civil Code, Article
13. Is this basis for finding that nocturnity is aggravating? The Revised Penal Code, Article 14,
provides that it is an aggravating circumstance when the crime is committed in the nighttime,
whenever nocturnity may facilitate the commission of the offense. There are two tests for
nocturnity as an aggravating circumstance: the objective test, under which nocturnity is
aggravating because it facilitates the commission of the offense; and the subjective test, under
which nocturnity is aggravating because it was purposely sought by the offender. These two tests
should be applied in the alternative.

In this case, the subjective test is not passed because there is no showing that the accused
purposely sought the cover of night time. Next, we proceed and apply the objective test, to
determine whether nocturnity facilitated the killing of the victim. A group of men were engaged
in a drinking spree, in the course of which one of them fled, chased by seven others. The criminal
assault on the victim at 3:00 a.m. was invited by nocturnal cover, which handicapped the view of
eyewitnesses and encouraged impunity by persuading the malefactors that it would be difficult
to determine their Identity because of the darkness and the relative scarcity of people in the
streets. There circumstances combine to pass the objective test, and e find that nocturnity is
aggravating because it facilitated the commission of the offense. Nocturnity enticed those with
the lust to kill to follow their impulses with the false courage born out of the belief that they
could not be readily Identified.

134
CRIMINAL LAW I – DIGESTED CASES BY: BOAQUINA, MARY CRISCHAEL
135

Evident Premeditation

US V. MANALINDE
GR No. L-5292. August 28, 1909

FACTS: Juan Igual, a Spaniard, was seated on a chair in the doorway of Sousa's store in Cotabato,
Moro Province, he suddenly received a wound on the head delivered from behind and inflicted
with a kris. Ricardo Doroteo, a clerk in the said store, who was standing behind the counter, upon
hearing the noise and the cry of the wounded man, ran to his assistance and found him lying on
the ground. Meanwhile the aggressor, the Moro Manalinde, approached a Chinaman named
Choa, who was passing along the street, and just as the latter was putting down his load in front
of the door of a store and was about to enter, attacked him with the same weapon, inflicting a
severe wound in the left shoulder, on account of which he fell to the ground. The Moro, who
came from the rancheria of Dupit and had entered the town carrying his weapon wrapped up in
banana leaves, in the meantime escaped by running away from the town. Both wounded men,
the Chinaman and the Spaniard, were taken to the hospital, where the former died within an
hour, the record not stating the result of the wound inflicted on the Spaniard Juan Igual.

ISSUE: Whether or not circumstance of premeditation should be considered when the crime is
committed upon offer money, reward or promise.

HELD: Yes. The fact that the arrangement between the instigator and the tool considered the
killing of unknown persons, the first encountered, does not bar the consideration of the
circumstance of premeditation. The nature and the circumstances which characterize the crime,
the perversity of the culprit, and the material and moral injury are the same, and the fact that
the victim was not predetermined does not affect nor alter the nature of the crime. Even though
in a crime committed upon offer of money, reward or promise, premeditation is sometimes
present, the latter not being inherent in the former, and there existing no incompatibility
between the two, premeditation can not necessarily be considered as included merely because
an offer of money, reward or promise was made, for the latter might have existed without the
former, the one being independent of the other. In the present case there can be no doubt that
after the crime was agreed upon by means of a promise of reward, the criminal by his subsequent
conduct showed a persistency and firm intent in his plan to carry out the crime which he
intentionally agreed to execute, it being immaterial whether Datto Mupuck did or did not
conceive the crime, once Manalinde obeyed the inducement and voluntarily executed it.

135
CRIMINAL LAW I – DIGESTED CASES BY: BOAQUINA, MARY CRISCHAEL
136

PEOPLE V. DISCALSOTA
GR No. 136892. April 11, 2002

FACTS: The victim, along with his friends decided to visit their friend, ‘Novieboy’ del Rosario. They
were welcomed by the latter. While peacefully, enjoying themselves, they were suddenly startled
by shouts coming from a group of men outside the house. The leader of the said group was
shouting: ua kamo dira kay pamatyon ta kamo! (You there, get out and we will kill you!) The four
(4) teen-agers were terrified since they did not know the men who were threatening them. Nor
did they know of any grudge or misunderstanding between their group and the men outside.
They called Mrs. Del Rosario who advised them not to go outside and called for the police.

The group was escorted out of the house by the tanods towards the footpath leading to the main
road. The men threatening them were still outside when they went out of the house and they
followed the group. The four boarded the pedicab outside. The pedicab had not left when Rowell
saw a man running towards them from the footwalk. Despite efforts by the barangay tanods to
stop him, the man rushed headlong towards Rowell and the victim. While the victim was running
away trying to escape, the man holding the knife caught up with him and thrust his knife at the
fleeing victim who was hit at the back.

ISSUE: Whether or not the evident premeditation can be appreciated in this case.

HELD: No. Where no sufficient lapse of time is appreciable from the determination to commit
the crime until its execution, evident premeditation cannot be appreciated. It is settled that
qualifying circumstances cannot be presumed, but must be established by clear and convincing
evidence as conclusively as the killing itself.

In this case, the first two elements of evident premeditation are present. As found by the RTC,
the time appellant determined to commit the crime was when he started shouting at the victim
and the latter's companions: "You, there, get out and we will kill you!" By staying outside the
house and following the victim's companions when they came out, he manifestly indicated that
he clung to his determination.

As for the third element, the prosecution evidence shows that appellant started shouting outside
Mrs. del Rosario's house at 3:30 p.m. When the victim's group left the house, it was not yet dark;
it was only past four o'clock in the afternoon.The police received information on the stabbing
incident at 4:30 p.m. on the same day. It took less than an hour from the time appellant evinced
a desire to commit the crime, as manifested by his shouts outside the house, up to the time he
stabbed the victim. The span of less than one hour could not have afforded the former full
opportunity for meditation and reflection on the consequences of the crime he committed.

136
CRIMINAL LAW I – DIGESTED CASES BY: BOAQUINA, MARY CRISCHAEL
137

The essence of premeditation is that the execution of the criminal act must be preceded by cool
thought and reflection on the resolution to carry out the criminal intent during a space of time
sufficient to arrive at a calm judgment.

"To justify the inference of deliberate premeditation, there must be a period sufficient in
a judicial sense to afford full opportunity for meditation and reflection and to allow the
conscience of the actor to overcome the resolution of his will if he desires to hearken to
its warning."

137
CRIMINAL LAW I – DIGESTED CASES BY: BOAQUINA, MARY CRISCHAEL
138

PEOPLE V. DE GUZMAN
GR No. 173477. February 4, 2009

FACTS: Nipales, a pedicab driver transported the appellant to Brgy. Guiguilonen, Mangaldan.
While on the way, he heard the appellant instruct someone on the phone to remove the plate
number of a certain motorcycle as they would execute someone. The appellant went down and
asked him to leave. When Nipales heard a gunshot, he moved his vehicle fast.

Malanum, who testified that he and the appellant were employed in the hardware business of
the victim, drove the victim in a tricycle to Mangaldan. While on their way, at the Embarcadero
Bridge, they saw two persons riding in tandem on a motorcycle. The back rider shot them three
times. Malanum recognized the appellant as the person who fired at them. Dr. Fidelito Manaois
(victim) was hit by the first volley of gunfire.

The prosecution also presented the victim’s daughter. She testified that she was married to the
appellant and they both lived with her father, Dr. Fidelito Manaois. The appellant however had
irreconcilable differences with her father. She said that the appellant had been threatening her
and her family serious harm, and even death, before the incident. The appellant did not deny
killing her father albeit he retorted that she was the one he planned to kill.

ISSUE: Whether or not the trial court committed error when it appreciated the presence of
evident premeditation despite the failure of the prosecution to present evidence that the killing
was aggravated by said aggravating circumstance.

HELD: Yes. For evident premeditation to be appreciated, there must be proof, as clear as the
evidence of the crime itself, of the following elements: (1) the time when the offender
determined to commit the crime; (2) an act manifestly indicating that the accused clung to his
determination; and (3) a sufficient lapse of time between determination and execution to allow
himself time to reflect upon the consequences of his act.

In this case, there is no showing of the specific time when the crime was executed. The evidence
showing (1) that the appellant said over his cellphone, at around 7:30 p.m. on November 14,
2003, that they were going to hit somebody, and (2) that sometime that same evening the crime
was committed, fails to prove beyond reasonable doubt that sufficient time had elapsed to allow
appellant time to reflect and that he clung to his determination to kill the victim. Hence, evident
premeditation was not proven in this case.

138
CRIMINAL LAW I – DIGESTED CASES BY: BOAQUINA, MARY CRISCHAEL
139

Treachery

US V. BALUYOT
GR No. L-14476. November 6, 1919

FACTS: The accused Baluyot was one of the competitors of the victim Conrado Lerma in the
general elections for governor of the Province of Bataan.

An assault was begun suddenly and unexpectedly by the firing of a pistol by the accused at
Governor Lerma, who was unarmed that time. After the second shot was fired, Governor Lerma
continued his flight along the corridor and, instead of attempting to pass out to the right into the
recorder's office, which would have exposed him to the danger of another shot while passing
through the open space, he took refuge in a closet at the end of the corridor. Once within, he
shut the door and placed himself in a position to obstruct the entrance of his pursuer. The
governor began to call aloud for help. The accused then tried to force open the door but was
unable to do so, owing to the resistance of the deceased from within. The accused, however,
judging the position of the deceased from the cries emitted, fired in the direction. The bullet
passed through the panel of the door and entered the head of the deceased which later on lead
to the latter’s death.

ISSUE: Whether or not treachery attended the commission of the crime.

HELD: Yes. With reference to the manner in which the attack was begun, the proof shows that
access was gained by Baluyot, to the governor's office upon the pretext that he desired a friendly
interview; and although the strained relations existing between the two, owing to their political
antagonisms, was appreciated by both, there was nothing in the situation to warn the governor
of impending trouble. The fact Baluyot had already been called into the office upon the
governor's first arrival and had withdrawn for a few moments to permit another person to have
an interview was also calculated to put the governor off his guard at the moment Baluyot
reentered the office. Being seated in a reclining chair, and hemmed in by obstacles which
prevented him from reaching his assailant, it is plain that the unarmed governor could make no
effectual defense against a person armed with such a deadly weapon as a revolver. It is obvious
also that the means and methods thus deliberately selected by the assailant were intended to
insure the execution of the crime without any risk to himself arising from the defense which the
offended party could make.

139
CRIMINAL LAW I – DIGESTED CASES BY: BOAQUINA, MARY CRISCHAEL
140

PEOPLE V. ESCOTE, JR.


GR No. 140756. April 4, 2003

FACTS: Rodolfo, a bus driver, drove the bus from its terminal to its destination in Bolinao,
Pangasinan. Also on board was Romulo, the conductor of the bus, as well as some passengers
including the deceased SPO1 Jose Manio Jr. At Camachile, Balintawak, 6 passengers boarded the
bus including the accused.

When the bus was traveling along the highway in Plaridel, Bulacan, Juan and Victor (two accused)
announced a holdup. Juan fired his gun upward to awaken and scare off the passengers. The
felons get the passengers’ money and valuables. Then, they went to Manio, Jr. was seated and
demanded his ID card and wallet and took it as well as his service gun. The accused told Manio
Jr. that they are going to shot him. The police officer pleaded for mercy but the accused ignored
the plea and shot him. Victor and Juan told Rodolfo to continue driving the bus and not report
the incident along the way. The accused alighted from the bus in Mexico, Pampanga. The robbery
was over in 25 minutes. When the bus reached Mabalacat, Pampanga, Rodolfo and Romulo
reported the incident to the police authorities.

ISSUE: Whether or not treachery was attended in the commission of the crime.

HELD: Yes. There is treachery when the following essential elements are present, viz: (a) at the
time of the attack, the victim was not in a position to defend himself; and (b) the accused
consciously and deliberately adopted the particular means, methods or forms of attack employed
by him.52 The essence of treachery is the sudden and unexpected attack by an aggressor on the
unsuspecting victim, depriving the latter of any chance to defend himself and thereby ensuring
its commission without risk of himself. Treachery may also be appreciated even if the victim was
warned of the danger to his life where he was defenseless and unable to flee at the time of the
infliction of the coup de grace.53 In the case at bar, the victim suffered six wounds, one on the
mouth, another on the right ear, one on the shoulder, another on the right breast, one on the
upper right cornea of the sternum and one above the right iliac crest. Juan and Victor were armed
with handguns. They first disarmed SPO1 Manio, Jr. and then shot him even as he pleaded for
dear life. When the victim was shot, he was defenseless. He was shot at close range, thus insuring
his death. The victim was on his way to rejoin his family after a hard day's work. Instead, he was
mercilessly shot to death, leaving his family in grief for his untimely demise. The killing is a grim
example of the utter inhumanity of man to his fellowmen.

140
CRIMINAL LAW I – DIGESTED CASES BY: BOAQUINA, MARY CRISCHAEL
141

PEOPLE V.CARATAO
GR No. 126281. June 10, 2003

FACTS: The appellant and his wife entered the commissary canteen of NALCO. Appellant’s wife
approached Sugala, a rice dispatching checker also employed at NALCO, and told her that her
husband was very angry because he (appellant Caratao) was not given additional rice by the
victim Edgardo “Tado” Bulawin, NALCO’s rice vale issuer.

Shortly thereafter, Sugala saw the victim walk out of the canteen. Sensing that appellant was
about to rush to the victim, Sugala restrained him and assured him that he would give him
additional 25 kilos. After this, he noticed that appellant was no longer near him. He next saw
appellant standing behind the victim. At that time, the left hand of Caratao was on the shoulder
of Bulawin and his right side was thrusting at the side of Bulawan which retracted very fast and
Sugala saw that that hand was holding a knife.

ISSUE: Whether or not the lower court erred in holding that accused committed murder by
treachery.

HELD: Yes. Treachery is present when two conditions concur, namely: (1) that the means,
methods and forms of execution employed gave the person attacked no opportunity to defend
himself or to retaliate; and (2) that such means, methods and forms of execution were
deliberately and consciously adopted by the accused without danger to his person. In the case at
bar, the first element was established by the fact that appellant suddenly attacked from behind
the unsuspecting and unarmed victim who was then astride his motorcycle. However, we find
the prosecution’s evidence insufficient to sustain the finding of the presence of the second
element, namely, that appellant deliberately adopted the mode of attack.

Repeatedly upheld has been the rule that chance encounters, impulse killing or crimes
committed at the spur of the moment, or those that were preceded by heated altercations are
generally not attended by treachery, for lack of opportunity of the accused deliberately to
employ a treacherous mode of attack. In the present case, it appears from the evidence that
appellant’s grudge against the victim was brought about only moments before the attack, when
the latter ignored his repeated pleas for rice. As observed by the trial court, the sight of the victim
leaving the compound without heeding appellant’s request must have worsened his anger. In his
testimony, appellant admitted that at that moment, he "forgot himself." Further, he explained
that it was then customary for him to bring a knife for his own safety, in defense against lawless
elements in their area at the time. It was thus only by chance and not by plan that he attacked
the victim the way he did. The stabbing was evidently a result of a rash and impetuous impulse
of the moment arising from what appellant perceived to be an unjust act of the victim, rather
than from a deliberated action. Hence, as the killing was done at the spur of the moment,
treachery cannot be appreciated.

141
CRIMINAL LAW I – DIGESTED CASES BY: BOAQUINA, MARY CRISCHAEL
142

PEOPLE V. SITCHON
GR No. 134362. February 27, 2002

FACTS: The appellant admitted killing the 2-year-old victim, the son of his live-in partner. He
claimed that he killed the boy only because he was under the influence of shabu, marijuana and
Valium 10 at that time. Appellant came upon Macky, the victim, playing with his feces scattering
them. He scolded the boy. Appellant got hold of Macky but the boy struggled to free himself from
appellant’s grasp. Appellant, still reeling from the Valium 10 he had just taken, became so angry
that he picked up a broom with a wooden handle, and hit the boy. Appellant did not realize that
he had hit Macky hard until he saw the boy sprawled on the floor, breathing with difficulty. He
dressed Macky and brought him to the Galang Medical Center at the corner of Abad Santos
Avenue and Tayabas Street, Manila. Macky was immediately attended by a doctor but, was later
on, announced dead.

ISSUE: Whether or not the killing in this case was attended by treachery.

HELD: Yes. There is treachery when the offender commits any of the crimes against persons,
employing means, methods or forms in the execution thereof which tend directly and especially
to insure its execution without risk to himself arising from the defense which the offended party
might make. It is beyond dispute that the killing of minor children who, by reason of their tender
years, could not be expected to put up a defense, is treacherous.

142
CRIMINAL LAW I – DIGESTED CASES BY: BOAQUINA, MARY CRISCHAEL
143

PEOPLE V. ANCHETA
GR No. 138306-07. December 21, 2001

FACTS: The accused SPO1 Ancheta was sleeping at home when he was awakened by the sound
of someon banging on his door. After a brief silence, he heard someone say: “Pare buksan mo
ito.” The accused took his gun under his pillow and ordered the person to identify himself. But
the stranger kept on banging the door. When the accused opened the door, Jonathan Aromin
immediately noticed that SPO1 Ancheta was armed with a gun. Intimidated, Jonathan began to
move away. As he left the house of the accused, Jonathan suddenly heard two (2) shots which
prompted him to hide behind the nearest wall. But when he looked back the accused SPO1
Ancheta was already aiming his revolver directly at his face and without hesitation shot him at
close range. Stunned by the gunshot wound, Jonathan momentarily blacked out but soon
regained consciousness when his neighbor, Leonila Lopez, came to his aid and rushed him to the
Jose Reyes Memorial Medical Center.

In his defense, the accused claimed that When he finally opened the door, he saw his brother
Julian Ancheta and his neighbor Jonathan Aromin. Upon seeing them, he inquired as to why his
brother addressed him as "pare" but instead of answering, Julian Ancheta angrily asked him why
he was holding a gun. To appease his brother, the accused lowered his pistol and explained that
the gun was only for protection as he had no idea who was banging his door in the middle of the
night. He then invited them into the house, but when he turned around his brother suddenly
grabbed his hand from behind to disarm him. As they grappled, the gun accidentally fired twice
and the next thing he saw was his brother sprawled on the ground and Jonathan Aromin was
nowhere to be found. He never knew what actually happened to Jonathan Aromin as his back
was turned against him when the gun went off.

ISSUE: Whether or not the killing of Julian Ancheta and the shooting of Jonathan Aromin were
qualified by treachery.

HELD: No. While it was established that accused-appellant intentionally shot his brother Julian,
the witnesses never saw how the killing started. Treachery cannot be considered where the
witnesses did not see the commencement of the assault and the importance of such testimonies
cannot be overemphasized considering that treachery cannot be presumed nor established from
mere suppositions. And where no particulars are shown as to the manner by which the
aggression was commenced or how the act which resulted in the death of the victim began and
developed, treachery can in no way be established. Hence, without the existence of treachery
accused-appellant can only be convicted of homicide.

Neither was treachery established in the shooting of Jonathan Aromin. Two (2) conditions must
concur for treachery to exist, namely: (a) the employment of means of execution that gave the
person attacked no opportunity to defend himself or to retaliate; and, (b) the means or method

143
CRIMINAL LAW I – DIGESTED CASES BY: BOAQUINA, MARY CRISCHAEL
144

of execution was deliberately or consciously adopted. Both these circumstances must be proved
as indubitably as the crime itself.

In the case at bar, however, there is no sufficient proof to establish with certainty that accused-
appellant deliberately and consciously adopted the means of executing the crime against
Jonathan Aromin. Furthermore, the victim was already aware of the danger as he saw accused-
appellant carrying a gun and heard two (2) gunshots prompting him to run and hide behind a
wall. Thus, there could be no treachery since prior to the attack the victim was forewarned of the
danger to his life and even managed to flee, albeit unsuccessfully. Consequently, accused-
appellant can only be convicted of frustrated homicide.

144
CRIMINAL LAW I – DIGESTED CASES BY: BOAQUINA, MARY CRISCHAEL
145

Ignominy

PEOPLE V. ALFANTA
320 SCRA 357

FACTS: The victim, Nita Fernandez, was asleep in the residence of a friend when at around 12
midnight, a man she had not seen before suddenly entered the house, boxed her jaw and covered
her mouth with his hand. He was pointing a bolo at her and threatened to kill her if she will resist.
Thereafter, she was taken and brought to a vacant house where the stranger succeeded in having
carnal knowledge of her. After the first intercourse, she was ordered to lie face down while the
man sodomized her. Not satisfied, the accused then inserted his finger inside her. Thereafter, the
man lay down beside her and again threatened to kill her. After a while, Nita noticed that the
man was asleep, she then stabbed the man with the knife and hacked him with the bolo when
the former broke. She was able to escape and go to the authorities, who apprehended the man
later on identified as the accused.

ISSUE: Whether the crime of rape should be aggravated by nighttime. Whether there was
ignominy when the accused forced the victim to engage in anal sex.

RULING: The law defines night as being from sunset to sunrise. By and itself, nighttime would not
be an aggravating circumstance unless it is specially sought by the offender or taken advantage
of by him, or it facilitated the commission of the crime by insuring the offender’s immunity from
capture. In the present case, the accused abducted the victim, brought her to an abandoned,
unlit house and then unleashed his carnal desire on her, assured of the stillness of a sleeping
world. With respect to ignominy, Art 14 par 17 of the RPC considers to be aggravating any means
employed or circumstance that adds disgrace and obloquy to the material injury caused by the
crime. The case of People vs Saylan is applicable. In this case wherein the accused entered the
victim from behind, the offender claimed that there was no ignominy because the studies of
experts have shown that the position is not novel and has been resorted to by couples in the act
of copulation. This may well be true if the sexual act is performed by consenting partners but not
otherwise.

145
CRIMINAL LAW I – DIGESTED CASES BY: BOAQUINA, MARY CRISCHAEL
146

PEOPLE V. DIZON
GR No. 134802. October 26, 2001

FACTS: Arlie was walking along the vicinity of Roosevelt Avenue, Quezon City while accused
suddenly seized her, pointing a fan knife to the side of her neck, and announced a holdup. They
went to a dark and empty basketball court. Accused kissed, mashed her breast and bit her nipples
three times as well as her vagina. After satisfying himself, accused ordered the victim to hold his
penis and insert it in his mouth, forcing her to admire his bolitas.

ISSUE: Whether or not the acts of the accused constitutes cruelty.

RULING: Yes. Accused subjected her to dehumanizing indignities. All the wrongs committed by
the accused were no longer necessary insofar as accused purpose of raping the victim was
concerned. By subjecting her to these unwarranted physical abuses on top of rapping her,
accused deliberately and inhumanly augmented her pain and sufferings.

146
CRIMINAL LAW I – DIGESTED CASES BY: BOAQUINA, MARY CRISCHAEL
147

PEOPLE V. REGALARIO
GR No. 174483. March 31, 2009

FACTS: A contest was being held in barangay when Rolando and his friend were enjoying thereat.
Accused approached them but the two distanced themselves. Nevertheless, a commotion
ensued and the accused took turns in hitting the victim.

ISSUE: Whether or not superior strength is present in the instant case.

RULING: Yes, superior strength is present. Accused used force out of proportion to the means
available to the victim. Since accused were armed of ‘night sticks’, paragraph 15 of Article 14 or
the Revised Penal Code can be taken into account.

Article 15: Alternative Circumstances


Nature and Kinds

PEOPLE V. ABELLO
GR No. 151952. March 25, 2009

FACTS: Accused is convicted under Republic Act 7160 or “Child abuse law” for sexually abusing
his step-daughter by putting his penis inside her mouth and mashing her breast while AAA is
sleeping.

ISSUE: Whether or not relationship aggravated the crime.

RULING: Yes. Under Article 15 of the Revised Penal Code, relationship as an alternative
circumstance is an aggravating circumstance in crimes against chastity and in rape.

PEOPLE V. CEBALLOS
GR No. 169642. September 14, 2007

FACTS: Accused is the father of a 14-year old victim who have sexual intercourse with his
daughter. He mashed the breast and inserted his finger in the vagina of the victim.

147
CRIMINAL LAW I – DIGESTED CASES BY: BOAQUINA, MARY CRISCHAEL
148

ISSUE: Whether or not relationship aggravated the crime.

RULING: Yes. Under Article 15 of the Revised Penal Code, relationship as an alternative
circumstance is an aggravating circumstance in crimes against chastity and in rape.

Article 17: Principals


Conspiracy

PEOPLE V. AGULOS
GR No. 121828. June 27, 2003

FACTS: Elisa Roldan was inside their store waiting for her husband to arrive. Joselito and Julian
are drinking beer. Although already drunk, Aguilos and Lagliba joined them. Aguilos had a heated
argument with Julian. Elisa pacified Aguilos and advised them to go home. Aguilos, then, punched
Julian. As Joselito tried to stop the fight, Lagliba stabbed him. In trial, there were inconsistencies
as to who stabbed the victim.

ISSUE: Whether or not conspiracy exists in the case.

RULING: Yes. The identity of the person who stabbed the victim does not negate the principle of
conspiracy. Conspiracy, under Article 8 of the Revised Penal Code, is when two or more persons
come to an agreement to commit felony and decide to commit it.

PEOPLE V. FEDERICO
247 SCRA 246

FACTS: On June 24, 1954, Federico Geronimo, et al. were charged with the complex crime of
rebellion with murders, robberies, and kidnapping. These are the ranking officers/ or members
of CCP and Huks. In the information it alleged 5 instances including an ambush on Mrs. Aurora
Quezon’s convoy on April 28, 1949 and ending on February 1954 where Geronimo killed
Policarpio Tipay a Barrio Lieutenant. He pleaded guilty to the accusation and the trial court found
him guilty of the complex crime of rebellion with murders, robberies, and kidnappings,
sentencing him to reclusion perpetua. He appealed raising the sole question of whether the crime
committed by him is not the complex crime of rebellion, but simply rebellion, thus punishable
only by prision mayor.

ISSUE: Can rebellion be complexed with murder, robbery or kidnapping?

148
CRIMINAL LAW I – DIGESTED CASES BY: BOAQUINA, MARY CRISCHAEL
149

RULING: No. Even if the crime is not committed in furtherance of rebellion, without political
motivation, the crime would be separately punishable and would not be absorbed in rebellion.
According to the Hernandez resolution, the complexing of rebellion will lead to undesirable
results. It cannot be taken with rebellion to constitute a complex crime, for the constitutive acts
and intent would be unrelated to each other. He would be held liable for separate crimes, and
these cannot be merged into a juridical whole.

Principals by Inducement

U.S. V. INDANAN
24 Phil 203

FACTS: Panglima Indanan ordered the killing of Sariol to his men Akiran, Kalyakan and Suhuri in
the Chinese cemetery. Indanan had an order to that effect from the Governor.

ISSUE: Whether or not Indanan is guilty of murder by inducement.

RULING: Yes. Article 17, paragraph 2 of the Revised Penal Code declares those to be principals in
a crime who directly force or induce others to commit it as principals. Indanan force another to
commit a crime who physically by actual force or grave fear to another to commit the crime. He
who suffers violent acts w/o will & against his will, is no more than an instrument, & therefore is
guilty of no wrong. The real culprits in such case, the only guilty persons, are those who use the
violence, those who force the other to commit the crime. One is induced directly to commit a
crime either by command, or for a consideration, or by any other similar act w/c constitutes the
real & moving cause of the crime & w/c was done for the purpose of inducing such criminal act
& was sufficient for that purpose.

PEOPLE V. KIICHI OMINE


61 Phil 609

FACTS: Defendants appeal from a decision of the CFI finding them guilty of frustrated
homicide, w/ the AC that advantage was taken of their superior strength & sentencing them each
to suffer an IS from 6 yrs of prision correccional to 12 yrs of prision mayor. Defendants Eduardo
Autor, Luis Ladion and Agapito Cortesano were working under co-defendant Kiichi Omine, the
overseer or manager of the hemp plantation owned by Angel Pulido. The 4 defendants lived
together in a house on the plantation. Kiichi Omine asked Angel Pulido permission to open a new
road through the plantation. Acdg to Omine, Pulido did give his permission that’s why he began
working on the new road. But acdg to Pulido, he refused to grant this request because there was
already an unfinished road. As Pulido and his son along w/ 2 others were returning home from a
cockpit, they noticed that a considerable number of hemp plants were destroyed by
the construction of the new road. Angered by this, they went to the defendants’ house and there

149
CRIMINAL LAW I – DIGESTED CASES BY: BOAQUINA, MARY CRISCHAEL
150

happened a violent altercation resulting to the owner Pulido’s death from a wound by a bolo
struck in his breast.

ISSUE: Whether or not Kiichi Omine is guilty of principal by inducement.

RULING: Although it is alleged that Kiichi Omine uttered words of inducement toE duardo Autor,
it would be insufficient to make him a principal by induction. Eduardo Autor though
working under the direction of Omine was still being paid by Pulido. Moreover, it is necessary
that inducement be made directly w/ the intention of procuring the commission of the
crime and that such inducement be the determining cause of the commission of the crime.
It must be precede the act induced and must be so influential in producing the criminal act that
w/o it the act wouldn’t have been performed. Moreover, as words of direct inducement, it is
essential that such advice or words have great dominance and great influence over the person
who acts, that they be as direct, as efficacious, as powerful as physical or moral coercion or as
violence itself. Hence, the 3 co-defendants of Autor are not responsible for the injury inflicted by
him on Angel Pulido.

Principals by Indispensable Cooperation

PEOPLE V. MONTEALEGRE
GR No. 67948. May 31, 1988

FACTS: Abadilla was eating at a restaurant when he detected the smell of marijuana smoke
coming from a nearby table. Intending to call a policeman, he went outside and saw a police and
reported the matter. The police approached the table and held Montealgre and Capalad. Capalad
suddenly pulled out his knife and started stabbing the police at the back. The police released the
2 in order to draw his gun but Montealegre restrained the police so that Capalad may continue
stabbing. The 3 grappled and the police was able to draw his gun and fired at the 2 assailants. A
chase ensued. Capalad was shot which resulted to his death. The police also died because of the
wounds inflicted by Capalad.

ISSUE: Can Montealegre be considered as principal?

RULING: The accused was correctly considered a co-principal for having collaborated with
Capalad in the killing of the police officer. The 2 acted in concert. Even if the accused did not
himself commit the act of stabbing, he is nonetheless equally guilty thereof for having prevented
the police from resisting the attack against him.

U.S. V. LIM BUANCO


14 Phil 472

150
CRIMINAL LAW I – DIGESTED CASES BY: BOAQUINA, MARY CRISCHAEL
151

FACTS: Lim Buanco drew a check on the Banco Español-Filipino for the sum of 1,000 pesos, and
this check was through the agency of another bank in which it was deposited by Lim Buanco,
presented in due course of business to and paid by the Banco Español-Filipino. Before the check
was thus paid, Reyes, acting in his capacity as an employee of the bank, indorsed thereon the
words "Corriente, P. O. Luciano de los Reyes," although at the time this indorsement was made,
Lim Buanco had no actual credit balance in the bank, and no permission had been given him by
any officer or officers in authority of said bank to overdraw his account. In this manner the
defendants Lim Buanco and Reyes, in furtherance of the conspiracy to cheat, deceive, and
defraud the bank, secured the payment of said check, although they both knew at the time that
the defendant Lim Buanco had no credit balance in said bank, but was in fact indebted to the
bank in the sum of more than 300,000 pesos, which had previously been withdrawn from the
bank by means of similar checks drawn by Lim Buanco, and fraudulently indorsed as correct by
Reyes. The amount of money thus fraudulently obtained from the bank by means of the check as
aforesaid, with interest thereon at the rate of 6 per cent per annum from the date of the check
to the date of the decision in the court below, to wit, January 16, 1909, amounted to the sum of
P1,635.50, Philippine currency, equivalent to 5,682.50 pesetas, which the court determined to
be the damage occasioned to and suffered by the bank by reason of the aforesaid fraudulent
conduct of the defendants acting together in furtherance of said conspiracy. No part of the said
sum has been returned or paid back to the Banco Español-Filipino by Lim Buanco, or by Reyes, or
by any person acting for or in his or their behalf.

ISSUE: Is there conspiracy?

RULING: No, all the accused are not co-principals. Where several acts are done in pursuance of a
conspiracy, each act being distinct from the other, the fact that they are in fact done in pursuance
of a conspiracy does not make one act the "same offense" as the other. While the conspiracy
may be single, and therefore subject to one indictment only, yet the felonies accomplished by
means of the conspiracy were separate and distinct, depending upon the different acts, provable
by different evidence, and accomplished by distinct though similar means.

Article 18: Accomplices


PEOPLE V. DOCTOLERO
G.R. No. 34386. February 7, 1991

FACTS: Conrado and Virgilio were throwing stones at the house of Marcial Sagun. While throwing
stones, Ludovico went inside the house then hacked the victims.

ISSUE: Whether or not Conrado and Virgilio Doctolero participated as accomplices.

151
CRIMINAL LAW I – DIGESTED CASES BY: BOAQUINA, MARY CRISCHAEL
152

HELD: The lower court held that Conrado and Virgilio participated as accomplices in the slaying
of the women and the infliction of injuries on the child. It is obvious that they themselves knew
what was going on inside the room of the house at the time, but they just stood by and did
nothing to stop their brother Ludovico Doctolero from brutally hacking his women victims to
death. It is, therefore, reasonable to believe that the two appellants, Conrado and Virgilio, merely
stood by as their brother Ludovico Doctolero was murdering the two deceased women, ready to
lend assistance. Indeed, there is no question that the presence of these two appellants upstairs
in the house of Marcial Sagun gave their brother Ludovico Doctolero the encouragement and
reliance to proceed as he did proceed, in committing the heinous crimes against two defenseless
women and a child.

We have held that where one goes with the principals, and in staying outside of the house while
the others went inside to rob and kill the victim, the former effectively supplied the criminals
with material and moral aid, making him guilty as an accomplice.
Appellants contend that the murders occurred as a consequence of a sudden thought or impulse,
thus negating a common criminal design in their minds. This pretension must be rejected since
one can be an accomplice even if he did not know of the actual crime intended by the principal
provided he was aware that it was an illicit act.

There being ample evidence of their criminal participation, but a doubt exists on the nature of
their liability, with no evidence of conspiracy among the appellants having been shown, the
courts should favor the milder form of liability or responsibility which is that of being mere
accomplices

PEOPLE OF THE PHILIPPINES vs FERNANDO WATIMAR


G.R. Nos. 121651-52. August 16, 2000

FACTS: Accused-appellant raped his daughter in their own house once on March 1990 and on
November 1992 with lewd design and at the point of a knife and threat to kill. Accused contended
that it was impossible to commit the crime in such place, that the victim nevertheless did not do
everything in her power to prevent the assault on her virtue, that there was no medical finding
or examination which would directly contribute to establish that rape was indeed committed,
and that it took her 3 years to file a complaint.

ISSUE: Whether or not the defendant is liable.

HELD: The court held that, first, rape does not necessarily have to be committed in an isolated
place and can in fact be committed in places which to many would appear to be unlikely and
high-risk venues for sexual advances.
Second, the law does not impose upon a rape victim the burden of proving resistance, especially
where there is intimidation. Indeed, it has been said that in rape cases, it is not necessary that
the victim should have resisted unto death or sustained injuries in the hands of the rapist. It
suffices that intercourse takes place against her will or that she yields because of a genuine

152
CRIMINAL LAW I – DIGESTED CASES BY: BOAQUINA, MARY CRISCHAEL
153

apprehension of great harm. In incestuous rape, actual force and intimidation is not even
necessary. The reason for this is that in a rape committed by a father against his own daughter,
the moral ascendancy of the former over the latter substitutes for violence and intimidation.
Third, a medical examination is not indispensable to the prosecution of rape as long as the
evidence on hand convinces the court that conviction for rape is proper.
Lastly, the delay in reporting rape incidents in the face of physical violence cannot be taken
against the victim. A rape victim’s action is oftentimes overwhelmed by fear rather than reason.
While indeed the complainant may have tarried in reporting her defilement, the three-year
hiatus in reporting the crimes adverted to by accused-appellant will not extricate him from his
predicament.

A daughter would not concoct a story of defloration against her father, accuse him of so grave a
crime as rape, allow an examination of her private parts, submit herself to public humiliation and
scrutiny via an open trial, if she were not truly aggrieved or her sordid tale was not true and her
sole motivation was not to have the culprit apprehended and punished. In short, a teenage
unmarried lass would not file a rape case against anybody, much less her own father, if it were
not true. When a victim of rape says that she has been defiled, she says in effect all that is
necessary to show that rape has been inflicted on her and so long as her testimony meets the
test of credibility, the accused may be convicted on the basis thereof.

PEOPLE OF THE PHILIPPINES vs BARTOLOME TAMPUS and IDA MONTESCLAROS


G.R. No. 181084. June 16, 2009

FACTS: The victim was forced to drink beer by her mother, Ida Montesclaros. Accused Bartolome
Tampus then took advantage of the victim’s drunkenness and raped her, which was also
permitted by Ida Montesclaros.

ISSUE: Whether or not Ida Montesclaros participated as an accomplice

HELD: The testimony of the victim establishes that Ida cooperated in the execution of the rape
by Tampus when prior to the act of rape by Tampus, she forced her to drink beer and she agreed
to Tampus’ request for him to have sexual intercourse with her. Ida’s acts show that she had
knowledge of and even gave her permission to the plan of Tampus to have sexual intercourse
with her daughter.

Ida’s act of forcing or intimidating her to drink beer and then acceding to the request of co-
accused Tampus to be allowed to have sexual intercourse with her did not prove their conspiracy.
Hence, it held that, "unndoubtedly, Ida Montesclaros participated in the commission of the crime
by previous acts but her participation, not being indispensable, was not that of a principal. She is
liable as an accomplice."
It is settled jurisprudence that the previous acts of cooperation by the accomplice should not be
indispensable to the commission of the crime; otherwise, she would be liable as a principal by
indispensable cooperation. The evidence shows that the acts of cooperation by Ida are not

153
CRIMINAL LAW I – DIGESTED CASES BY: BOAQUINA, MARY CRISCHAEL
154

indispensable to the commission of rape by Tampus. First, because it was both Ida and Tampus
who forced the victim to drink beer, and second because Tampus already had the intention to
have sexual intercourse with her and he could have consummated the act even without Ida’s
consent.

Article 19: Accessories


Profiting from the Effects of the Crime

MENDOZA v. PEOPLE
G.R. No. L-46484. January 29, 1988

FACTS: Three hundred ten [310] bags of American rice valued at P5,908.60 belonging to the Rice
and Corn Administration [RCA] were unloaded from a vessel to a truck, and instead of delivering
it to its proper destination, Reponte and Escopin together with one named Frank, diverted the
cargo to the grocery store of Leonardo Mendoza, wherein they were caught red-handed.
After due investigation, the Assistant City Fiscal of Quezon City filed in the then Court of First
Instance of Rizal at Quezon City, an information for qualified theft naming Reponte, Escopin and
John Doe alias Frank as principals, and Leonardo Mendoza as accessory after the fact for
'purchasing and receiving from the principal the said bags of rice thereby aiding the accused to
profit from the effects of the crime.
Leonardo Mendoza asserted that his guilt as such accessory could not be sustained in the absence
of evidence that he had knowledge of the alleged commission of the crime, the finding being
based solely on presumption and suspicion, in total disregard and violation of his constitutional
right to be presumed innocent until the contrary is proved.

ISSUE: Whether or not Leonardo Mendoza participated as an accessory

HELD: It is impossible for a person to accept the responsibility of having in custody for one night
310 bags of rice valued approximately at P5,908.60 without ascertaining the real ownership of
the same. And being an outlet or retailer of the RCA he knows that the RCA rice are supposed to
be placed in the bodega of the RCA and that they are distributed to the retailers not in big
quantities such as in the instant case. It is surprising why he will accept from a person whom he
does not even know the full name, such a big quantity of rice if he is not interested in buying the
said rice. If he were not an RCA retailer he would not know the conditions of how RCA rice are
distributed, but being a retailer of the RCA he is of full knowledge how RCA rice are distributed
and sold to the public. The court cannot believe in his defense that he received that rice only for
storing purposes.
The inescapable conclusion is that he agreed to buy the goods which he knew was stolen for
certainly its price must be much lower. And even if he did not agree to buy but merely to have it

154
CRIMINAL LAW I – DIGESTED CASES BY: BOAQUINA, MARY CRISCHAEL
155

deposited in his premises, the obvious fact is that he knew that it was pilfered and he agreed to
assist in keeping the same and in profiting thereby.
Hence, while there is no direct proof that Mendoza knew that the rice had been stolen, the
totality of circumstantial evidence point to the fact that he knew that the rice he was receiving
from Frank was stolen. Circumstantial evidence may be the basis for conviction if there is more
than one circumstance, the facts from which the inferences may be derived are proven, and the
combination of all the circumstances is such as to produce a conviction beyond a reasonable
doubt.

TAER v. COURT OF APPEALS


G.R. No. 85204 June 18, 1990

FACTS: Accused Cirilo Saludes slept in the house of his compadre accused Jorge Taer. Accused
Emilio Namocatcat and Mario Cago then arrived at Taer's house with two (2) male carabaos
owned by and which Namocatcat wanted Taer to tend. The said carabaos were left at Taer's
place.

Tirso Dalde and Eladio Palaca discovered the next morning that their respective male carabaos,
3 to 4 years old, were missing. After searching in vain, Dalde and Palaca found their missing
carabaos tied to a bamboo thicket near the house of accused Taer who was then not in the house.
Upon their query why their carabaos were found at his place, accused Taer, according to Dalde
and Palaca replied that the carabaos reached his place tied together without any person in
company. According to accused Taer, what he told Dalde and Palaca was that the carabaos were
brought to his place by the accused Namocatcat who asked him to tell anybody looking for them
that they just strayed thereat.

ISSUE: Whether or not Taer participated as an accessory

HELD: There is conspiracy when two or more persons come to an agreement regarding the
commission of an offense and decide to commit it. Although the facts may show a unity of
purpose and unity in the execution of the unlawful objective, essential however is an agreement
to commit the crime and a decision to commit it.
Thus mere knowledge, acquiescence to, or approval of the act, without cooperation or
agreement to cooperate, is not enough to constitute one a party to a conspiracy absent the
intentional participation in the transaction with a view to the furtherance of the common design
and purpose.
Without having participated either as principal or as an accomplice, for he did not participate in
the taking of the carabaos, he took part subsequent to the commission of the act of taking by
profiting himself by its effects. By employing the two carabaos in his farm, Taer was profiting by
the objects of the theft. Taer is thus only an accessory after the fact.

Presidential Decree 1612

155
CRIMINAL LAW I – DIGESTED CASES BY: BOAQUINA, MARY CRISCHAEL
156

DIZON-PAMINTUAN V. PEOPLE
G.R. No. 111426 July 11, 1994

FACTS: Five robbers ransacked the house of Teodoro Encarnacion and took away jewelries and
other personal properties including cash. After the intruders left the house, Encarnacion reported
the matter immediately to the police. He was later told that some of the jewelry stolen were
displayed at the stall being tended by Norma Dizon Pamintuan in Chinatown area. The petitioner
admitted that she got the items but she did not know they were stolen.

ISSUE: Whether or not Norma participated as an accessory

HELD: Fencing, as defined in Section 2 of P.D. No. 1612 (Anti-Fencing Law), is "the act of any
person who, with intent to gain for himself or for another, shall buy, receive, possess, keep,
acquire, conceal, sell or dispose of, or shall buy and sell, or in any manner deal in any article, item,
object or anything of value which he knows, or should be known to him, to have been derived
from the proceeds of the crime of robbery or theft."

The petitioner disclosed that she was engaged in the purchase and sale of jewelry and that she
used to buy from a certain Fredo who was not established as a licensed dealer or supplier of
jewelry. Section 6 of P.D. No. 1612 provides that "all stores, establishments or entitles dealing in
the buy and sell of any good, article, item, object or anything of value obtained from an
unlicensed dealer or supplier thereof, shall before offering the same for sale to the public, secure
the necessary clearance or permit from the station commander of the Integrated National Police
in the town or city where such store, establishment or entity is located."

Since Section P.D. No. 1612 expressly provides that "mere possession of any good, article, item,
object, or anything of value which has been the subject of robbery or thievery shall be prima facie
evidence of fencing," it follows that the petitioner is presumed to have knowledge of the fact
that the items found in her possession were the proceeds of robbery or theft. The presumption
is reasonable for no other natural or logical inference can arise from the established fact of her
possession of the proceeds of the crime of robbery or theft. This presumption does not offend
the presumption of innocence enshrined in the fundamental law.

RAMON C. TAN vs. PEOPLE OF THE PHILIPPINES


G.R. No. 134298. August 26, 1999

FACTS: Manuelito Mendez was arrested in the Visayas and admitted that he and his companion
Gaudencio Dayop stole from the complainant's warehouse some boat spare parts such as bronze
and stainless propellers and brass screws. Manuelito Mendez asked the complainant's
forgiveness and pointed to petitioner Ramon C. Tan as the one who bought the stolen items and
who paid the amount of P13,000.00, in cash to Mendez and Dayop, and they split the amount

156
CRIMINAL LAW I – DIGESTED CASES BY: BOAQUINA, MARY CRISCHAEL
157

with one another. Complainant did not file a case against Manuelito Mendez and Gaudencio
Dayop.

The accused denied having talked to Manuelito Mendez over the phone on the day of the delivery
of the stolen items and could not have accepted the said items personally for everytime goods
are delivered to his store, the same are being accepted by his staff. It is not possible for him to
be at his office at about 7:00 to 8:00 o'clock in the morning, because he usually reported to his
office at 9:00 o'clock.
ISSUE: Whether or not Tan participated as an accessory

HELD: Short of evidence establishing beyond reasonable doubt the existence of the essential
elements of fencing, there can be no conviction for such offense. "It is an ancient principle of our
penal system that no one shall be found guilty of crime except upon proof beyond reasonable
doubt.” Complainant Rosita Lim testified that she lost certain items and Manuelito Mendez
confessed that he stole those items and sold them to the accused. However, Rosita Lim never
reported the theft or even loss to the police. She admitted that after Manuelito Mendez, her
former employee, confessed to the unlawful taking of the items, she forgave him, and did not
prosecute him. Theft is a public crime. It can be prosecuted de oficio, or even without a private
complainant, but it cannot be without a victim. As complainant Rosita Lim reported no loss, we
cannot hold for certain that there was committed a crime of theft. Thus, the first element of the
crime of fencing is absent, that is, crime of robbery or theft has been committed.
Without petitioner knowing that he acquired stolen articles, he cannot be guilty of "fencing".
Consequently, the prosecution has failed to establish the essential elements of fencing, and thus
petitioner is entitled to an acquittal.

Concealing or Destroying the Body of the Crime

THE PEOPLE OF THE PHILIPPINES vs. RICARDO VERZOLA & JOSEFINA MOLINA
G.R. No. L-35022 December 21, 1977

FACTS: Bernardo Molina was clubbed to death by Ricardo Verzola in the presence of appellant
Josefina Molina inside Molina's house. The body of the victim was subsequently carried by the
two appellee to the ground and left at the foot of the stairs.

ISSUE: Whether or not Josefina Molina participated as an accessory

HELD: Although appellant Josefina Molina admitted that she was the paramour of her co-
appellant for over a year, there is no proof that she had knowledge of the criminal design of her
co-appellant. Neither has she cooperated with him by previous or simultaneous acts, much less
is there any showing that she supplied the principal with material or moral aid. Her only
participation was in assisting her co-appellant in bringing the body of the deceased to the ground.
An accessory does not participate in the criminal design, nor cooperate in the commission of the
felony, but, with knowledge of the commission of the crime, he subsequently takes part in three

157
CRIMINAL LAW I – DIGESTED CASES BY: BOAQUINA, MARY CRISCHAEL
158

(3) ways: (a) by profiting from the effects of the crime; (b) by concealing the body, effects or
instruments of the crime in order to prevent its discovery; and (c) by assisting in the escape or
concealment of the principal of the crime, provided he acts with abuse of his public functions or
the principal is guilty of treason, parricide, murder, or an attempt to take the life of the Chief
Executive or is known to be habitually guilty of some other crime. The main difference separating
accessories after the fact the responsibility of the accessories is subsequent to the consummation
of the crime and subordinate to that of the principal.

There is no iota of proof that Josefina Molina ever attempted "to destroy the body of the crime"
or to make it appear that death of the victim was accidental. It must be noted that Josefina
testified that she helped her co- appellant bring the body of the deceased down the stairs
because of fear. Even if she assisted her co-appellant without duress, simply Verzola in bringing
the body down the house to the foot of the stairs and leaving said body for anyone to see, cannot
be classified as an attempt to or destroy the body of the crime the effects or instruments thereof,
must be done to prevent the discovery of the crime. In the case at bar, the body was left at the
foot of the stairs at a place where it was easily visible to the public. Under such circumstances
there could not have been any attempt on the part of Josefina to conceal or destroy the body of
the crime. Hence, said appellant was acquitted.

Assisting the Principal to Escape

PEOPLE OF THE PHILIPPINES vs. NEMESIO TALINGDAN, MAGELLAN TOBIAS, AUGUSTO BERRAS,
PEDRO BIDES and TERESA DOMOGMA
G.R. No. L-32126 July 6, 1978

FACTS: Appellants Nemesio Talingdan, Magellan Tobias, Augusto Berras and Pedro Bides, all
armed with long firearms and acting inconspiracy with each other gunned down Bernardo as the
latter was sitting by the supper table in their house. They were actually seen committing the
offense by the witness Corazon, Bernardo’s daughter. After the killing, Teresa warned Corazon
not to tell anyone or else she will kill her.
Corazon testified that two nights before, the deceased Bernardo and appellant Teresa had a
violent quarrel during which he slapped her several times. She went to seek the help of the police,
and it was appellant Talingdan, a policeman of their town, who went to the vicinity of their house
and challenged her father to come down, but the latter refused because the former was a
policeman and was armed. And so, Talingdan left after shouting to her father that "If I will find
you someday, I will kill you."

Corazon further declared the amorous relationship between her mother and appellant Talingdan.
And that after the quarrel between her father and her mother and the threat made by Talingdan
to the former, she saw all the herein four male accused-appellants meeting with her mother in a
small hut, near where she was then washing clothes. As she was approaching them, accused
Teresa Domogma shoved her away saying "You tell your father that we will kill him".

158
CRIMINAL LAW I – DIGESTED CASES BY: BOAQUINA, MARY CRISCHAEL
159

ISSUE: Whether or not Teresa Domogma participated as an accessory

HELD: Although it is not clear that she helped directly in the planning and preparation thereof,
we are convinced that she knew it was going to be done and did not object.
There is in the record morally convincing proof that she is at the very least an accessory to the
offense committed by her co-accused. She was inside the room when her husband was shot. As
she came out after the shooting, she inquired from Corazon if she was able to recognize the
assailants of her father. When Corazon Identified appellants Talingdan, Tobias, Berras and Bides
as the culprits, Teresa did not only enjoin her daughter not to reveal what she knew to anyone,
she went to the extent of warning her, "Don't tell it to anyone. I will kill you if you tell this to
somebody." Later, when the peace officers who repaired to their house to investigate what
happened, instead of helping them with the information given to her by Corazon, she claimed
she had no suspects in mind. In other words, whereas, before the actual shooting of her husband,
she was more or less passive in her attitude regarding her co-appellants' conspiracy, known to
her, to do away with him, after Bernardo was killed, she became active in her cooperation with
them. These subsequent acts of her constitute "concealing or assisting in the escape of the
principal in the crime" which makes her liable as an accessory after the fact under paragraph 3
of Article 1 of the Revised Penal Code.

LITO VINO vs. THE PEOPLE OF THE PHILIPPINES and THE COURT OF APPEALS
G.R. No. 84163 October 19, 1989

FACTS: Lito Vino and Sgt. Jesus Salazar were charged with murder. The assailant was Salazar with
a rifle aboard the bicycle driven by Vino. Petitioner was charged as a principal in the commission
of the crime.
Petitioner also filed a motion for reconsideration after being convicted as an accessory in view of
the acquittal of Salazar in a separate proceeding.

ISSUES
1. Whether or not Lito Vino participated as an accessory
2. Whether or not the trial of an accessory can proceed without awaiting the result of the
separate charge against the principal.
3. Whether or not Lito Vino must be acquitted in view of the acquittal of the alleged
principal.

HELD
1. There is no doubt that the crime of murder had been committed and that the evidence
tended to show that Jessie Salazar was the assailant. That the petitioner was present
during its commission or must have known its commission is the only logical conclusion
considering that immediately thereafter, he was seen driving a bicycle with Salazar
holding an armalite, and they were together when they left shortly thereafter. At least

159
CRIMINAL LAW I – DIGESTED CASES BY: BOAQUINA, MARY CRISCHAEL
160

two witnesses, Ernesto and Julius Tejada, attested to these facts. It is thus clear that
petitioner actively assisted Salazar in his escape. Petitioner's liability is that of an
accessory.
2. The corresponding responsibilities of the principal, accomplice and accessory are distinct
from each other. As long as the commission of the offense can be duly established in
evidence the determination of the liability of the accomplice or accessory can proceed
independently of that of the principal.
3. Salazar was acquitted only because the identity of the assailant was not clearly
established. On the other hand, the commission of the crime of murder and the
responsibility of the petitioner as an accessory was established. In such case, the Court
holds that the accessory can be prosecuted and held liable independently of the assailant.

Hence, in the trial of the case against Vino, wherein he did not even adduce evidence in his
defense, his liability as such an accessory was established beyond reasonable doubt in that he
assisted in the escape of the assailant from the scene of the crime. The identity of the assailant
is of no material significance for the purpose of the prosecution of the accessory. Even if the
assailant cannot be identified, the responsibility of Vino as an accessory is indubitable.

Article 48: Complex Crimes


Compound (Delito Compuesto)

THE PEOPLE OF THE PHILIPPINES vs. JOSE PINCALIN, RODOLFO BELTRAN, EDUARDO EMPLEO
and ALEJANDRO JANDOMON
G.R. No. L-38755 January 22, 1981

FACTS: The herein accused, Jose Pincalin, Rodolfo Beltran, Eduardo Empleo and Alejandro
Jandomon, all Visayans (except Beltran) and members of the Oxo and Happy-Go-Lucky gangs,
conspired to kill some of their fellow-prisoners in dormitory 6-A of the New Bilibid Prison, who
were members of the Sputnik gang, to avenge the killings of certain Visayan prisoners.
About an hour later, the accused proceeded to implement the objective of the conspiracy. While
Abril was seated on his bed watching someone who was making a basket Beltran and Empleo
approached him frontally and stabbed him Abril fell on the floor. While in that position, Empleo
stabbed him six times while Beltran stabbed him five times.
The second victim, Tilosa was standing near the door of the dormitory when Jandomon stabbed
him on the right side of his body. As Tilosa resisted, Jandomon stabbed him repeatedly until he
collapsed on the floor.

The third victim, Francisco, was standing near a wall facing the prison hospital and, as he heard
Abril asking why he was assaulted when he had not done anything wrong, Francisco was stabbed

160
CRIMINAL LAW I – DIGESTED CASES BY: BOAQUINA, MARY CRISCHAEL
161

by Pincalin in the abdomen near the waist. Francisco avoided further assaults from Pincalin by
climbing a window.

ISSUE: Whether or not the crime committed is a complex crime

HELD: We find that the four accused are guilty of the complex crime of double murder and
frustrated murder aggravated by quasi-recidivism. This case is governed by the rule that when
for the attainment of a single purpose, which constitutes an offense various acts are executed,
such acts must be considered as only one offense, a complex one.
In other words, where a conspiracy animates several persons with a single purpose, their
individual acts done in pursuance of that purpose are looked upon as a single act, the act of
execution, giving rise to a complex offense. Various acts committed under one criminal impulse
may constitute a single complex offense.

DISSENTING OPINION
The appellants should be guilty of two separate murders, not of the complex crime of double
murder.

Article 48 of the Revised Penal Code states that "when a single act (not a single purpose)
constitutes two or more grave or less grave felonies ... , the penalty for the most serious crime
shall be imposed, the same to be applied in its maximum period".
As stressed in People vs. Pineda (L-26222, 20 SCRA 754, July 21, 1967), cited in Gamboa vs. CA,
supra, "to apply the first half of Article 48 ... there must be singularity of criminal acts; singularity
of criminal impulse is not written in to the law."

THE PEOPLE OF THE PHILIPPINES vs. ANTONIO TOLING y ROVERO and JOSE TOLING y ROVERO
G.R. No. L-27097 January 17, 1975

FACTS: The Toling twins boarded a train where they were seated with other passengers. There
were more than one hundred twenty passengers in the coach. Some passengers were standing
on the aisle.
Not long after, Antonio with a pair of scissors, and Jose with his knife, stabbed several passengers
whom they encountered inside the coach. The Toling brothers committed multiple murder (nine
victims), multiple frustrated murder (six victims) and triple homicide (as to three persons who
died after jumping from the running train to avoid being stabbed).

ISSUE: Whether or not the crime committed is a complex crime

HELD: The eight killings and the attempted murder were perpetrated by means of different acts.
Hence, they cannot be regarded as constituting a complex crime under article 48 of the Revised
Penal Code which refers to cases where "a single act constitutes two or more grave felonies, or
when an offense is a necessary means for committing the other".

161
CRIMINAL LAW I – DIGESTED CASES BY: BOAQUINA, MARY CRISCHAEL
162

The twins are liable for eight (8) murders and one attempted murder. (See People vs. Salazar,
105 Phil. 1058 where the accused Moro, who ran amuck, killed sixteen persons and wounded
others, was convicted of sixteen separate murders, one frustrated murder and two attempted
murders).

THE PEOPLE OF THE PHILIPPINES vs. MARIO TABACO


G.R. Nos. 100382-100385 March 19, 1997

FACTS: The group of the late Mayor Jorge Arreola of Buguey, Cagayan, arrived at the cockpit
arena while the accused Mario Tabaco was seated on the arm of the bench situated at the lower
portion of the arena about more than three (3) meters away, from the place where the late
Mayor and his group were seated when he suddenly without warning or provocation, shot the
late mayor Jorge Arreola, with his M-14 rifle, followed by several successive burst of gunfire,
resulting in the shooting to death of Mayor Arreola, Capt. Oscar Tabulog, Felicito Rigunan and
Pat. Romeo Regunton, although the latter managed to run passing through the western gate near
the gaffers cage but was chased by accused Tabaco. Regunton was later found dead inside the
canteen of Mrs. Amparo Go inside the Octagon cockpit arena.

Pat. Mariano Retreta of INP Buguey, who was then at the Co’s canteen, saw the accused going
out rushing from the cockpit arena, at a distance of one meter. He tried to pacify Tabaco.
Meanwhile, Sgt. Benito Raquepo,one of those assigned to maintain peace and order at the
Octagon cockpit arena, who was at the canteen taking snacks, heard five (5) successive gun
reports coming from inside the cockpit arena, and saw the accused Tabaco coming from inside
the cockpit arena. They stood face to face holding their rifles and when Tabaco pointed his gun
towards Sgt. Raquepo, Pat. Retreta grappled for the possession of the gun to disarm Tabaco, and
in the process, the gun went off hitting Sgt. Raquepo and also the late Jorge Siriban who
happened to be near Raquepo. Siriban died on the spot while Raquepo survived his wounds on
his legs due to adequate medical treatment.
There were other persons injured that evening namely: (1) Antonio Chan injured on his right foot;
(2) Salvador Berbano injured on his right forearm and on his right abdomen and (3) Rosario
Peneyra on his face and right shoulder. But, the three, did not file their complaints.”

ISSUE: Whether or not the crime committed is a complex crime

HELD: It was duly proved beyond doubt that the gun used by the accused, is admittedly an
automatic powerful weapon, more powerful than an M-16 armalite rifle. It is so powerful that
the bullets can penetrate even more than five persons resulting to their deaths.
We hold that the trial court was in error in imposing only a single penalty of reclusion perpetua
for all four murder cases. The trial court holding that a complex crime was committed since “the
evidence shows that the four (4) victims were FELLED by one single shot/burst of fire and/or
successive automatic gun fires, meaning continuous (emphasis ours)” does not hold water.

162
CRIMINAL LAW I – DIGESTED CASES BY: BOAQUINA, MARY CRISCHAEL
163

The trial court misappreciated the facts in People vs. Pama. In said case, there was only one bullet
which killed two persons. Hence, there was only a single act which produced two crimes, resulting
in a specie of complex crime known as a compound crime, wherein a single act produces two or
more grave or less grave felonies. In the case at bench, there was more than one bullet expended
by the accused-appellant in killing the four victims. The evidence adduced by the prosecution
show that Tabaco entered the cockpit with a fully loaded M-14 sub-machine gun. He fired the
weapon, which contained 20 rounds of bullets in its magazine, continuously. When the rifle was
recovered from Tabaco, the magazine was already empty. Moreover, several spent shells were
recovered from the scene of the crime. Hence, the ruling enunciated in People vs. Pama cannot
be applied.

On the contrary, what is on all fours with the case at bench is the ruling laid down in People vs.
Desierto. The accused in that case killed five persons with a Thompson sub-machine gun, an
automatic firearm which, like the M-14, is capable of firing continuously. As stated therein:
“In the case at bar, Article 48 of the Revised Penal Code is not applicable because the death of
each of the five persons who were killed by appellant and the physical injuries inflicted upon each
of the two other persons injured were not caused by the performance by the accused of one
simple act as provided for by said article. Although it is true that several successive shots were
fired by the accused in a short space of time, yet the factor which must be taken into
consideration is that, to each death caused or physical injuries inflicted upon the victims,
corresponds a distinct and separate shot fired by the accused, who thus made himself criminally
liable for as many offenses as those resulting from every singe act that produced the same.
Although apparently he perpetrated a series of offenses successively in a matter of seconds, yet
each person killed and each person injured by him became the victim, respectively, of a separate
crime of homicide or frustrated homicide. Except for the fact that five crimes of homicide and
two cases of frustrated homicide were committed successively during the tragic incident, legally
speaking there is nothing that would connect one of them with its companion offenses.”
In Desierto, although the burst of shots was caused by one single act of pressing the trigger of
the Thompson sub-machine gun, in view of its special mechanism, the person firing it has only to
keep pressing the trigger with his finger and it would fire continually. Hence, it is not the act of
pressing the trigger which should produce the several felonies, but the number of bullets which
actually produced them.

Clarifying the applicability of Art. 48 of the Revised Penal Code, the Supreme Court further stated
in People vs. Pineda that "to apply the first half of Article 48, . . . there must be singularity of
criminal act; singularity of criminal impulse is not written into the law." The firing of several
bullets by Tabaco, although resulting from one continuous burst of gunfire, constitutes several
acts. Each person, felled by different shots, is a victim of a separate crime of murder. There is no
showing that only a single missile passed through the bodies of all four victims. The killing of each
victim is thus separate and distinct from the other. In People vs. Pardo 34 we held that:
Where the death of two persons does not result from a single act but from two different shots,
two separate murders, and not a complex crime, are committed.

163
CRIMINAL LAW I – DIGESTED CASES BY: BOAQUINA, MARY CRISCHAEL
164

Consequently, the four murders which resulted from a burst of gunfire cannot be considered a
complex crime. They are separate crimes. The accused-appellant must therefore be held liable
for each and every death he has caused, and sentenced accordingly to four sentences of reclusion
perpetua.

Complex Crime Proper

JUAN PONCE ENRILE vs. JUDGE JAIME SALAZAR et al


G.R. No. 92163 June 5, 1990

FACTS: Petitioner was arrested on the strength of a warrant of arrest issued on an information
charging him and three others with the complex crime of rebellion with murder and multiple
frustrated murder committed during the failed coup d’etat. Petitioner was taken to and held
overnight at the NBI headquarters without bail, none having been recommended in the
Information and none fixed in the arrest warrant.

ISSUE: Whether or not the crime committed is a complex crime and that the court must abandon
Hernandez doctrine (Rebellion can absorb other crimes).

HELD: No. The decision in People v. Hernandez, 99 Phil. 515 (1956) that there is no such crime in
our statute books as rebellion complexed with murder, that murder committed in connection
with a rebellion is absorbed by the crime of rebellion, and that a resort to arms resulting in the
destruction of life or property constitutes neither two or more offenses nor a complex crime but
one crime – rebellion pure and simple.
11 Members of the Court voted against abandoning Hernandez. The ruling remains good law, its
substantive and logical bases have withstood all subsequent challenges and no new ones are
presented here persuasive enough to warrant a complete reversal. The President in effect by
legislative fiat reinstated Hernandez as binding doctrine with the effect of law. The Court can do
no less than accord it the same recognition, absent any sufficiently powerful reason against so
doing.
---------------------
There is an apparent need to restructure the law on rebellion, either to raise the penalty therefor
or to clearly define and delimit the other offenses to be considered as absorbed thereby, so that
it cannot be conveniently utilized as the umbrella for every sort of illegal activity undertaken in
its name. The Court has no power to effect such change, for it can only interpret the law as it
stands at any given time, and what is needed lies beyond interpretation. Hopefully, Congress will
perceive the need for promptly seizing the initiative in this matter, which is properly within its
province.
WHEREFORE, the Court reiterates that based on the doctrine enunciated in People vs.
Hernandez, the questioned information filed against petitioners Juan Ponce Enrile and the
spouses Rebecco and Erlinda Panlilio must be read as charging simple rebellion only, hence said
petitioners are entitled to bail, before final conviction, as a matter of right.

164
CRIMINAL LAW I – DIGESTED CASES BY: BOAQUINA, MARY CRISCHAEL
165

DISSENTING OPINION
The relevance of the distinction is significant, more particularly, if applied to contemporaneous
events happening in our country today. Theoretically, a crime which is indispensable in the
commission of another must necessarily be an element of the latter; but a crime that is merely
necessary but not indispensable in the commission of another is not an element of the latter, and
if and when actually committed, brings the interlocking crime within the operation of the
complex crime provision (Art. 48) of the Revised Penal Code.
With that distinction, common crimes committed against Government forces and property in the
course of rebellion are properly considered indispensable overt acts of rebellion and are logically
absorbed in it as virtual ingredients or elements thereof, but common crimes committed against
the civilian population in the course or on the occasion of rebellion and in furtherance thereof,
may be necessary but not indispensable in committing the latter, and may, therefore, not be
considered as elements of the said crime of rebellion.
To illustrate, the deaths occurring during armed confrontation or clashes between government
forces and the rebels are absorbed in the rebellion, and would be those resulting from the
bombing of military camps and installations, as these acts are indispensable in carrying out the
rebellion. But deliberately shooting down an unarmed innocent civilian to instill fear or create
chaos among the people, although done in the furtherance of the rebellion, should not be
absorbed in the crime of rebellion as the felonious act is merely necessary, but not indispensable.
In the latter case, Article 48 of the Revised Penal Code should apply.

PEOPLE OF THE PHILIPPINES vs. ROLANDO VALDEZ


G.R. No. 127663 March 11, 1999

FACTS: Rolando Valdez and his companions were armed with guns. Without warning, they
pointed their guns and fired at Montano's group. Thereafter, Valdez and companions left. The
shooting incident left Ramon Garcia, Jean Marie Garcia, Sandra Montano and Willie Acosta dead.
Accused-appellant Rolando Valdez seeks reversal of the judgment of conviction promulgated by
the trial court sentencing him to death for the complex crime of Multiple Murder with Double
Frustrated Murder.

ISSUE: Whether or not the crime committed is a complex crime

HELD: The concept of a complex crime is defined in Article 48 of the Revised Penal Code, to wit:
Art. 48. Penalty for complex crimes — When a single act constitutes two or more grave or less
grave felonies or when an offense is a necessary means for committing the other, the penalty for
the most serious crime shall be imposed, the same to be applied in its maximum period. (As
amended by Act No. 4000.
The case at bar does not fall under any of the two instances defined above. The Office of the
Provincial Prosecutor of Pangasinan erroneously considered the case as falling under the first. It
is clear from the evidence on record, however, that the four crimes of murder resulted not from
a single act but from several individual and distinct acts. For one thing, the evidence indicates

165
CRIMINAL LAW I – DIGESTED CASES BY: BOAQUINA, MARY CRISCHAEL
166

that there was more than one gunman involved, and the act of each gunman is distinct from that
of the other. It cannot be said therefore, that there is but a single act of firing a single firearm.
There were also several empty bullet shell recovered from the scene of the crime. This confirms
the fact that several shots were fired. Furthermore, considering the relative position of the
gunmen and their victims, some of whom were riding the motorized tricycle itself while the
others were seated inside the sidecar thereof, it was absolutely impossible for the four victims
to have been hit and killed by a single bullet. Each act by each gunman pulling the trigger of their
respective firearms, aiming each particular moment at different persons constitute distinct and
individual acts which cannot give rise to the complex crime of multiple murder. We therefore
rule that accused-appellant is guilty, not of a complex crime of multiple murder, but of four
counts of murder for the death of the four victims in this case. In the same manner, accused-
appellant is likewise held guilty for two counts of frustrated murder.
In the case at bar, accused-appellant, being guilty of four separate counts of murder, the proper
penalty should be four sentences of reclusion perpetua. In addition, he being guilty of two counts
of frustrated murder, accused-appellant must be meted out an indeterminate sentence ranging
from a minimum of 6 years and 1 day of prison mayor to maximum of 12 years and 1 day of
reclusion perpetua for each offense.

PEOPLE V. GARCIA
GR 141125, February 28, 2002

FACTS: Victim, a sophomore student was abducted and made unconscious on her way home
from school. After regaining consciousness, she realized she was totally undressed in a bed and
there were four naked men inside the room. The four men proceeded with force in having sexual
intercourse with her and made her unconscious after, she woke up on a roadside and managed
to get home with the help of a taxi driver. Complaints have been filed and medical findings
attested to the charges. Accused-Appellant was arrested in connection with a different charge
for rape. The other 3 accused remained at-large and accused-appellant was convicted of Forcible
abduction with rape and 3 counts of rape.

ISSUE: Whether court erred in finding appellant guilty of beyond reasonable doubt for the
complex crime of forcible abduction with rape.

HELD: The trial court, therefore, did not err in convicting accused-appellant of the complex crime
of forcible abduction with rape. The two elements of forcible abduction, as defined in Article 342
of the Revised Penal Code, are: (1) the taking of a woman against her will and (2) with lewd
designs. The crime of forcible abduction with rape is a complex crime that occurs when there is
carnal knowledge with the abducted woman under the following circumstances: (1) by using
force or intimidation; (2) when the woman is deprived of reason or otherwise unconscious; and
(3) when the woman is under twelve years of age or is demented.

166
CRIMINAL LAW I – DIGESTED CASES BY: BOAQUINA, MARY CRISCHAEL
167

The prosecution sufficiently proved beyond reasonable doubt that accused-appellant succeeded
in forcibly abducting the complainant with lewd designs, established by the actual rape. Hence,
accused-appellant is guilty of the complex crime of forcible abduction with rape.
However, as correctly held by the trial court, there can only be one complex crime of forcible
abduction with rape. The crime of forcible abduction was only necessary for the first rape. Thus,
the subsequent acts of rape can no longer be considered as separate complex crimes of forcible
abduction with rape. They should be detached from and considered independently of the forcible
abduction. Therefore, accused-appellant should be convicted of one complex crime of forcible
abduction with rape and three separate acts of rape.
The penalty for complex crimes is the penalty for the most serious crime which shall be imposed
in its maximum period. Rape is the more serious of the two crimes and, when committed by more
than two persons, is punishable with reclusion perpetua to death under Article 266-B of the
Revised Penal Code, as amended by Republic Act No. 8353. Thus, accused-appellant should be
sentenced to the maximum penalty of death for forcible abduction with rape.

PEOPLE V. ALFECHE
GR No. 124213, August 17, 1998

FACTS: The case is regarding trial for two separate charges of rape against Dante. During the
commission of the crime, victim was 17 years old and on two separate occasions, accused, with
2 other malefactors, entered the dwelling where victim was working as a maid and raped her
when the owners were away. Victim opted not to report the same to anyone until her mother
discover of her pregnancy that the former was forced to reveal what transpired. The RTC
convicted accused twice of rape with 1 aggravating circumstance of dwelling on one incident and
dwelling on both instances. Penalty of death with civil indemnification for damages were
imposed for each conviction.

ISSUE: Whether guilt was proven beyond reasonable doubt for each of the conviction.

HELD: We agree with DANTE that the first alleged rape was not sufficiently proved. Since rape is
not normally committed in the presence of witnesses, the only evidence that can oftentimes be
offered to establish the guilt of the accused is the complainant's testimony. Said testimony must
be clear and positive to prove that the acts which constitute the elements of rape were
committed. Although ANALIZA was unconscious when the first rape was allegedly committed,
she could have described circumstances constituting an unbroken chain of events that would
indicate that the offense was actually perpetrated by the accused. In this case, ANALIZA's
unconsciousness broke the chain of events from which we could have inferred the occurrence of
the offense.
The only issue left is the penalty which may be imposed on DANTE for the 2nd alleged
rape on 18 September 1994. The trial court imposed the death penalty pursuant to Article 335
of the Revised Penal Code, as amended by R.A. No. 7659, in view of the finding that the crime
was committed by three persons and with the use of a knife, and was attended with the
aggravating circumstance of dwelling. It must be first observed that the complaints in Criminal

167
CRIMINAL LAW I – DIGESTED CASES BY: BOAQUINA, MARY CRISCHAEL
168

Cases Nos. 4615-0 and 4616-0 charge DANTE and his co-accused with rape "in violation of Article
335, Revised Penal Code, instead of "Article 335, Revised Penal Code, as amended by R.A. No.
7659." Before Article 335 was amended by R.A. No. 7659 the penalty for rape when committed
with the use of a deadly weapon or by two or more persons was Reclusion Perpetua to death.
Its third paragraph read: Whenever the crime of rape is committed with the use of a deadly
weapon or by two or more persons, the penalty shall be Reclusion Perpetua to death. This
paragraph was not touched by R.A. No. 7659. Nevertheless, from the ratification on 2 February
1987 of the Constitution until the effectivity of R.A. No. 7659 on 31 December 1993 the
imposition of the death penalty was constitutionally prohibited pursuant to Section 14(2), Article
III of the former. The complaints in the cases below do not use the language of the law, viz., "with
the use of a deadly weapon or by two or more persons." In lieu of "deadly weapon," the
complaints use "knife"; and, there is no specific allegation that the crime was committed by "two
or more persons," but only an allegation of conspiracy among the three accused. These
allegations are, however, sufficient for purposes of the above-quoted third paragraph of Article
335.
By way of conclusion, we do not hesitate to rule that in Criminal Case No. 4615-0, the crime was
committed with the use of a deadly weapon and by two or more persons under the third
paragraph of Article 335 of the Revised Penal Code, as amended. The generic aggravating
circumstance of dwelling justified the imposition of the greater penalty of death. Dwelling is
considered an aggravating circumstance because primarily of the sanctity of privacy the law
accords to human abode. The dwelling need not be owned by the victim. Premises considered,
his conviction in Criminal Case No. 4615-0 and the penalty imposed, death, must stand. Four
members of this Court maintain their position that Republic Act No. 7659, insofar as it prescribes
the death penalty, is unconstitutional; but they nevertheless submit to the ruling of the majority
of the Court that the law is constitutional and the death penalty should be imposed in the case
at bar.
WHEREFORE, judgment is hereby rendered REVERSING the judgment in Criminal Case No. 4616-
0 and ACQUITTING accused-appellant DANTE ALFECHE y Tamparong on ground of reasonable
doubt. However, the judgment in Criminal Case No. 4615-0 convicting said accused-appellant of
the crime of rape committed on 18 September 1994 and sentencing him to suffer the penalty of
death is AFFIRMED, with the modification that (1) an indemnity is hereby imposed in the amount
of P75,000; (2) the award of moral damages is increased from P30,000 to P50,000; and (3)
exemplary damages in the amount of P10,000 is also awarded in favor of the victim ANALIZA
DUROJA.

BATULANON V. PEOPLE
GR No. 139857, September 15, 2006

FACTS: Accused-appellant Batulanon was charged by her employer, Polomolok Cooperative


Credit Incorporated (PCCI) with 4 counts of Estafa through Falsification of Commercial
documents. Accused allegedly falsified individual deposits and loan ledgers (2 instances) and
cash/check vouchers (4 instances) to release fictitious loans to 4 people when in truth and fact,

168
CRIMINAL LAW I – DIGESTED CASES BY: BOAQUINA, MARY CRISCHAEL
169

none of them applied for and received proceeds of such loans. The Trial Court convicted
defendant without the testimonies of the persons whose name appears to have been forged and
was affirmed upon appeal with modification on offense that it was through falsification of Private
documents.

ISSUE: Whether there is a necessity to present persons whose names were allegedly forged in
cases of falsification.

HELD: As there is no complex crime of estafa through falsification of private document, it is


important to ascertain whether the offender is to be charged with falsification of a private
document or with estafa. If the falsification of a private document is committed as a means to
commit estafa, the proper crime to be charged is falsification. If the estafa can be committed
without the necessity of falsifying a document, the proper crime to be charged is estafa.
We find that the Court of Appeals correctly held Batulanon guilty beyond reasonable doubt of
Falsification of Private Documents. The Court of Appeals correctly ruled that the subject vouchers
are private documents and not commercial documents because they are not documents used by
merchants or businessmen to promote or facilitate trade or credit transactions nor are they
defined and regulated by the Code of Commerce or other commercial law. Rather, they are
private documents, which have been defined as deeds or instruments executed by a private
person without the intervention of a public notary or of other person legally authorized, by which
some disposition or agreement is proved, evidenced or set forth.
Article 172 punishes the crime of Falsification of a Private Document with the penalty of prision
correccional in its medium and maximum periods with a duration of two (2) years, four (4)
months and one (1) day to six (6) years. There being no aggravating or mitigating circumstances,
the penalty should be imposed in its medium period, which is three (3) years, six (6) months and
twenty-one (21) days to four (4) years, nine (9) months and ten (10) days. Taking into
consideration the Indeterminate Sentence Law, Batulanon is entitled to an indeterminate penalty
the minimum of which must be within the range of arresto mayor in its maximum period to
prision correccional in its minimum period, or four (4) months and one (1) day to two (2) years
and four (4) months. Thus, in Criminal Case Nos. 3625, 3626 and 3453, the Court of Appeals
correctly imposed the penalty of six (6) months of arresto mayor, as minimum, to four (4) years
and two (2) months of prision correccional, as maximum, which is within the range of the allowed
imposable penalty.

In Criminal Case No. 3627, the trial court convicted petitioner Batulanon for falsifying Dennis
Batulanon's signature in the cash voucher based on the Information charging her of signing the
name of her 3-year-old son, Dennis. The records, however, reveal that in Cash Voucher No. 374A,
petitioner Batulanon did not falsify the signature of Dennis. What she did was to sign: "by:
lbatulanon" to indicate that she received the proceeds of the loan in behalf of Dennis. Said act
does not fall under any of the modes of falsification under Article 171 because there in nothing
untruthful about the fact that she used the name of Dennis and that as representative of the
latter, obtained the proceeds of the loan from PCCI. The essence of falsification is the act of
making untruthful or false statements, which is not attendant in this case. As to whether, such

169
CRIMINAL LAW I – DIGESTED CASES BY: BOAQUINA, MARY CRISCHAEL
170

representation involves fraud which caused damage to PCCI is a different matter which will make
her liable for estafa, but not for falsification. Hence, it was an error for the courts below to hold
that petitioner Batulanon is also guilty of falsification of private document with respect to
Criminal Case No. 3627 involving the cash voucher of Dennis. In the instant case, there is no doubt
that as Cashier/Manager, Batulanon holds the money for administration and in trust for PCCI.
Knowing that she is no longer qualified to obtain a loan, she fraudulently used the name of her
son who is likewise disqualified to secure a loan from PCCI. Her misappropriation of the amount
she obtained from the loan is also not disputed as she even admitted receiving the same for
personal use. Although the amount received by Batulanon is reflected in the records as part of
the receivables of PCCI, damage was still caused to the latter because the sum misappropriated
by her could have been loaned by PCCI to qualified members, or used in other productive
undertakings. At any rate, the disturbance in property rights caused by Batulaono's
misappropriation is in itself sufficient to constitute injury within the meaning of Article 315.
WHEREFORE, the Decision appealed from is AFFIRMED with modifications.

Continued Crime

PEOPLE V. DE LEON
49 Phil. 437

FACTS: Vicente De Leon, herein appellant was convicted by the Municipal Trial court for 2
separate crimes of theft for stealing 2 fighting roosters belonging to 2 individuals. Upon appeal
to the Regional Trial court, he was convicted only of 1 crime of theft only. Accused sought appeal
of his case.
ISSUE: Whether the crime should constitute a single offense or 2 separate offenses.

HELD: The Attorney-General urges that the penalty for two crimes of theft be imposed upon the
accused for each of the stolen roosters. The crime of theft is an offense against personal property
and what is punished is the alarm caused in the community by the perpetration of the act which
is violative of the individual rights guaranteed by the law, as well as the damage that said act may
occasion to the members of the community. Under sound principles, the act of taking the two
roosters, in response to the unity of thought in the criminal purpose on one occasion, is not
susceptible of being modified by the accidental circumstance that the article unlawfully belonged
to two distinct persons. There is no series of acts here for the accomplishment of different
purposes, but only one of which was consummated, and which determines the existence of only
one crime.

It is not an element of the crime of theft that the culprit knows the owner of the thing stolen, the
crime being consummated provided that being stolen belongs to another and the same is taken
with intent to gain. Therefore, we are of the opinion that the unity of the intention to take a thing
belonging to another on one occasion and in the same place, constitutes the commission of only

170
CRIMINAL LAW I – DIGESTED CASES BY: BOAQUINA, MARY CRISCHAEL
171

one crime of theft; and fact that the things taken belong to different persons does not produce
a multiplicity of crimes, which must be punished separately.

The judgment appealed from must be, as is hereby, modified and the accused Vicente de Leon y
Flora is sentenced to suffer the penalty of six years and three months presidio mayor, with the
accessories of the law, and to pay the costs. So ordered.

SANTIAGO V. GARCHITORENA
GR 109266, Dec. 2, 1993

FACTS: This petition for certiorari originated from a criminal charge against petitioner Miriam
Defensor-Santiago of the Sandiganbayan with violation of Section 3(e) of R.A. No. 3019, as
amended, otherwise known as the Anti-Graft and Corrupt Practices Act, allegedly committed by
her favoring "unqualified" aliens with the benefits of the Alien Legalization Program. Petitioner
filed a motion for inhibition of Presiding Justice Garchitorena and the former moved to defer the
arraignment on the grounds that there was a pending motion for inhibition but the movement
to defer was denied. Petitioner filed a motion for a bill of particulars stated that while the
information alleged that petitioner had approved the application or legalization of "aliens" and
gave them indirect benefits and advantages it lacked a list of the favored aliens. According to
petitioner, unless she was furnished with the names and identities of the aliens, she could not
properly plead and prepare for trial.

Upon motion of petitioner, the Supreme court directed the Sandiganbayan (First Division) to
reset the arraignment to a later date and to dispose of the two incidents pending before it (Re:
disqualification of Presiding Justice Garchitorena and the motion for the bill of particulars). At
the hearing on the motion for a bill of particulars, the prosecution stated categorically that they
would file only one amended information against petitioner. However, the prosecution filed a
motion to admit the 32 Amended Informations and the Sandiganbayan (First Division)
promulgated a resolution, admitting the 32 Amended Informations and ordering petitioner to
post the corresponding bail bonds within ten days from notice. Hence, the filing of the instant
petition.

Acting on the petition for the issuance of a restraining order, we issued the Resolution ordering
Presiding Justice Garchitorena "to CEASE and DESIST” from sitting in the case until the question
of his disqualification is finally resolved by this Court and from enforcing the resolution ordering
petitioner to post bail bonds for the 32 Amended Informations and from proceeding with the
arraignment.

ISSUE: Whether the splitting of 1 information into 32 amended informations are valid.

HELD: The Supreme court’s attention was attracted by the allegation in the petition that the
public prosecutors filed 32 Amended Informations against petitioner, after manifesting to the

171
CRIMINAL LAW I – DIGESTED CASES BY: BOAQUINA, MARY CRISCHAEL
172

Sandiganbayan that they would only file one amended information. We find that, technically,
there was only one crime that was committed in petitioner's case, and hence, there should only
be one information to be file against her. The 32 Amended Informations charge what is known
as delito continuado or "continued crime" and sometimes referred to as "continuous crime."

According to Cuello Calon, for delito continuado to exist there should be a plurality of acts
performed during a period of time; unity of penal provision violated; and unity of criminal intent
or purpose, which means that two or more violations of the same penal provisions are united in
one and same instant or resolution leading to the perpetration of the same criminal purpose or
aim. The American courts following the "single larceny" rule, look at the commission of the
different criminal acts as but one continuous act involving the same "transaction" or as done on
the same "occasion". An American court held that a contrary rule would violate the constitutional
guarantee against putting a man in jeopardy twice for the same offense. The original information
also averred that the criminal act: (1) committed by petitioner was in violation of a law —
Executive Order No. 324 (2) caused an undue injury to one offended party, the Government, and
(3) was done on a single day. The 32 Amended Informations reproduced verbatim the allegation
of the original information, except that instead of the word "aliens" in the original information
each amended information states the name of the individual whose stay was legalized. The 32
Amended Informations aver that the offenses were committed on the same period of time, i.e.,
on or about October 17, 1988. The strong probability even exists that the approval of the
application or the legalization of the stay of the 32 aliens was done by a single stroke of the pen,
as when the approval was embodied in the same document. WHEREFORE, the Resolution dated
March 3, 1993 in Criminal Case No. 16698 of the Sandiganbayan (First Division) is AFFIRMED and
its Resolution dated March 11, 1993 in Criminal Case No. 16698 is MODIFIED in the sense that
the Office of the Special Prosecutor of the Office of the Ombudsman is directed to consolidate
the 32 Amended Informations (Criminal Cases Nos. 18371 to 18402) into one information
charging only one offense under the original case number, i.e., No. 16698.

GAMBOA V. COURT OF APPEALS


GR L-41054, November 28, 1975

FACTS: This is a petition to review on certiorari the judgment of the respondent Court of Appeals
involving the proper appreciation of the rule on plurality of crimes, otherwise known as
"concursus delictuorum", and the theory of "continuous crime". 124 complaints for Estafa were
filed against private respondent Benjamin Hayco by his former employer, Jose Gamboa. Only 75
proceeded after the preliminary investigation. The lower court dismissed the petition for
prohibition on the ground that the series of deposits and the subsequent withdrawals thereof
involved in the criminal cases were not the result of only one criminal impulse on the part of
private respondent.

Upon appeal, the Appellate Court reversed the order of the lower court and granted the petition
for prohibition. It directed the respondent City Fiscal "to cause the dismissal of the seventy-five
(75) criminal cases filed against petitioner-appellant, to consolidate in one information all the

172
CRIMINAL LAW I – DIGESTED CASES BY: BOAQUINA, MARY CRISCHAEL
173

charges contained in the seventy-five (75) informations and to file the same with the proper
court."

ISSUE: Whether the basic accusations contained in the seventy-five (75) informations against
private respondent constitute but a single crime of estafa.

HELD: There is plurality of crimes or "concurso de delitos" when the actor commits various
delictual acts of the same or different kind. "Ideal plurality" or "concurso ideal" occurs when a
single act gives rise to various infractions of law. This is illustrated by the very article under
consideration: (a) when a single act constitutes two or more grave or less grave felonies
(described as "delito compuesto" or compound crime); and (b) when an offense is a necessary
means for committing another offense (described as "delito complejo" or complex proper). "Real
plurality" or "concurso real", on the other hand, arises when the accused performs an act or
different acts with distinct purposes and resulting in different crimes which are juridically
independent. Unlike "ideal plurality", this "real plurality" is not governed by Article 48.

Apart and isolated from this plurality of crimes (ideal or real) is what is known as "delito
continuado" or "continuous crime". This is a single crime consisting of a series of acts arising from
a single criminal resolution or intent not susceptible of division. For Cuello Calon, when the actor,
there being unity of purpose and of right violated, commits diverse acts, each of which, although
of a delictual character, merely constitutes a partial execution of a single particular delict, such
concurrence or delictual acts is called a "delito continuado". In order that it may exist, there
should be "plurality of acts performed separately during a period of time; unity of penal provision
infringed upon or violated and unity of criminal intent and purpose, which means that two or
more violations of the same penal provision are united in one and the same intent leading to the
perpetration of the same criminal purpose or aim."

It is not difficult to resolve whether a given set of facts constitutes a single act which produces
two or more grave or less grave offenses or a complex crime under the definition of Article 48.
So long as the act or acts complained of resulted from a single criminal impulse it is usually held
to constitute a single offense to be punished with the penalty corresponding to the most serious
crime, imposed in its maximum period. The test is not whether one of the two offenses is an
essential element of the other.

In the case before Us, the daily abstractions from and diversions of private respondent of the
deposits made by the customers of the optical supply company from October 2, 1972 to
December 30, 1972, excluding Saturdays and Sundays, which We assume ex hypothesi, cannot
be considered as proceeding from a single criminal act within the meaning of Article 48. The
abstractions were not made at the same time and on the same occasion, but on variable dates.
ACCORDINGLY, the judgment of the Court of Appeals, subject matter of this proceeding, is hereby
reversed and set aside.

PEOPLE V. PEÑAS

173
CRIMINAL LAW I – DIGESTED CASES BY: BOAQUINA, MARY CRISCHAEL
174

GR 46353-46355, December 5, 1938

FACTS: The appellant was convicted on three separate charges for estafa with falsification of
public documents by the Court of First Instance of Cebu. Appellant who was then assistant post-
master, falsified 3 money orders by making it appear that the same was issued to him, imitated
signatures of the post-master and thereafter collected a sum of P600.

ISSUE: Whether 3 falsification of documents be considered as individual offenses.

HELD: On the dates alleged in the three informations, the appellant was assistant postmaster in
the municipality of Barotac Viejo, for which reason he was aware that no money order could be
brought or issued for a sum greater than $100, as expressly provided in section 1968 of the
Administrative Code. In accordance, therefore, with said provision of the law, in order to issue a
money order for P600, it was necessary to make three separate money orders. As shown by
Exhibits A, B and C, which are the falsified money orders subject of the three informations, the
appellant falsified the same on a single date: November 24, 1936, and he collected them also on
a single date, January 4, 1937 — from which it is inferred that the three acts of falsification and
the said three acts of appropriation of the sum of P200 in each case proceed from a single
purpose of the appellant, namely, to appropriate for himself the sum of P600. If he had to resort
to this means falsifying three money orders, it was because he was aware that he could not do
otherwise, in view of the legal provision, which he was supposed to know. When, for the
attainment of a single purpose which constitutes an offense, various acts are executed, such acts
must be considered only as one offense. Under this view, the appellant committed only the
complex crime of estafa with falsification of three postal money orders which are, without doubt,
official and public documents, the falsification being the means he employed to appropriate, as
he did for himself the sum of P600, to the prejudice of the Government.

PEOPLE V. MADRIGAL-GONZALES
Nos. L-16688-90, 7 SCRA 942
FACTS: Accused-Appellee Pacita Madrigal-Gonzales, together with 7 other co-accused, were
charged with 27 separate informations for falsification of public documents and 1 for
malversation of public funds. In said 27 separate informations, the eight accused were alleged to
have conspired in the commission of said offense in or about and during the period comprised
between December, 1954 and September, 1955, by having allegedly caused it to appear: that
cash aids were given when no such aids were indeed distributed to the persons named and at
the time and place and in such amounts specified, or by making and/or causing it to appear that
certain relief supplies or merchandise were purchased by the accused Pacita Madrigal-Gonzales
when in truth and in fact no such relief supplies were purchased, thereby making untruthful
statements in a narration of fact in said public and official documents.

The prosecution filed ex parte petition for the consolidation of all said 27 falsification cases and
one malversation case before only one branch of the Court of First Instance of Manila and was

174
CRIMINAL LAW I – DIGESTED CASES BY: BOAQUINA, MARY CRISCHAEL
175

granted accordingly. the prosecution suddenly reversed its stand and sought a reconsideration
of said order of consolidation and prayed instead for the distribution or farming out of said 27
falsification cases before all branches of the Court of First Instance of Manila, as a consequence
of which, and over the repeated objections of the defense, said 27 cases for falsification were
ordered distributed among the 18 different branches of said Court. The prosecution, however,
maintained the assignment of the lone case for malversation with Branch II of the lower Court to
which was assigned also three cases for falsification as above enumerated.

Herein accused-appellees filed a motion to quash Criminal Cases Nos. 36878 and 36883 to 36884,
inclusive, assigned to Branch XVIII of the lower court, on the ground of double jeopardy, said
motion having been filed also in each of the other 16 branches of the lower court before which
the said cases were docketed. The aforesaid motion were file based on 2 grounds and 1 of which
is that said 27 separate informations for falsification indeed constitute only one indictable
offense of falsification considering that the falsifications allegedly committed separately as
described under said separate informations were but the result or product of one single criminal
impulse or intent, and the same are therefore in the nature of a continuing offense which should
be alleged and prosecuted only under one information. The same was granted. The prosecution
has interposed the instant appeal on purely questions of law.

ISSUE: Whether the twenty-seven (27) falsifications were the product of only one criminal intent.

HELD: The Court cannot assume that the purpose of committing the twenty-seven (27)
falsifications was to conceal the malversation. This is so because there is no showing that for
every particular amount, they had malversed on a certain period, they had purposedly
perpetrated the corresponding falsification to cover up such amount, until the whole amount
proposed to be malversed, shall have been completely misappropriated. In the absence of such
showing, it is to be presumed that in the falsification of each document, the criminal intent was
separated and distinct. In effect, it will be noted that although all the informations in the 27
falsification cases were uniformly worded, the numbers of the vouchers alleged to have been
falsified and the amounts thereof are different. These undeniable facts, alleged in the
informations, evidently show that different acts of falsification were committed on different
vouchers and covering distinct amounts. Each information did not refer to all said acts of
falsification. Neither is there merit in the argument that said acts of falsification constituted a
continuing offense, so as to have them all prosecuted in only one information.

We need not proceed any further to determine whether the 27 falsifications perpetrated on
separate vouchers, at different dates and in various amounts, constitute 27 separate and
independent crimes, which were not continuous.
WHEREFORE, the Order of the lower court (Branch XVIII) dismissing Criminal Cases Nos. 36894,
36899 and 36904 on the ground of double jeopardy is set aside and another entered remanding
the said case for further proceedings.

175
CRIMINAL LAW I – DIGESTED CASES BY: BOAQUINA, MARY CRISCHAEL
176

Special Complex Crime

PEOPLE V. PASCUAL
G.R. No. 172326, January 19, 2009

FACTS: Accused-Appellant was convicted with the crime of rape with homicide for the death of
Lorelyn Tamayo. That Accused-Appellant was having a drinking spree before Noche Buena with
the prosecution witness, Rodolfo Jundos. That Accused left the session 3 times, the last of which
he no longer returned and twenty minutes later, they heard the wife of accused yelling for help
to chase the accused whom the witness thought was making a ruckus for the former was already
drunk. They failed to catch the accused and the wife instead asked witness to help her check the
victim who is residing on the 2nd floor of the house. Thereafter they found the lifeless body of
the victim. Accused denied the crime charged against him and claimed to be in a different place
and presented witnesses. Court of appeals affirmed the trial court’s decision.

ISSUE: Was the circumstantial evidence presented against the accused-appellant sufficient for
his conviction?

HELD: It is settled that in the special complex crime of rape with homicide, both the rape and the
homicide must be established beyond reasonable doubt. In this regard, we have held that the
crime of rape is difficult to prove because it is generally unwitnessed and very often only the
victim is left to testify for herself. It becomes even more difficult when the complex crime of rape
with homicide is committed because the victim could no longer testify. Thus, in crimes of rape
with homicide, as here, resort to circumstantial evidence is usually unavoidable.

Considering that no one witnessed the commission of the crime charged herein, the weight of
the prosecution’s evidence must then be appreciated in light of the well-settled rule that an
accused can be convicted even if no eyewitness is available, as long as sufficient circumstantial
evidence is presented by the prosecution to prove beyond doubt that the accused committed
the crime. Under Section 4, Rule 133 of the Revised Rules of Court, circumstantial evidence is
sufficient for conviction if the following requisites concur:
(a) there is more than one circumstance;
(b) the facts from which the inferences are derived have been established; and
(c) the combination of all the circumstances is such as to warrant a finding of guilt beyond
reasonable doubt.

Verily, for circumstantial evidence to be sufficient to support a conviction, all the circumstances
must be consistent with each other, consistent with the hypothesis that accused is guilty and at
the same time inconsistent with the hypothesis that he is innocent, and with every other rational
hypothesis except that of guilt. Thus, a judgment of conviction based on circumstantial evidence
can be sustained only when the circumstances proved form an unbroken chain which leads to a
fair and reasonable conclusion pointing to the accused, to the exclusion of all others, as the
culprit.

176
CRIMINAL LAW I – DIGESTED CASES BY: BOAQUINA, MARY CRISCHAEL
177

Here, the circumstances testified to by the prosecution witnesses lead to the inevitable
conclusion that the accused-appellant is the author of the crime charged. Viewed in its entirety,
the evidence in this case inevitably leads to the conclusion that accused-appellant is guilty
beyond reasonable doubt of the special complex crime of Rape with Homicide. Rape with
Homicide under Article 335 of the Revised Penal Code, in relation to Republic Act (R.A.) 7659,
provides that when by reason or on the occasion of the rape, a homicide is committed, the
penalty shall be death. However, in view of the subsequent passage of R.A. No. 9346, entitled
“An Act Prohibiting the Imposition of the Death Penalty in the Philippines,” the Court is mandated
to impose on the accused-appellant the penalty of reclusion perpetua. WHEREFORE, the
appealed decision of the CA in CA-G.R. CR HC No. 01493 is hereby AFFIRMED with
MODIFICATION.

PEOPLE V. ALGARME
G.R. No. 175978, February 12, 2009

FACTS: The prosecution charged the appellants before the RTC with the special complex crime of
robbery with homicide under an Information that on September 1995, the above-named accused
conspiring, confederating and helping one another with evident premeditation and treachery
and with intent to kill, did then and there, willfully, unlawfully and feloniously assault, attack and
stab to death one Loreto Batarilan y Ladiona, a tricycle driver, in order to rob, steal, and take
away a belt bag containing money and the wrist watch. The state presented witnesses including
an eye-witness during the commission of the felony and was able to identify the assailants. The
RTC convicted appellants Samuel Algarme and Rizaldy Gelle of the special complex crime of
robbery with homicide while the other accused, a.k.a “Stingray” is still at-large.

ISSUE: Whether the court erred with the conviction of a special complex crimes instead of 2
separate crimes.

HELD: Art. 294 of the Revised Penal Code provides that Robbery with violence against or
intimidation of persons – Penalties. - Any person guilty of robbery with the use of violence against
or intimidation of any person shall suffer:
1. The penalty of reclusion perpetua to death, when by reason or on occasion of the robbery,
the crime of homicide shall have been committed, or when the robbery shall have been
accompanied by rape or intentional mutilation or arson.
A special complex crime of robbery with homicide takes place when a homicide is committed
either by reason, or on the occasion, of the robbery. To sustain a conviction for robbery with
homicide, the prosecution must prove the following elements: (1) the taking of personal property
belonging to another; (2) with intent to gain; (3) with the use of violence or intimidation against
a person; and (4) on the occasion or by reason of the robbery, the crime of homicide, as used in
its generic sense, was committed. A conviction requires certitude that the robbery is the main
purpose, and objective of the malefactor and the killing is merely incidental to the robbery. The

177
CRIMINAL LAW I – DIGESTED CASES BY: BOAQUINA, MARY CRISCHAEL
178

intent to rob must precede the taking of human life but the killing may occur before, during or
after the robbery.
We have held in several cases that where the evidence satisfactorily establishes that the
appellant did kill and unlawfully take the personal property of the victim, but the original criminal
design to commit robbery was not duly proven – the accused-appellant should be held liable for
the separate crimes of homicide or murder.

In the case before us, the testimonies of Norman and Alicia merely established two (2) facts: that
the victim carried a belt bag containing money on that fateful morning of September 19, 1995;
and the appellants were seen carrying the said belt bag walking near the victim’s body. From
these established facts, we hold that the prosecution failed to establish the linkage required by
law between a robbery and a homicide to characterize the crime as the special complex crime of
robbery with homicide; there was no showing of the appellants’ intention – determined by their
acts, prior to, contemporaneous with and subsequent to the commission of the crime – to
commit robbery. There was likewise no testimony to show whether the appellants intended to
kill the victim in order to steal the belt bag, or whether the killing was merely an afterthought.
Thus, the appellants’ primary intent remains a mystery. The fact that they were in possession of
the victim’s belt bag after the killing does not give rise to the conclusion that their overriding
intention was to rob the victim. The special complex crime is therefore negated and we are now
looking at 2 separate crimes.

What is to be ascertained is whether the killing is homicide or qualified to murder. The


Information alleged the aggravating circumstance of treachery. However, we cannot appreciate
this circumstance as the prosecution failed to show proof that the appellants made some
preparation to kill the victim in a manner that would ensure the execution of the crime or make
it impossible or difficult for the person attacked to defend himself. The Information likewise
alleged the aggravating circumstance of evidence premeditation. For this to be appreciated, the
following must be proven: 1) the time when the accused decided to commit the crime; 2) an
overt act manifestly indicating that the accused clung to such determination; and 3) between the
decision and the execution, a sufficient lapse of time that allowed for reflection on the
consequences of the act contemplated. None of these elements have been established in the
case before us. In the absence of any circumstance which would qualify the victim’s killing to
murder, we hold that the appellant should be held liable only for the crime of homicide.
WHEREFORE, in view of these considerations, the Decision of the Court of Appeals in CA-G.R.
CEB-CR-HC No. 00239 is MODIFIED where Appellant Rizaldy Gelle is found GUILTY of the separate
crimes of homicide and theft.

Indeterminate Sentence Law (Death Penalty)

PEOPLE VS. BON


GR No. 166401, October 30, 2006

178
CRIMINAL LAW I – DIGESTED CASES BY: BOAQUINA, MARY CRISCHAEL
179

FACTS: Accused-Appellant was charged with rape on separate 8 informations before the RTC
allegedly raping two minors who are sisters on several instances over a span of 6 years. These
allegations were denied by the accused- appellant. The RTC convicted appellant on all eight (8)
counts of rape. As the penalty imposed consisted of eight (8) death sentences, the records of the
case were automatically elevated to this Court for review. However, in the aftermath of the
pronouncement of the Court in People v. Mateo27 the present case was transferred to the Court
of Appeals for appropriate action and disposition. The same court upheld the 6 rulings of the trial
court and downgraded the other 2 to attempted rape for the reason that there was not a moral
certainty that appellant's penis ever touched the labia of the pudendum of the victim based on
her testimonies. The twin aggravating circumstances of minority and relationship were properly
appreciated in this case. The minority of the victims and their relationship with appellant were
aptly established in the lower court proceedings.

ISSUE: Whether or not death penalty should imposed

HELD: We shall not dwell at length on the proper penalty imposable on appellant for the six (6)
counts of rape. The sentence of death imposed by the RTC and affirmed by the Court of Appeals
can no longer be affirmed in view of Rep. Act No. 9346, titled "An Act Prohibiting the Imposition
of Death Penalty in the Philippines." Section 2 of the law mandates that in lieu of the death
penalty, the penalty of reclusion perpetua shall be imposed. Correspondingly, the Court can no
longer uphold the death sentences imposed by lower courts, but must, if the guilt of the accused
is affirmed, impose instead the penalty of reclusion perpetua, or life imprisonment when
appropriate. Since the passage of Rep. Act No. 9346, the Court has had occasion to effectuate
such reduction in recent cases.
The question of what should be the appropriate penalty for the two (2) counts of attempted rape.
The Court of Appeals had sentenced appellant, for the attempted rape of AAA, to "an
indeterminate penalty of ten (10) years of prision mayor, as minimum, to seventeen (17) years
and four (4) months of reclusion temporal as maximum," for each count of attempted rape. There
is no doubt as to the validity of this sentence at the time it was meted prior to the enactment of
Rep. Act No. 9346. Article 51 of the Revised Penal Code establishes the penalty to be imposed
upon the principals of an attempted felony:
ART. 51. xxx — A penalty lower by two degrees than that prescribed by law for the consummated
felony shall be imposed upon the principals in an attempt to commit a felony.
Article 71 of the Revised Penal Code (Article 71) warrants special attention, crucial as it is to our
disposition of this question.
Art. 71. Graduated scales. — In the case in which the law prescribes a penalty lower or higher by
one or more degrees than another given penalty, the rules prescribed in Article 61 shall be
observed in graduating such penalty.
The lower or higher penalty shall be taken from the graduated scale in which is comprised the
given penalty:
The courts, in applying such lower or higher penalty, shall observe the following graduated scales:
SCALE NO. 1
1. Death

179
CRIMINAL LAW I – DIGESTED CASES BY: BOAQUINA, MARY CRISCHAEL
180

2. Reclusion perpetua
3. Reclusion temporal
4. Prision mayor
Following the scale prescribed in Article 71, the penalty two degrees lower than death is
reclusion temporal, which was the maximum penalty imposed by the Court of Appeals on
appellant for attempted rape. Reclusion temporal is a penalty comprised of three divisible
periods, a minimum, a medium and a maximum. At the same time, the Indeterminate Sentence
Law prescribes that "the court shall sentence the accused to an indeterminate sentence, the
maximum term of which shall be that which, in view of the attending circumstances, could be
properly imposed under the rules of the said Code, and the minimum which shall be within the
range of the penalty next lower to that prescribed by the Code for the offense." The purpose of
the prescription of minimum and maximum periods under the Indeterminate Sentence Law is to
effect the privilege granted under the same law, for prisoners who have served the minimum
penalty to be eligible for parole per the discretion of the Board of Indiscriminate Sentence.58
Thus, convicts sentenced to suffer death penalty or life-imprisonment are ineligible under that
law, as are persons sentenced to reclusion perpetua, an indivisible penalty without minimum or
maximum periods.59

Hence, the Court of Appeals sentenced appellant to suffer the penalty for attempted rape, with
a maximum penalty within the range of reclusion temporal, and a minimum penalty within the
range of the penalty next lower, or prision mayor. If Rep. Act No. 9346 had not been enacted, the
Court would have affirmed such sentence without complication. However, the enactment of the
law has given rise to the problem concerning the imposable penalty. Appellant was sentenced to
a maximum term within reclusion temporal since that is the penalty two degrees lower than
death. With the elimination of death as a penalty, does it follow that appellant should now be
sentenced to a penalty two degrees lower than reclusion perpetua, the highest remaining penalty
with the enactment of Rep. Act No. 9346? If it so followed, appellant would be sentenced to
prision mayor in lieu of reclusion temporal.

PEOPLE V. SIMON
234 SCRA 555

FACTS: Herein accused-appellant Martin Simon y Sunga was charged with a violation of Section
4, Article II of Republic Act No. 6425, as amended, otherwise known as the Dangerous Drugs Act
of 1972 under an indictment alleging that he sold four tea bags of marijuana to a Narcotics
Command (NARCOM) poseur-buyer which tea bags, when subjected to laboratory examination,
were found positive for marijuana. Accused was apprehended during an entrapment operation.

ISSUE: Whether or not the Indeterminate Sentence Law is applicable to the case.

HELD: Apparently, it does, since drug offenses are not included in nor has appellant committed
any act which would put him within the exceptions to said law and the penalty to be imposed
does not involve reclusion perpetua or death, provided, of course, that the penalty as ultimately

180
CRIMINAL LAW I – DIGESTED CASES BY: BOAQUINA, MARY CRISCHAEL
181

resolved will exceed one year of imprisonment. The more important aspect, however, is how the
indeterminate sentence shall be ascertained.

It is true that Section 1 of said law, after providing for indeterminate sentence for an offense
under the Revised Penal Code, states that "if the offense is punished by any other law, the court
shall sentence the accused to an indeterminate sentence, the maximum term of which shall not
exceed the maximum fixed by said law and the minimum shall not be less than the minimum
term prescribed by the same." We hold that this quoted portion of the section indubitably refers
to an offense under a special law wherein the penalty imposed was not taken from and is without
reference to the Revised Penal Code, as discussed in the preceding illustrations, such that it may
be said that the "offense is punished" under that law.
There can be no sensible debate that the aforequoted rule on indeterminate sentence for
offenses under special laws was necessary because of the nature of the former type of penalties
under said laws which were not included or contemplated in the scale of penalties in Article 71
of the Code, hence there could be no minimum "within the range of the penalty next lower to
that prescribed by the Code for the offense," as is the rule for felonies therein. In the illustrative
examples of penalties in special laws hereinbefore provided, this rule applied, and would still
apply, only to the first and last examples. Republic Act No. 7659, has unqualifiedly adopted the
penalties under the Revised Penal Code in their technical terms, hence with their technical
signification and effects. In fact, for purposes of determining the maximum of said sentence, we
have applied the provisions of the amended Section 20 of said law to arrive at prision correccional
and Article 64 of the Code to impose the same in the medium period. Such offense, although
provided for in a special law, is now in effect punished by and under the Revised Penal Code.
Correlatively, to determine the minimum, we must apply the first part of the aforesaid Section 1
which directs that "in imposing a prison sentence for an offense punished by the Revised Penal
Code, or its amendments, the court shall sentence the accused to an indeterminate sentence the
maximum term of which shall be that which, in view of the attending circumstances, could be
properly imposed under the rules of said Code, and the minimum which shall be within the range
of the penalty next lower to that prescribed by the Code for the offense."

Extinction of Criminal Liability (Article 89-99)


Death of the Convict

PEOPLE V. DUMLAO
G.R. No. 168918, March 2, 2009

FACTS: The accused Hermenegildo C. Dumlao, Aber Canlas, Jacobo C. Clave, Roman A. Cruz, Jr.,
and Fabian C. Ver, being then the members of the Board of Trustees of the Government Service
Insurance System (GSIS) which is a government corporation and therefore all public officers,
conspiring and confederating together and mutually helping one another, while in the

181
CRIMINAL LAW I – DIGESTED CASES BY: BOAQUINA, MARY CRISCHAEL
182

performance of their official functions, did then and there willfully, unlawfully and criminally
enter into contract of lease-purchase with Emilio G. La’o, a private person whereby the GSIS
agreed to sell to said Emilio G. La’o, a GSIS acquired property consisting of three parcels of land
with an area of 821 square meters together with a 5-storey building for the sum of ₱2,000,000.00
with a down payment of ₱200,000.00 with the balance payable in fifteen years at 12% interest
per annum compounded yearly, with a yearly amortization of ₱264,278.37 including principal
and interest granting Emilio G. La’o the right to sub-lease the ground floor for his own account
during the period of lease, from which he collected yearly rentals in excess of the yearly
amortization which contract is manifestly and grossly disadvantageous to the government.

ISSUE: Whether death or acquittal of co-conspirator/s extinguishes criminal liability of all.

HELD: Respondent Dumlao maintains he was charged with conspiring with the other GSIS Board
Members in approving the Lease-Purchase Agreement. However, of the seven members, two
died, two were acquitted and the other two were not charged. He was left alone. He argues that
since a conspiracy requires two or more persons agreeing to commit a crime, he can no longer
be charged because he was left alone to face a charge of conspiracy.
His assumption that he can no longer be charged because he was left alone -- since the co-
conspirators have either died, have been acquitted or were not charged -- is wrong. A conspiracy
is in its nature a joint offense. One person cannot conspire alone. The crime depends upon the
joint act or intent of two or more person. Yet, it does not follow that one person cannot be
convicted of conspiracy. As long as the acquittal or death of a co-conspirator does not remove
the basis of a charge of conspiracy, one defendant may be found guilty of the offense.32 In the
case at bar, the absence or presence of conspiracy is again factual in nature and involves
evidentiary matters. The same is better left ventilated before the trial court during trial, where
the parties can adduce evidence to prove or disprove its presence.
We agree. However, said issue has already been mooted by the death of respondent La’o.20 The
death of an accused prior to final judgment terminates his criminal as well as civil liability based
solely thereon.21 Accordingly, the case against respondent La’o was dismissed.
WHEREFORE, premises considered, the instant petition is GRANTED. The resolution of the
Sandiganbayan in Criminal Case No. 16699 dated 14 July 2005 granting the Motion to
Dismiss/Quash of respondent Hermenegildo C. Dumlao, is hereby REVERSED and SET ASIDE. The
Sandiganbayan is forthwith DIRECTED to set the case for the reception of evidence for the
prosecution.
As to respondent Emilio G. La’o, on account of his demise, the case against him is DISMISSED.

REPUBLIC V. DESIERTO
G.R. No. 131966, August 31, 2005

FACTS: This resolves the Motion for Reconsideration and the Second Motion for Reconsideration
filed by private respondent Cojuangco, Jr. as well as the Motion for Reconsideration filed by
petitioner Republic of the Philippines.

182
CRIMINAL LAW I – DIGESTED CASES BY: BOAQUINA, MARY CRISCHAEL
183

On September 23, 2002, the Court, in the above-entitled case, granted the petition for certiorari
filed by the Republic of the Philippines. The resolution of the Ombudsman in OMB-0-90-2811
dismissing the Republic’s complaint was set aside, and the Ombudsman was ordered to proceed
with the preliminary investigation in said case and to exclude respondents Teodoro D. Regala and
Jose C. Concepcion as defendants. Respondent Eduardo M. Cojuangco, Jr. and petitioner Republic
of the Philippines then filed their respective motions for reconsideration of the aforesaid
decision. Both were denied by the Court in its Resolution dated August 16, 2004.
Respondent Cojuangco, Jr. filed a second motion for reconsideration on the ground, among
others, that the Resolution dated August 16, 2004, was promulgated without the requisite
division of five as required by A.M. No. 99-8-09-SC. The Court in its Resolution dated April 25,
2005 recalled the Resolution dated August 16, 2004 and the (first) Motion for Reconsideration
together with the Second Motion for Reconsideration is deemed submitted for resolution.

ISSUE: Whether death before final judgment extinguishes criminal liability.

HELD: During the pendency of this petition, respondent Maria Clara L. Lobregat died on January
2, 2004.19 The death of an accused prior to final judgment terminates his criminal liability as well
as the civil liability based solely thereon.20 Consequently, the case against respondent Lobregat
should be dismissed.

WHEREFORE, the first and second Motions for Reconsideration filed by private respondent
Eduardo M. Cojuangco, Jr., and the motion for reconsideration of petitioner Republic of the
Philippines are hereby DENIED with FINALITY.
The Decision dated September 23, 2002 is MODIFIED to the effect that the charges against
deceased respondent Maria Clara L. Lobregat in OMB-0-90-2811, pending preliminary
investigation before the Office of the Ombudsman, is ordered dismissed.

Pardon and Amnesty

PEOPLE V. PATRIARCA
GR 135457, September 29, 2000

FACTS: An information for murder was filed against Jose Patriarca, Jr., alias "Ka Django," "Carlos
Narra", "Ka Jessie," et al., charging them of murder, they were members of the New People’s
Army. The above-named accused conspiring, confederating and mutually helping one another,
armed with guns, forcibly took away Alfredo Arevalo from his residence and brought him to Sitio
Abre, Mabini, Donsol, Sorsogon, and did then and there willfully, unlawfully and feloniously with
intent to kill, with treachery and evident premeditation, attack, assault and shoot Alfredo Arevalo
thereby inflicting upon him mortal wounds, which directly caused his death to the damage and
prejudice of his legal heirs. Accused-Appellant was also charged with Murder for the killing of
one Rudy de Borja and a certain Elmer Cadag on 2 separate informations. A decision was
rendered convicting the accused. Accused-appellant applied for amnesty under Proclamation No.
724 amending Proclamation No. 347, dated March 25, 1994, entitled "Granting Amnesty to

183
CRIMINAL LAW I – DIGESTED CASES BY: BOAQUINA, MARY CRISCHAEL
184

Rebels, Insurgents, and All Other Persons Who Have or May Have Committed Crimes Against
Public Order, Other Crimes Committed in Furtherance of Political Ends, and Violations of the
Article of War, and Creating a National Amnesty Commission." His application was favorably
granted by the National Amnesty Board.

ISSUE: Whether Patriarca is guilty of the crime of murder, an offense committed in pursuance or
in furtherance of rebellion
HELD: Amnesty commonly denotes a general pardon to rebels for their treason or other high
political offenses, or the forgiveness which one sovereign grants to the subjects of another, who
have offended, by some breach, the law of nations. Amnesty looks backward, and abolishes and
puts into oblivion, the offense itself; it so overlooks and obliterates the offense with which he is
charged, that the person released by amnesty stands before the law precisely as though he had
committed no offense.

Paragraph 3 of Article 89 of the Revised Penal Code provides that criminal liability is totally
extinguished by amnesty, which completely extinguishes the penalty and all its effects. In the
case of People vs. Casido,9 the difference between pardon and amnesty is given:

"Pardon is granted by the Chief Executive and as such it is a private act which must be pleaded
and proved by the person pardoned, because the courts take no notice thereof; while amnesty
by Proclamation of the Chief Executive with the concurrence of Congress, is a public act of which
the courts should take judicial notice. Pardon is granted to one after conviction; while amnesty
is granted to classes of persons or communities who may be guilty of political offenses, generally
before or after the institution of the criminal prosecution and sometimes after conviction. Pardon
looks forward and relieves the offender from the consequences of an offense of which he has
been convicted, that is, it abolishes or forgives the punishment, and for that reason it does 'not
work the restoration of the rights to hold public office, or the right of suffrage, unless such rights
be expressly restored by the terms of the pardon,' and it 'in no case exempts the culprit from the
payment of the civil indemnity imposed upon him by the sentence' (Article 36, Revised Penal
Code). While amnesty looks backward and abolishes and puts into oblivion the offense itself, it
so overlooks and obliterates the offense with which he is charged that the person released by
amnesty stands before the law precisely as though he had committed no offense."
This Court takes judicial notice of the grant of amnesty upon accused-appellant Jose N. Patriarca,
Jr. Once granted, it is binding and effective. It serves to put an end to the appeal.10
WHEREFORE, IN VIEW OF THE FOREGOING, the decision of the Regional Trial Court at Sorsogon,
Sorsogon, Branch 52 in Criminal Case No. 2773 is REVERSED and SET ASIDE. Accused-appellant
Jose N. Patriarca, Jr. is hereby ACQUITTED of the crime of murder.

Prescription
DOMINGO V. SANDIGANBAYAN

184
CRIMINAL LAW I – DIGESTED CASES BY: BOAQUINA, MARY CRISCHAEL
185

GR No 109376; January 20, 2000

FACTS: Accused Panfilo Domingo, then the President of PNB conspire with Rodolfo Cuenca, the
Construction and Development Corporation of the Philippines (CDCP) while taking advantage of
their close personal relation with President Ferdinand E. Marcos wherein the act of the accused
inflicted undue injury and prejudice to PNB which was unjustly forced to assume CDCP’s
obligation to the Republic National Bank of Dallas after the latter had defaulted in the payment
thereof, amounting to US $29 Million, and which likewise granted unwarranted benefits to CDCP
in the same amount. On February 19, 1993 petitioner filed a motion to quash the information
against him on the grounds that (1) the criminal action or liability has been extinguished by
prescription and (2) the facts charged do not constitute an offense.

ISSUE: Whether or not granting a writ of injunction or prohibition to stay a criminal proceeding
would be proper

HELD: No. Writs of injunction and prohibition will not lie to restrain a criminal prosecution
because public interest requires that criminal acts be immediately investigated and prosecuted
for the protection of society. The writ may issue only in specified cases (e.g., to prevent the use
of the strong arm of the law in an oppressive manner and to afford adequate protection to
constitutional rights). Such exceptions do not obtain in this case.
The petition in this case is dismissed. The Sandiganbayan is directed to try and decide Criminal
Case No. 17847 with purposeful dispatch.1âwphi1.nêt

PEOPLE V. PACIFADOR
G.R. No. 139405

FACTS: On October 27, 1988, Arturo F. Pacificador then Chairman of the Board of the National
Shipyard and Steel Corporation, a government-owned corporation, and therefore, a public officer
was charged before the Sandiganbayan with the crime of violation of Republic Act No. 3019, as
amended, otherwise known as the Anti-Graft and Corrupt Practices Act. The crime was
committed from December 6, 1975 to January 6, 1976, in Metro Manila by Pacificador. After his
arraignment, the respondent filed a Motion to Dismiss on the ground of prescription of the
offense. Sandiganbayan on November 10, 1998 dismissed the Information against the
respondent on the ground of prescription. The Urgent Motion for Reconsideration of the
Solicitor General was denied by the Sandiganbayan. Republic Act No. 3019 provides for its own
prescriptive period. Section 11 of R.A. No. 3019, as amended by B.P. Blg. 195, provides that the
offenses committed under the said statute shall prescribe in fifteen (15) years. It appears
however, that prior to the amendment of Section 11 of R.A. No. 3019 by B.P. Blg. 195 which was

185
CRIMINAL LAW I – DIGESTED CASES BY: BOAQUINA, MARY CRISCHAEL
186

approved on March 16, 1982, the prescriptive period for offenses punishable under the said
statute was only ten (10) years.

ISSUE: Whether or not the crime had not yet prescribed as the special law governing the same
have been amended increasing the prescriptive period from ten (10) to fifteen (1) years.

HELD: The longer prescriptive period of fifteen (15) years, as provided in Section 11 of R.A. No.
3019 as amended by B.P. Blg. 195, does not apply in this case for the reason that the amendment,
not being favorable to the accused, cannot be given retroactive effect. Hence the crime
prescribed on January 6, 1986 or ten (10) years from January 6, 1976. The crime had already
prescribed when the Information in this case was filed with the Sandiganbayan on October 27,
1988.It bears emphasis, as held in a number of cases that in the interpretation of the law on
prescription of crimes, that which is more favorable to the accused is to be adopted. The said
legal principle takes into account the nature of the law on prescription of crimes which is an act
of amnesty and liberality on the part of the state in favor of the offender.

SERMONIA V. CA
G.R. No. 109454 June 14, 1994

FACTS: On 26 May 1992, petitioner Jose C. Sermonia was charged with bigamy before the RTC of
Pasig, Br. 151, for contracting marriage with Ma. Lourdes Unson on 15 February 1975 while his
prior marriage to Virginia C. Nievera remained valid and subsisting.
Petitioner moved to quash the information on the ground that his criminal liability for bigamy
has been extinguished by prescription. In the order of 1 October 1992, respondent judge denied
the motion to quash. On 27 October 1992, he likewise denied the motion to reconsider his order
of denial.
Petitioner challenged the above orders before the Court of Appeals through a petition
for certiorari and prohibition. In the assailed decision of 21 January 1993, his petition was
dismissed for lack of merit. In this recourse, petitioner contends that his criminal liability for
bigamy has been obliterated by prescription. He avers that since the second marriage contract
was duly registered with the Office of the Civil Registrar in 1975, such fact of registration makes
it a matter of public record and thus constitutes notice to the whole world. The offended party
therefore is considered to have had constructive notice of the subsequent marriage as of 1975;
hence, prescription commenced to run on the day the marriage contract was registered. For this
reason, the corresponding information for bigamy should have been filed on or before 1990 and
not only in 1992.

186
CRIMINAL LAW I – DIGESTED CASES BY: BOAQUINA, MARY CRISCHAEL
187

On the other hand, the prosecution maintains that the prescriptive period does not begin from
the commission of the crime but from the time of discovery by complainant which was in July
1991.

ISSUE: Whether or not the prosecution of Jose C. Sermonia for bigamy has already prescribed.

HELD: No. The non-application to the crime of bigamy of the principle of constructive notice is
not contrary to the well-entrenched policy that penal laws should be construed liberally in favor
of the accused. To compute the prescriptive period for the offense of bigamy from registration
thereof would amount to almost absolving the offenders thereof for liability therefor. While the
celebration of the bigamous marriage may be said to be open and made of public record by its
registration, the offender however is not truthful as he conceals from the officiating authority
and those concerned the existence of his previous subsisting marriage. He does not reveal to
them that he is still a married person. He likewise conceals from his legitimate spouse his
bigamous marriage. And for these, he contracts the bigamous marriage in a place where he is not
known to be still a married person. And such a place may be anywhere, under which
circumstance, the discovery of the bigamous marriage is rendered quite difficult and would take
time. It is therefore reasonable that the prescriptive period for the crime of bigamy should be
counted only from the day on which the said crime was discovered by the offended party, the
authorities or their agency.

ROMUALDEZ V. CA
G.R. Nos. 165510-33 ; July 28, 2006

FACTS: Romualdez is being charged with violations of Section 7 of RA No. 3019 for failure to file
his Statements of Assets and Liabilities for the period 1967-1985 during his tenure as Ambassador
Extraordinary and Plenipotentiary and for the period 1963-1966 during his tenure as Technical
Assistant in the Department of Foreign Affairs. Romualdez claims that the Office of the
Ombudsman gravely abused its discretion in recommending the filing of 24 informations against
him for violation of Section 7 of Republic Act (RA) No. 3019 or the Anti-Graft and Corrupt Practices
Act; Romualdez asserts that the Ombudsman (Marcello) cannot revive the aforementioned cases
which were previously dismissed by the Sandiganbayan in its Resolution of February 10, 2004.
He also claims that the case should be dismissed on the ground of prescription.
The Ombudsman, however, contends that the dismissal of the information in Criminal Case Nos.
13406-13429 does not mean that petitioner was thereafter exempt from criminal prosecution,
and that new information may be filed by the Ombudsman should it find probable cause in the
conduct of its preliminary investigation. He also claimed that that the filing of the complaint with
the Presidential Commission on Good Government (PCGG) in 1987 and the filing of the

187
CRIMINAL LAW I – DIGESTED CASES BY: BOAQUINA, MARY CRISCHAEL
188

information with the Sandiganbayan in 1989 interrupted the prescriptive period and that the
absence of the petitioner from the Philippines from 1986 until 2000 also interrupted the
aforesaid period based on Article 91 of the Revised Penal Code.
The PCGG avers that the Omdudsman need not wait for a new complaint with a new docket
number for it to conduct a preliminary investigation on the alleged offenses of the petitioner.
And since both RA No. 3019 and Act No. 3326 (the Act To Establish Periods of Prescription For
Violations Penalized By Special Acts and Municipal Ordinances and to Provide When Prescription
Shall Begin To Run) are silent as to whether prescription should begin to run when the offender
is absent from the Philippines, the RPC should be applied.
ISSUE: whether or not the offenses charged in the subject criminal cases have prescribed
HELD: In resolving the issue of prescription of the offense charged, the following should be
considered: (1) the period of prescription for the offense charged; (2) the time the period of
prescription starts to run; and (3) the time the prescriptive period was interrupted. In such a
situation, Act No. 3326 must prevail over Article 91 because it specifically and directly applies to
special laws while the Revised Penal Code shall apply... to special laws only suppletorily and only
when the latter do not provide the contrary. Indeed, elementary rules of statutory construction
dictate that special legal provisions must prevail over general ones.
In view of the foregoing, the applicable 10-and-15-year prescriptive periods in the instant case,
were not interrupted by any event from the time they began to run on May 8, 1987. As a
consequence, the alleged offenses committed by the petitioner for the years 1963-1982...
prescribed 10 years from May 8, 1987 or on May 8, 1997. On the other hand, the alleged offenses
committed by the petitioner for the years 1983-1985 prescribed 15 years from May 8, 1987 or
on May 8, 2002.
Therefore, when the Office of the Special Prosecutor initiated the preliminary investigation of
Criminal Case Nos. 13406-13429 on March 3, 2004 by requiring the petitioner to submit his
counter-affidavit, the alleged offenses subject therein have already prescribed.

Probation Law

LLAMADO V. CA
G.R. No. 84850; June 29, 1989

FACTS: Petitioner Ricardo A. Llamado was Treasurer of Pan Asia Finance Corporation. Together
with Jacinto N. Pascual, Sr., President of the same corporation, petitioner Llamado was
prosecuted for violation of Batas Pambansa Blg. 22 in Criminal Case No. 85-38653, Regional Trial
Court of Manila, Branch 49. The two (2) had co-signed a postdated check payable
to private respondent Leon Gaw in the amount of P186,500.00, which check was dishonored for
lack of sufficient funds. In a decision dated 10 March 1987, the trial court convicted the petitioner

188
CRIMINAL LAW I – DIGESTED CASES BY: BOAQUINA, MARY CRISCHAEL
189

alone, since jurisdiction over the person of Pascual, who had thoughtfully fled the country, had
not been obtained. Petitioner was sentenced to imprisonment for a period of one (1) year of
prision correccional and to pay a fine of P 200,000.00 with subsidiary imprisonment in case of
insolvency. Petitioner was also required to reimburse respondent Gaw the amount
ofP186,500.00 plus the cost of suit. He sought advice from another counselor that also assisted
him in filing a probation petition invoking Presidential Decree No. 968, as amended. The Petition
was not, however, accepted by the lower court, since the records of the case had already been
forwarded to the Court of Appeals.
ISSUE: Whether or not petitioner's application for probation which was filed after a notice of
appeal had been filed with the trial court, after the records of the case had been forwarded to
the Court of Appeals and the Court of Appeals had issued the notice to file Appellant's Brief, after
several extensions of time to file Appellant's Brief had been sought from and granted by the Court
of Appeals but before actual filing of such brief, is barred under P.D. No. 968,as amended.
HELD: Yes, the application for probation is barred under the law. There were two amendments
that happened to the law, and the present law allows applications for probation “after the TC
shall have convicted and sentenced a defendant and —within the period of perfecting
an appeal”. It prohibits the grant of an application for probation if the defendant has perfected
an appeal from the judgment of conviction. Probation Law of 1976, was promulgated on 24 July
1976. Section 4 of this statute provided as follows:
Sec. 4. Grant of Probation. Subject to the provisions of this Decree, the court may, after it shall
have convicted and sentenced a defendant and upon application at any time of said
defendant, suspend the execution of said sentence and place the defendant on probation for
such period and upon such terms and conditions as it may deem best.
First amendment: PD 1257- “After the trial court shall have convicted and sentenced a defendant
but before he begins to serve his sentence.” The cut-off time — the commencement of service
of sentence— takes place not only after an appeal has been taken from the sentence of
conviction, but even after judgment has been rendered by the appellate court and
after judgment has become final.- In this last situation, it provides that “the application for
probation shall be acted upon by the trial court on the basis of the judgment of the appellate
court”

Second Amendment: PD 1990- “after the shall have convicted and sentenced a defendant
and — within the period of perfecting an appeal”. It prohibits the grant of an application for
probation if the defendant has perfected an appeal from the judgment of conviction. The court
has lost jurisdiction over the case when petitioner perfected his appeal. The oral manifestation
made after judgment was rendered was considered by the RTC as being equal to a written notice
of appeal.

BALA V. MARTINEZ

189
CRIMINAL LAW I – DIGESTED CASES BY: BOAQUINA, MARY CRISCHAEL
190

G.R. 67301 January 29, 1990

FACTS: The petitioner had been indicted for removing and substituting the picture of Diazen
which had been attached to her United States of America passport, with that of Notarte, in effect
falsifying a genuine public or official document. The trial court adjudged petitioner Bala in
Criminal Case No. 24443, guilty of the crime of falsification of a public document. The petitioner
seasonably appealed, but the Court of Appeals, on April 9, 1980, affirmed in toto the lower
court's decision. After the case had been remanded to the court of origin for execution of
judgment, the Petitioner applied for and was granted probation by the respondent judge in his
order dated August 11, 1982. The petitioner was then placed under probation for a period of one
(1) year, subject to the terms and conditions enumerated therein.

The probationer (petitioner) asked his supervising probation officer for permission to transfer his
residence from BF Homes to Phil-Am Life Subdivision in Las Pinas specifically 33 Jingco Street. The
probation officer verbally granted the probationer's request as he found nothing objectionable
to it.- By the terms of the petitioner's probation, it should have expired on August 10, 1983, one
year after the order granting the same was issued. But, the order of final discharge could not
be issued because the respondent probation officer had not yet submitted his final report on the
conduct of his charge. Subsequently, the respondent filed a motion to revoke the probation of
the petitioner before Branch XX of the Regional Trial Court (RTC) of Manila, presided over by the
respondent judge. The motion alleged that the petitioner had violated the terms and conditions
of his probation.

On January 4, 1984, the petitioner filed his opposition to the motion on the ground that he was
no longer under probation, his probation period having terminated on August 10, 1983, as
previously adverted to. As such, no valid reason existed to revoke the same, he contended. As if
to confirm the Manila Assistant City Fiscal's motion to revoke the petitioner’s probation, the
respondent probation officer filed on January 6, 1984, a motion to terminate Manuel Bala's
probation, at the same time attaching his progress report on supervision dated January 5, 1984.
The same motion, however, became the subject of a "Manifestation," dated January 10,1984,
which stated that the probation officer was not pursuing the motion to terminate dated January
6,1984; instead, he was submitting
a supplementalreport which recommended the revocation of probation "in the light of new
facts, information, and evidences."- Thereafter, the petitioner filed a motion to dismiss and/or
strike out the motion to revoke probation, questioning the jurisdiction of the court over his case
inasmuch as his probation
period had alreadyexpired. Moreover, his change of residenceautomatically transferred the
venue of the case from the RTC of Manila to the Executive. Judge, of the RTC of Makati which
latter court include under its jurisdiction the Municipality of Las Piñas the probationer's place of
residence, invoking Section 13,P.D. No. 968, which provides Sec. 13.

190
CRIMINAL LAW I – DIGESTED CASES BY: BOAQUINA, MARY CRISCHAEL
191

ISSUE: WON his transfer of residence automatically transferred jurisdiction over his probation
from the Manila Regional Trial Court to the same court in his new address.

HELD: NO- In criminal cases, venue is an element of jurisdiction. Such being the case,
the Manila RTC would not be deprived of its jurisdiction over the probation case. To uphold the
petitioner's contention would mean a depreciation of the Manila court's power to grant
probation in the first place. It is to be remembered that when the petitioner-accused applied for
probation in the then CFI of Manila, he was a resident of Las Piñas as he is up to now, although
in a different subdivision. As pointed out earlier, he merely moved from BF Homes to Philam Life
Subdivision 33 Jingco Street, also in Las Piñas. On the other hand, pursuing the petitioner's
argument on this score to the limits of it logic would mean that his probation was null and void
in the place, because then the Manila CFI was without jurisdiction to grant him
probation as he was a resident of Las Piñas. It is therefore incorrect to
assume that the petitioner's change of abode compels change of venue, and necessarily, control
over the petitioner, to the Executive Judge of the RTC of his new residence. Thus, in the
apportionment of the regional trial courts under Batas Pambansa Blg.129, otherwise known as
the Judiciary ReorganizationAct of 1980, Las Piñas is one among themunicipalities included in
the National Capital Judicial Region (Metro Manila) with a seat at Makati.

Needless to say, the Regional Trial Court in Makati,like the Manila Regional Trial Court, forms part
of the Regional Trial Court of the National Capital Region. Accordingly, the various branches of
the regional trial courts of Makati or Manila under the National Capital
Region, are coordinate and co-equal courts, the totality of which is only one Regional Trial
Court. Jurisdiction is vested in the court, not in the judges. In other words, the case does
not attach to the branch or judge. Therefore, in this case, RTC Branch XX of Manila, which granted
the probation, has not lost control and supervision over the probation of the petitioner.

SALGADO V. CA
G.R. No. 89606; August 30, 1990

FACTS: Petitioner was found guilty with the crime of serious physical injuries before the RTC after
trial. Subsequently, the accused filed an application for probation with the trial court. The
application was granted in an Order which contained the condition to indemnify the victim
FRANCISCO LUKBAN, JR., in a monthly installment of P2,000.00 every month during the entire
period of his probation. From May to October, petitioner complied. Private respondent Francisco
Lukban, Jr. filed a motion for the issuance of a writ of execution for the enforcement of the civil
liability adjudged in his favor in the criminal case and it was granted. A motion for reconsideration

191
CRIMINAL LAW I – DIGESTED CASES BY: BOAQUINA, MARY CRISCHAEL
192

was filed by petitioner but it was denied. Then, the petitioner filed directly with the Supreme
Court a petition for review of the trial court's order granting the motion for issuance of a writ of
execution. It was referred to the Court of Appeals which they affirmed.
The petitioner went to this Court via a petition for review and argued that the CA erred in holding
the order as far as the civil aspect is concerned and in holding that the condition in the probation
order modifying or altering the civil liability of the offender is unauthorized and not sanction by
law.

ISSUE: Whether or not the trial court may impose as a condition of probation the manner in
which a probationer may settle his civil liability against the offended party during the period of
probation.

HELD: Under Section 4 of Presidential Decree No. 968 (Probation Law of 1976) provides:
Sec. 4. Grant of Probation. — Subject to the provisions of this Decree, the court may, after it shall
have convicted and sentenced a defendant but before he begins to serve his sentence and upon
his application, suspend the execution of said sentence and place the defendant on probation for
such period and upon such terms and conditions as it may deem best.
The trial court is given the discretion to impose conditions in the order granting probation "as it
may deem best." As already stated, it is not only limited to those listed under Section 10 of the
Probation Law. Thus, under Section 26, paragraph (d) of the Rules on Probation Methods and
Procedures, among the conditions which may be imposed in the order granting probation is:
Sec. 26. Other conditions of Probation. The Probation Order may also require the probationer in
appropriate cases to. . . comply with a program of payment of civil liability to the victim or his
heirs . .
However, this is not to say that the manner by which the probationer should satisfy the payment
of his civil liability in a criminal case during the probation period may be demanded at will by him.
It is necessary that the condition which provides for a program of payment of his civil liability will
address the offender's needs and capacity. Such need may be ascertained from the findings and
recommendations in the post-sentence investigation report submitted by the Probation Officer
after investigation of the financial capacity of the offender and that such condition is to the end
that the interest of the state and the reformation of the probationer is best served.
In the instant case, in the absence of any showing to the contrary, it is presumed that when the
trial court issued the order of April 15, 1987, the condition that the petitioner has to pay private
respondent P2,000.00 a month for the satisfaction of the civil liability adjudged against him was
recommended by the probation officer who prepared the post-sentence investigation and that
such condition is, in the judgment of the trial court, "deemed best" under the circumstances.

192
CRIMINAL LAW I – DIGESTED CASES BY: BOAQUINA, MARY CRISCHAEL
193

MORENO vs. COMELEC


G.R. No. 168550. August 10, 2006

FACTS: Norma L. Mejes (Mejes) filed a petition to disqualify Moreno from running for Punong
Barangay on the ground that the latter was convicted by final judgment of the crime of Arbitrary
Detention. The Comelec en banc granted her petition and disqualified Moreno. Moreno filed an
answer averring that the petition states no cause of action because he was already granted
probation. Allegedly, following the case of Baclayon v. Mutia, the imposition of the sentence of
imprisonment, as well as the accessory penalties, was thereby suspended. Moreno also argued
that under Sec. 16 of the Probation Law of 1976 (Probation Law), the final discharge of the
probation shall operate to restore to him all civil rights lost or suspended as a result of his
conviction and to fully discharge his liability for any fine imposed.
However, the Comelec en banc assails Sec. 40(a) of the Local Government Code which provides
that those sentenced by final judgment for an offense involving moral turpitude or for an offense
punishable by one (1) year or more of imprisonment, within two (2) years after serving sentence,
are disqualified from running for any elective local position. Since Moreno was released from
probation on December 20, 2000, disqualification shall commence on this date and end two (2)
years thence. The grant of probation to Moreno merely suspended the execution of his sentence
but did not affect his disqualification from running for an elective local office.
On his petition, Moreno argues that the disqualification under the Local Government Code
applies only to those who have served their sentence and not to probationers because the latter
do not serve the adjudged sentence. The Probation Law should allegedly be read as an exception
to the Local Government Code because it is a special law which applies only to probationers.
Further, even assuming that he is disqualified, his subsequent election as Punong Barangay
allegedly constitutes an implied pardon of his previous misconduct.
ISSUE: Does Moreno’s probation grant him the right to run in public office?
HELD: Yes. Sec. 16 of the Probation Law provides that "[t]he final discharge of the probationer
shall operate to restore to him all civil rights lost or suspended as a result of his conviction and
to fully discharge his liability for any fine imposed as to the offense for which probation was
granted." Thus, when Moreno was finally discharged upon the court's finding that he has fulfilled
the terms and conditions of his probation, his case was deemed terminated and all civil rights
lost or suspended as a result of his conviction were restored to him, including the right to run for
public office.
It is important to note that the disqualification under Sec. 40(a) of the Local Government Code
covers offenses punishable by one (1) year or more of imprisonment, a penalty which also covers
probationable offenses. In spite of this, the provision does not specifically disqualify probationers
from running for a local elective office.
Probation Law should be construed as an exception to the Local Government Code. While the
Local Government Code is a later law which sets forth the qualifications and disqualifications of

193
CRIMINAL LAW I – DIGESTED CASES BY: BOAQUINA, MARY CRISCHAEL
194

local elective officials, the Probation Law is a special legislation which applies only to
probationers. It is a canon of statutory construction that a later statute, general in its terms and
not expressly repealing a prior special statute, will ordinarily not affect the special provisions of
such earlier statute.
COLINARES VS. PEOPLE
G.R. No. 182748, December 13, 2011

FACTS: Accused-appellant Arnel Colinares (Arnel) was charged with frustrated homicide for
hitting the head of the private complainant with a piece of stone. He alleged self-defense but the
trial court found him guilty of the crime charged and sentenced him to suffer imprisonment from
2 years and 4 months of prision correccional, as minimum, to 6 years and 1 day of prision mayor,
as maximum. Since the maximum probationable imprisonment under the law was only up to 6
years, Arnel did not qualify for probation.
Arnel appealed to the Court of Appeals (CA), invoking self-defense and, alternatively, seeking
conviction for the lesser crime of attempted homicide with the consequent reduction of the
penalty imposed on him. His conviction was affirmed by the CA. Hence, this appeal to the
Supreme Court.
ISSUE: Given a finding that Arnel is entitled to conviction for a lower [lesser] offense [of
attempted homicide] and a reduced probationable penalty, may he may still apply for probation
on remand of the case to the trial court?
HELD: YES, Arnel may still apply for probation on remand of the case to the trial court.
Ordinarily, Arnel would no longer be entitled to apply for probation, he having appealed from the
judgment of the RTC convicting him for frustrated homicide. But, the Court finds Arnel guilty only
of the lesser crime of attempted homicide and holds that the maximum of the penalty imposed
on him should be lowered to imprisonment of four months of arresto mayor, as minimum, to two
years and four months of prision correccional, as maximum. With this new penalty, it would be
but fair to allow him the right to apply for probation upon remand of the case to the RTC.
While it is true that probation is a mere privilege, the point is not that Arnel has the right to such
privilege; he certainly does not have. What he has is the right to apply for that privilege. The
Court finds that his maximum jail term should only be 2 years and 4 months. If the Court allows
him to apply for probation because of the lowered penalty, it is still up to the trial judge to decide
whether or not to grant him the privilege of probation, taking into account the full circumstances
of his case.
If the Court chooses to go by the dissenting opinion’s hard position, it will apply the probation
law on Arnel based on the trial court’s annulled judgment against him. He will not be entitled to
probation because of the severe penalty that such judgment imposed on him. More, the
Supreme Court’s judgment of conviction for a lesser offense and a lighter penalty will also have
to bend over to the trial court’s judgment—even if this has been found in error. And, worse,

194
CRIMINAL LAW I – DIGESTED CASES BY: BOAQUINA, MARY CRISCHAEL
195

Arnel will now also be made to pay for the trial court’s erroneous judgment with the forfeiture
of his right to apply for probation. Ang kabayo ang nagkasala, ang hagupit ay sa kalabaw (the
horse errs, the carabao gets the whip). Where is justice there?
Here, Arnel did not appeal from a judgment that would have allowed him to apply for
probation. He did not have a choice between appeal and probation. He was not in a position to
say, “By taking this appeal, I choose not to apply for probation.” The stiff penalty that the trial
court imposed on him denied him that choice. Thus, a ruling that would allow Arnel to now seek
probation under this Court’s greatly diminished penalty will not dilute the sound ruling
in Francisco. It remains that those who will appeal from judgments of conviction, when they have
the option to try for probation, forfeit their right to apply for that privilege.

In a real sense, the Court’s finding that Arnel was guilty, not of frustrated homicide, but only of
attempted homicide, is an original conviction that for the first time imposes on him a
probationable penalty. Had the RTC done him right from the start, it would have found him guilty
of the correct offense and imposed on him the right penalty of two years and four months
maximum. This would have afforded Arnel the right to apply for probation.

The Supreme Court voted to PARTIALLY GRANT the appeal, MODIFIED the CA decision and found
Arnel GUILTY of ATTEMPTED (not frustrated) HOMICIDE and SENTENCED him to and
indeterminate but PROBATIONABLE penalty of 4 months of arresto mayor as minimum and 2
years and 4 months of prision correccional as maximum. The Court also voted 8-7 to allow Arnel
to APPLY FOR PROBATION within 15 days from notice that the record of the case has been
remanded for execution to trial court.

SABLE V. PEOPLE
G.R. No. 17796: April 7, 2009

FACTS: Petitioner convicted of the crime of Falsification of Public Documents under Article 172(1)
in relation to Article 171 of the Revised Penal Code on November 28, 2000 but acquitted Ildefonsa
Anoba for finding not guilty. However, the court finds that Lourdes Abellanosa Sable was guilty
beyond reasonable doubt of the crime charged and hereby sentences her to suffer an
indeterminate penalty of 4 years ,2 months and one day to 6 years.
On August, 25, 2003 petitioner intimated her desire to apply for probation instead of appealing
the judgment of conviction which was denied.

ISSUE: Whether or not the denial of application for probation is tenable.

195
CRIMINAL LAW I – DIGESTED CASES BY: BOAQUINA, MARY CRISCHAEL
196

HELD: The court held that probation is a special privilege granted by the state to a penitent
qualified offender. It essentially rejects appeals and encourages an otherwise eligible convict to
immediately admit his liability and save the state the time, effort and expenses to jettison an
appeal.
The pertinent provision of the Probation Law, as amended, reads:
Sec. 4. Grant of Probation.—Subject to the provisions of this Decree, the trial court may,
after it shall have convicted and sentenced a defendant and upon application by said
defendant within the period for perfecting an appeal, suspend the execution of the
sentence and place the defendant on probation for such period and upon such terms and
conditions as it may deem best; Provided, That no application for probation shall be
entertained or granted if the defendant has perfected the appeal from the judgment of
conviction.
Probation may be granted whether the sentence imposes a term of imprisonment or a fine only.
An application for probation shall be filed with the trial court. The filing of the application shall
be deemed a waiver of the right to appeal.
It is quite clear from the afore-quoted provision that an application for probation must be made
within the period for perfecting an appeal, and the filing of the application after the time of
appeal has lapsed is injurious to the recourse of the applicant.
In the present petition before us, petitioner filed the application for probation on 25 August 2003,
almost eight months from the time the assailed judgment of the RTC became final. Clearly, the
application for probation was filed out of time pursuant to Rule 122, Sec. 6 of the Rules of Court,
which states that an "appeal must be taken within fifteen (15) days from promulgation of the
judgment or from notice of the final order appealed from."
In Palo v. Militante, this Court held that what the law requires is that the application for probation
must be filed within the period for perfecting an appeal. The need to file it within such period is
intended to encourage offenders, who are willing to be reformed and rehabilitated, to avail
themselves of probation at the first opportunity.
Furthermore, the application for probation must necessarily fail, because before the application
was instituted, petitioner already filed a Notice of Appeal before the RTC on 17 June 2003. The
Probation Law is patently clear that "no application for probation shall be entertained or granted
if the defendant has perfected the appeal from the judgment of conviction."
The law expressly requires that an accused must not have appealed his conviction before he can
avail himself of probation. This outlaws the element of speculation on the part of the accused --
to wager on the result of his appeal -- that when his conviction is finally affirmed on appeal, the
moment of truth well-nigh at hand and the service of his sentence inevitable, he now applies for
probation as an "escape hatch," thus rendering nugatory the appellate court’s affirmation of his
conviction. Consequently, probation should be availed of at the first opportunity by convicts who
are willing to be reformed and rehabilitated; who manifest spontaneity, contrition and remorse.

196
CRIMINAL LAW I – DIGESTED CASES BY: BOAQUINA, MARY CRISCHAEL
197

This was the reason why the Probation Law was amended, precisely to put a stop to the practice
of appealing from judgments of conviction even if the sentence is probationable, for the purpose
of securing an acquittal and applying for the probation only if the accused fails in his bid.

Civil Liability (Articles 100-113)

Subsidiary Liability

BASILIO V. CA
G.R. No. 113433; March 17, 2000

FACTS: Simplicio Pronebo, driver of a dump truck owned and registered in the name of Luisito
Basilio, was charged with reckless imprudence resulting in damage to property with double
homicide and double physical injuries resulting from hitting/bumping and sideswiping: a) a
motorized tricycle; an automobile Toyota Corona; a motorized tricycle; an automobile
Mitsubishi; and a Ford Econo.
The trial court also found that at the time of the vehicular accident accused Simplicio Pronebo
was employed as the driver of the dump truck owned by petitioner Luisito Basilio. Consequently,
Luisito Basilio filed with the trial court a “Special Appearance and Motion for Reconsideration”
praying that the judgment be reconsidered and set aside insofar as it subjected him to a
subsidiary liability for the civil aspect of the criminal case. The motion was denied for lack of merit
and for being filed beyond the reglementary period.
Private respondent filed a Motion for Execution of the subsidiary civil liability of Basilio which
was granted. Basilio filed an appeal before the CA which was denied for lack of persuasive force
and effect. Hence, this petition for review.
ISSUE: Whether or not Basilio can be made subsidiarily liable for the civil indemnity of Simplicio,
driver
HELD: The statutory basis for an employer’s subsidiary liability is found in Article 103 of the
Revised Penal Code. This liability is enforceable in the same criminal proceeding where the award
is made. However, before execution against an employer ensues, there must be a determination,
in a hearing set for the purpose of the following:
1) the existence of an employer-employee relationship;
2) that the employer is engaged in some kind of industry;

197
CRIMINAL LAW I – DIGESTED CASES BY: BOAQUINA, MARY CRISCHAEL
198

3) that the employee is adjudged guilty of the wrongful act and found to have committed the
offense in the discharge of his duties (not necessarily any offense he commits “while” in the
discharge of such duties); and
4) that said employee is insolvent.
There are two instances when the existence of an employer-employee relationship of an accused
driver and the alleged vehicle owner may be determined. One, during the criminal proceeding,
and the other, during the proceeding for the execution of the judgment. In both instances,
petitioner should be given the opportunity to be heard, which is the essence of due process.
Petitioner knew of the criminal case that was filed against accused because it was his truck that
was involved in the incident. Further, it was the insurance company, with which his truck was
insured, that provided the counsel for the accused, pursuant to the stipulations in their contract.
Petitioner did not intervene in the criminal proceedings, despite knowledge, through counsel,
that the prosecution adduced evidence to show employer-employee relationship. With the
convict’s application for probation, the trial court’s judgment became final and executory. All
told, it is our view that the lower court did not err when it found that petitioner was not denied
due process. He had all his chances to intervene in the criminal proceedings, and prove that he
was not the employer of the accused, but he chooses not to intervene at the appropriate time.

PHIL. RABBIT BUS LINES, INC. V. MANGAWANG


G.R. No. 160355 May 16, 2005

FACTS: This is a petition for review of the Decision of the Court of Appeals affirming the Decision
of the Regional Trial Court (RTC) convicting the accused Ernesto Ancheta of reckless imprudence
resulting in homicide. Ernesto Ancheta was employed by the Philippine Rabbit Bus Lines, Inc.
(PRBLI) as driver of one of its passenger buses. On July 23, 1993, an Information was filed with
the RTC of Capas, Tarlac, Branch 66, charging Ancheta with reckless imprudence resulting in
homicide. The accused was assisted by Atty. Crispiniano Lamorena, Jr., whom the PRBLI assigned
as counsel de parte. Atty. Andres Pangilinan entered his appearance as private prosecutor.
The trial court rendered judgment convicting the accused of the crime charged. The accused
appealed the decision to the CA. CA dismissed the appeal due Ancheta’s failure to file his brief as
accused-appellant. The resolution of the CA dismissing the appeal became final and executor.
The PRBLI, as Ancheta’s employer, filed a Notice of Appeal of the decision of the RTC claiming
among others that the decision of the trial court convicting the employee is binding and
conclusive upon the employer not only with regard to the civil liability but also, with regard to its
amount," should not apply in the case because they are not served with a copy of the decision of
the RTC; hence, could not have appealed the same.
ISSUE: Whether or not PRBLI can be held subsidiarily liable for the offenses committed by the
employee

198
CRIMINAL LAW I – DIGESTED CASES BY: BOAQUINA, MARY CRISCHAEL
199

HELD: Yes. The petitioner, as the employer of the said accused, had no right to appeal from the
said decision because, in the first place, it was not a party in the said case. While the subsidiary
liability provided for by Articles 102 and 103 of the Revised Penal Code may render the petitioner
a party in substance and, in effect, it is not, for this reason, entitled to be furnished a copy of the
decision of the RTC, as well as the resolution and decision of the CA.
It is true that an employer, strictly speaking, is not a party to the criminal case instituted against
his employee but in substance and, in effect, he is considering the subsidiary liability imposed
upon him by law. It is his concern, as well as of his employee, to see to it that his interest be
protected in the criminal case by taking virtual participation in the defense of his employee. He
cannot leave him to his own fate because his failure is also his. And if because of his indifference
or inaction the employee is convicted and damages are awarded against him, he cannot later be
heard to complain, if brought to court for the enforcement of his subsidiary liability, that he was
not given his day in court.
In Ozoa v. Vda. de Madula, the Court explained the effect of a judgment of conviction against the
employee on the subsidiary liability of the employer, as follows: To be sure, the correctness of
the legal principles cited by the Court a quo cannot be gainsaid. A person criminally liable is also
civilly liable; and upon the institution of the criminal action, the civil action for the recovery of
the civil liability arising from the crime is also impliedly instituted unless waived, or the filing of a
separate action therefor is reserved. The employer is subsidiarily answerable for the adjudicated
civil liability ex delicto of his employee in the event of the latter’s insolvency; and the judgment
in the criminal action pronouncing the employee to be also civilly liable is conclusive on the
employer not only as to the actuality of that liability but also as to its amount.

Persons Civilly Liable for Felonies

DIONISIO CARPIO V. DOROJA


G.R. No. 84516 Dec. 5, 1989

FACTS: Accused Edwin Ramirez is the driver of a passenger jeepney owned and operated by one
Eduardo Toribio. While driving said vehicle, Ramirez bumped into DionisioCarpio while the latter
was crossing the street. This caused him to suffer a fractured left clavicle requiring medical
attention for 3 months. Accused was convicted of reckless imprudence resulting in less serious
physical injuries after the accused pleaded guilty to said lesser offense.
After issuance of a writ of execution for the civil liabilities, the same was returned unsatisfied due
to the fact that accused was an indigent. Herein petitioner moved for the issuance of a subsidiary
writ of execution against the subsidiary liability of the owner-operator. This was denied by the
trial court on the following grounds: one, the decision did not mention of the subsidiary liability

199
CRIMINAL LAW I – DIGESTED CASES BY: BOAQUINA, MARY CRISCHAEL
200

of one Eduardo Toribio; two, the nature of the accident is one of culpa aquiliana and not culpa-
contractual.
ISSUE: Whether or not the subsidiary liability of the owner-operator may be enforced in the same
criminal proceeding against the driver where the award was given, or in a separate civil action.
HELD: The Court ruled that a separate civil action is unnecessary. The Court ruled that the
subsidiary liability of an employer arises after conviction of the employee in a civil action, and
that the following were proved during trial: (1) that the employer, etc. is engaged in any kind of
industry, (2) that the employee committed the offense in the discharge of his duties and (3) that
the employee is insolvent. Once all requisites are met, which was so in this case, the employer
becomes ipso facto subsidiarily liable without need of a separate action.
The case at hand is not one where the operator-owner is being sued for primary liability under
the Civil Code. The operator-owners liability is incident and dependent upon the employee’s
criminal liability. The subsidiary liability in Art. 103 should be distinguished from the primary
liability of employers, which is quasi-delictual in character as provided in Art. 2180 of the New
Civil Code. Under Art. 103, the liability emanated from a delict. On the other hand, the liability
under Art. 2180 is founded on culpa-aquiliana. The present case is neither an action for culpa-
contractual nor for culpa aquiliana. This is basically an action to enforce the civil liability arising
from crime under Art. 100 of the Revised Penal Code.

What Civil Liability Includes

HEIRS OF RAYMUNDO CASTRO V. BUSTOS


GR No. L-25913 ; February 29, 1969

FACTS: Trial court found Bustos guilty only of homicide and, crediting him with two mitigating
circumstances, namely, passion or obfuscation and voluntary surrender, and ordered him to
indemnify the petitioners six thousand pesos. Court of appeals modified the judgment and
awarded to the heirs of the deceased moral damages in the amount of P6,000 plus P13,380.00
to compensate for the loss of earning of the decedent at the annual salary of P2,676.00, which
they later eliminated upon motion for reconsideration of the respondent. Petitioner appealed
that the previous decision of the Court of Appeals be reinstated in toto.
ISSUE: Whether appellant are entitled to moral damages and compensation for loss of earning
capacity.
HELD: YES. Art. 2206 provides that defendant shall be liable for the loss of the earning capacity
of the deceased, and the indemnity shall be paid to the heirs of the latter; such indemnity shall
in every case be assessed and awarded by the court; as well as for moral damages for mental
anguish caused to the heirs of the deceased. Even if compensation for loss of earning capacity is
not pleaded in the complaint, such may be awarded as it is deemed included the prayer for

200
CRIMINAL LAW I – DIGESTED CASES BY: BOAQUINA, MARY CRISCHAEL
201

"actual damages" and for other "just and equitable reliefs", especially if taken in the light of Art.
2206, in connection with Art. 1764, of the Civil Code.
When the commission of a crime results in death, the civil obligations arising therefrom are
governed by the penal laws. The amount of damages for death caused by a crime or quasi-delict
shall be at least twelve thousand pesos, even though there may have been mitigating
circumstances and should include loss of earning capacity of the deceased and moral damages.

Extinction of Civil Liability

PEOPLE V. BAYOTAS
G.R. No. 102007 September 2, 1994

FACTS: A criminal case for rape was filed against the accused Rogelio Bayotas. He was also
convicted by the trial court. However, pending appeal of his conviction he died at the National
Bilibid Hospital due to Cardiorespiratory arrest secondary to Hepatic Encephalopathy secondary
to hipato carcinoma gastric malingering. The solicitor general, in his comment, contends that
death of the accused did not extinguished his civil liability as a result of his commission of the
offense charged. Counsel for the accused on the other hand, argued that the death of the accused
extinguishes both his criminal and civil liability.
ISSUE: Whether the civil liability of the accused is extinguished by reason of his death pending
appeal of judgment of conviction.
HELD: Yes. Death of the accused pending appeal of his conviction extinguishes his criminal liability
as well as the civil liability based soley thereon. The claim for civil liability on the other hand,
survives notwithstanding the death of the accused if the same may also be predicated on a source
of obligation other than the delict (law, contracts, quasi- contracts and quasi- delicts). In this case,
the death of the accused Bayotas extinguished his criminal liability and the civil liability based
soley on the act complained of which is rape. Consequently, the appeal is hereby dismissed
without qualification.

201

You might also like